AP Inter 1st Year Sanskrit Model Paper Set 7 with Solutions

Self-assessment with AP Inter 1st Year Sanskrit Model Papers Set 7 allows students to take charge of their own learning.

AP Inter 1st Year Sanskrit Model Paper Set 7 with Solutions

Time : 3 Hours
Max Marks : 100

सूचना : प्रथम, द्वितीय, तृतीयप्रश्नान् अनुवादप्रश्नं च विहाय अन्ये प्रश्नाः संस्कृतभाषायामेव समाधातव्याः ।

1. एकं श्लोकं पूरयित्वा भावं लिखत । (6 × 1 = 6 M)

अ) दुर्जनः परिहर्तव्यो ……………. किमसौ न भयङ्करः ॥
ज.
दुर्जनः परिहर्तव्यो विद्ययाऽलङ्कतोऽपि सन् ।
मणिना भूषितः सर्पः किमसौ न भयङ्करः ||
A wicked person, even though adorned with education, should be avoided. Is not a serpent terrible even though bedecked with a jewel ?

आ) चला लक्ष्मीः ……………. एको हि निश्चलः ||
ज.
चला लक्ष्मीः, चलाः प्राणाः, चलं जीवितयौवनम् ।
चलाचले च संसारे धर्म एको हि निश्चयः ॥
Wealth is impermanent, life is impermanent, youth is impermanent, and in this impermanent world, dharma alone is permanent.

II. एकं निबन्धप्रश्नं समाधत्त । (6 × 1 = 6 M)

अ) दशरथस्य पश्चात्तापः इति पाठस्य सारांश लिखत ।
ज.
Introduction : The lesson दशरथस्य पश्चात्ताप: is an extract from the Ayodhya Kanda of Ramayana, written by Sage Valmiki. Rama went to the forest at the command of Kaikeyi. Dasaratha could not bear separation from Rama. Then he told Kausalya that he was cursed by a sage during his youth.

Dasaratha’s Lamentation :
Dasaratha told Kausalya that one would receive the good or bad fruits of one’s own deeds only. Dasaratha committed a sin during his youth. He was famous for his ability to shoot at sound. He received the fruit of his action just as a boy who had eaten poison out of ignorance.

Going for hunting at night:
Once, when it was rainy season, Dasaratha went to the bank of river Sarayu at night for hunting. There, he heard the sound of a pitcher being filled with water. He mistook it for the sound made by an elephant and released an arrow towards that sound. अचक्षुर्विषये घोषं वारणस्येव नर्दतः ।

The painful human cry :
Then he heard a human voice crying how anyone could hit a hermit, who came to the river to carry water. He grieved for his old parents. It was a cruel stupid, who killed all of them.

Dasaratha found a boy :
Dasaratha rushed to the bank of the river to find a hermit hit by his arrow. His hair was scattered, and the water pitcher fell at a distance. The boy asked the king what harm he did to him. He was about to carry water to his parents. Dasaratha killed his parents also who were weak and blind. He asked Dasaratha to carry the news to his father. पितुस्त्वमेव मे गत्वा शीघ्रमाचक्ष्व राघव ।

The old and blind couple :
Carrying the water pot, Dasaratha went to the hermitage and saw the weak and blind parents of the boy who were without any support, and who looked like birds with clipped wings. The old man thought that it was his son, and asked him to give him some water. Then Dasaratha said that he was not their son, and he was Dasaratha, a Kshatriya. He told them that he went at night to the bank of the river Sarayu to kill any wild animal or an elephant that came to drink water. He mistook the sound of filling a pitcher with water for that of an elephant, and killed it with an arrow. द्विपोऽयमिति मत्वाऽयं बाणेनाभिहतो मया । But he saw their son with the arrow struck in the heart.

The curse uttered by the old man :
On hearing those cruel words, the sage could not control his emotion. He cursed Dasaratha saying that he would die lamenting for his son. एवं त्वं पुत्रशोकेन राजन् कालं करिष्यसि ।

आ) “अमरवाणीप्रशस्तिः” इत्यस्य पाठ्यभागस्य सारांश लिखत ।
ज.
Introduction: The lesson अमरवाणीप्रशस्तिः is taken from सुरवाणीमणिहारः written by Sri E. Sathakopacharya. The greatness of Sanskrit, the language of the gods is described in a beautiful way in this lesson.

Salutations to Amaravani: The poet salutes to Sanskrit, which is affectionately fondled by Brahma and other gods, praised by Valmiki, Vyasa and other poets, and decorated by Kalidasa and other poets.

The poet says that the speech of the gods has descended to the earth showing compassion on humans, who cannot roam in the heaven or enjoy with the celestial damsels. धरणिममरवाणि प्राप्य नाकीकरोषि ।

He further says that had not the goddess come down thus, men would have fallen in the well of birth and death. Because they would not have known god.

The greatness of Sanskrit: The poet says that Sanskrit is the mother of all languages. भवसि सकलभाषाजन्मदात्री यत्स्तवम् । Other language s shine by their association with Sanskrit just as moon lotuses bloom because of their association with the moon. This language shines with many meanings in every syllable like the ocean with gems. विराजसे अनल्परन्नाद्याश्रयः सागरो यथा | Even if some people criticize her, it does not diminish her greatness. Bursting meteors do not dry up the ocean.

It the greatness of Sanskrit that even after ages passed, one still remembers Valmiki, Vyasa and Kalidasa.

None equals the goddess of speech: Men may make rockets, travel in space reaching the moon or go to the heaven with human bodies, still they pray for the grace of the mother of speech. She subdues the suffering of birth by teaching brahmavidya, delights with the poems of Kalidasa etc. and leads to the path of prosperity by preaching dharma. There is none equal to her. त्वत्समा कास्ति मान्या ।

There may be many languages in the world. But they are of no use. A river which quenches the thirst of the traveller in summer is the best.

The poet asks man that what happens to him if the bombs he is making explode. If one wants to follow the panchasheel of Nehru, then one should resort to Sanskrit that develops the spiritual knowledge.

III. एकं निबन्धप्रश्नं समाधत्त । (6 × 1 = 6 M)

अ) पुण्डरीकमुद्दिश्य कपिञ्जलः किम् उपदिदेश ?
Answer:
Introduction: The lesson Kapinjalopadesa is an extract from the Kadambari, written by Bana. Pundarika, an ascetic fell in love with a celestial nymph Mahaswetha, His friend Kapinjala advised him that it was not proper for him to lose his self control. He asked him to condemn the Love-god.

Kapinjala’s questions :
Kapinjala told Pundarika that it was not proper for Pundarika to behave in such a way as the common men would behave. The ascetics had the wealth of courage. धैर्यधना हि साधवः । He asked Pundarika why he did not restrain himself. How he was attacked by the senses. Where had his courage, self-restraint, calmness, celibacy, learning, disinterestedness and distaste for pleasures gone? If people like him were also sullied by passion then there was no use of intellect, study of the scriptures, culture, knowledge received from the teachers, and awakening. , निरर्थकः He asked him how did he not notice the crystal rosary that fell from his hand and taken away by that evil woman. He exclaimed that Pundarika was ab- sent minded, and thought of rescuing him before his heart was stolen by that evil woman. अहो विगत चेतनत्वम् ।

When Pundarika accepted that he was attracted by the beauty of Mahaswetha, Kapinjala further advised thus.

Kapinjala’s advice :
He asked Pundarika whether it was taught by his teachers of he studied it in the scriptures. Was it a means to acquire Dharma, a new kind of penance, a mystic vow or path leading to heavens ? Could he even imagine this, let alone tell or see ? Why did he allow the Cupid make him a laughing-stock? Only fools are tor- mented by the Cupid मूढो हि मदनेन आयास्यते Could he have happiness in some- thing condemned by the virtuous and honoured only by the vile ?

He said that he was a fool, who watered a garden of poisonous plants, em- braced a sword creeper, held a black serpent or touched a burning coal thinking them to be something else. If he could not restrain his senses or control his mind, what was the use of his knowledge like that of the light of a firefly ? And he ad- vised him to condemn the stupid Love-god.

आ) वर्षादेव्याः परिदेवनं संक्षेपेण लिखत |
Answer:
Introduction: The lesson Varshaparidevanam was written by Prlf. G.S.R. Krishna Murthy. He wrote Vanaki, Suvarna svachalitam etc. story books. In Varsha- paridevanam, the author describes how the goddess of rain laments at the envi- ronmental pollution.

The Early morning :
Once the early mornings were filled with the cooing of the cuckoos, chirping of the birds, fragrant breezes etc. Every where there was purity. All were pure in body, mind and speech. But time changed. Roads became dusty. Vehicle and factory were emitting smoke. Garbage piled up. Culverts were clogged. People suffered from epidemics. Even the animals and the trees suffered. The goddess of rain was thinking about that. She requested the sun and saw that clouds were formed, and there was rain. She swept the universal yard, and decorated it with the rainbow.

IV. चतुर्णां प्रश्नानां समाधानानि लिखत । (4 × 2 = 8 M)

अ) चाणक्यः कस्य राज्ञः अमात्यः ?
जवाब:
चाणक्यः चन्द्रगुप्तस्य राज्ञः अमात्यः । सः नन्दवंशम् उन्मूल्य चन्द्रगुप्तं सिंहासने अध्यारोपयत् ।

आ) राजसान्नवाच्याः के भवन्ति ?
जवाब:
अतिकषायपदार्थः, अतिलवणम्, अत्युष्णापदार्थः अतिक्षारपदार्थः, अत्याम्लपदार्थः, अतिक्षातिदाहपदठार्थाः राजसान्नवाच्या भवन्ति ।

इ) बोपदेवविरचितः व्याकरणशास्त्रग्रन्थः कः ?
जवाब:
बोपदेवविरचितः व्याकरणशास्त्रग्रन्थः मुग्धबोधः ।

ई) कृपणस्य अग्रे गगनं दृष्ट्वा सर्वे किं कृतवन्तः ?
जवाब:
कृपणस्य अग्रे गमनं दृष्ट्वा सर्वे अट्टहासं कृतवन्तः ।

उ) नागार्जुनः कः ?
जवाब:
नागार्जुनः रसायनशास्त्रज्ञः प्रसिद्धः चिकित्सकः च ।

ऊ) पेटिकायां व्यापारी किम् अपश्यत् ?
जवाब:
पेटिकायां व्यापारी एवं जीर्णं धौतवस्त्रम् एकं प्राचीनं करांशुकम् एकं पुरातनं पादत्राणम् च अपश्यत् ।

V. द्वयोः संदर्भ व्याख्यानं लिखत ।

अ) अचक्षुर्विषये घोषं वारणस्येव नर्दतः ।
जवाब:
परिचयः – एतत् वाक्यं दशरथस्य पश्चात्तापः इति पाठ्यभागात् सवीकृतम् । अयं पाठ्यभागः
वाल्मीकिरामायणे अयोद्याकाण्डतः स्वीकृतः ।
सन्दर्भ: – दशरथः कौसल्यां प्रति मुनिशापवृत्तान्तम् कथयन् एवम् उक्तवान् ।
भावः – अदृष्टिगोचरे रात्रौ गजस्य शब्दः इव श्रुतः ।
विवरणम् – दशरथः कौसल्याम् उक्तवान् यत् सः एकदा रात्रौ मृगयार्थं सरयूतीरं गतवान् । तत्र रात्रौ जलेन पूर्यमाणस्य कुम्भस्य शब्दः गजस्य शब्दः इव श्रुतः ।

आ) न रत्नमन्विष्यति मृग्यते हि तत् ।
जवाब:
परिचयः एतत् वाक्यं मायावटुः इति पाठ्यभागात् स्वीकृतम् । एषः भागः कालिदासस्य कुमारसम्भवे पञ्चमसर्गात् स्वीकृतः ।
सन्दर्भः – मायावटुः पार्वतीम् एवं वदति ।
भावः – रत्नं अन्वेषणं न करोति । तत् अन्विष्यते ।
विवरणम् : पार्वत्याः तपोवनं एकः जटिलः प्रविवेश । सः अवदत् यत् यदि पार्वती पतिमिच्छति तपसा अलम् । रत्नं न अन्विष्यति । तत् अन्वेष्यते ।

इ) भवति सकलभाषाजन्मदात्री यतस्त्वम् ।
जवाब:
परिचयः – एतत् वाक्यं अमरवाणीप्रशस्तिः इति पाठ्यभागात् स्वीकृतम् । अस्य कविः श्रीमान् ई. शठकोपाचार्यः ।
सन्दर्भः – कविः अमरवाणीं प्रशंसन् एवं वदति ।
भावः – त्वम् सकलभाषाणां जननी ।
विवरणम् – जगति सर्वाः भाषाः संस्कृतसंपर्केण विराजन्ते । संस्कृतभाषा सकलभाषाजननी ।

ई) यतः प्रकृष्येत मनुष्यवर्गः ।
जवाब:
परिचयः – एतत् वाक्यं कपीनामुपवासः इति पाठ्यभागात् स्वीकृतम् । अस्य कविः श्रीशैलं ताताचार्यः ।
सन्दर्भः – कपिसार्वभौमः अन्यान् कपीन् एवं वदति ।
भावः – मनुष्याः कथं प्रकर्षम् आप्नुवन्ति ।
विवरणम् – कपिसार्वभौमः वदति यत् कपयः लोकत्रये प्रसिध्दाः बभूवुः । परन्तु केन मनुष्याः प्रकर्षम् वति ।

VI. द्वयो: संदर्भ व्याख्यानं लिखत ।

अ) अहो विगतचेतनत्वम् ।
जवाब:
परिचयः एतत् वाक्यम् ‘कपिञ्जलोपदशः इति पाठ्यभागात् स्वीकृतम् । अस्य मूलग्रन्थः कादम्बरी । अस्य कविः बाणः ।
सन्दर्भः महाश्वेतायाम् अनुरक्तम् पुण्डरीकम् सन्मार्गे प्रवर्तयितुं कपिञ्जलः एवम् उपदिशति ।
अर्थः – अहो, अचेतनत्वम् ।
विवरणम् – कपिञ्जलः पुण्डरीकं वदति यत् पुण्डरीकस्य स्थितिः तस्य अनुरूपा न । सः अचेतनः इवास्ति । करतलात् भ्रष्टाम् अक्षमालाम् अपि न लक्षयति ।

आ) कुम्भकारोऽपि यो विद्वान् स तिष्ठतु पुरे मम ।
जवाब:
परिचयः – एतत् वाक्यं भोजस्य औदार्यम् इति पाठ्यभागात् स्वीकृतम् । अस्य कविः बल्लालः ।
सन्दर्भः – भोजः मुख्यामात्यम् एवम् अवदत् ।
भावः – विद्यावान् चेत् कुम्भकारः अपि मम नगरे निवसतु ।
विवरणम् – कदाचित् भोजः मुख्यामात्यम् एवम् अवदत् – अविद्वान् चेत् ब्राह्मणः अपि नगरात् बहिः गच्छेत् । विद्वान् चेत् कुम्भकारः अपि नगरे वसतु ।

इ) पुण्यरूपाणि एतानि पुष्पाणि वन्दे |
जवाब:
परिचयः – एतत् वाक्यं स्वेदस्य पुष्पाणि इति पाठ् यभागात् स्वीकृतम् । अस्य रचयित्री डा. माधवी जोषी ।
सन्दर्भः – रामः शबरीम् एवं वदति ।
भावः – मातङ्गस्य पुण्यरूपाणि एतानि पुष्पाणि नमस्करोमि ।
विवरणम् – शबरी स्वेदपुष्पाणां कथां रामाय अकथयत् । तत् श्रुत्वा रामः तानि पुष्पाणि नमस्कृतवान् ।

ई) स्वच्छतायै प्रयतामहे ।
जवाब:
परिचयः – एतत् वाक्यम् वर्षापरिदेवनम् इति पाठ्यभागात् स्वीकृतम् । अस्य रचयिता आचार्य गुल्लपल्लि श्रीरामकृष्णमूर्तिः ।
सन्दर्भः – स्वच्छतापरिरक्षणार्थं प्रयत्नं करणीयमिति वदन्ती वर्षादेवी एवं प्रबोधयति ।
भावः – स्वच्छतार्थं प्रयत्नं कुर्मः ।
विवरणम् – विश्वं कलुषितम् इति दुःखित्वा वर्षादेवी स्वच्छतापरिरक्षणार्थं जनान् प्रबोधयति । स्वच्छतार्थं प्रयत्नं कुर्मः । यः क्रियावान् स पण्डितः । इति ।

VII. त्रयाणां प्रश्नानां समाधानानि लिखत ।

अ) मुनिः दशरथं किमिति शशाप ?
जवाब:
मुनिः दशरथं राजन् पुत्रशोकेन त्वं कालं करिष्यसि इति शशाप |

आ) धर्मसाधनेषु आद्यं किम् ?
जवाब:
धर्मसाधनेषु आद्यं शरीरम् ।

इ) पुराकृतानि पुण्यानि मानवान् कथं रक्षन्ति ?
जवाब:
पुराकृतानि पुण्यानि मानवान् वने रणे शत्रुजलाग्निमध्ये, समुद्रे, पर्वते, सुप्तमपि, प्रमत्तमपि, विषमस्थितमपि रक्षन्ति ।

ई) किम् अश्वमेधेन समं विदुः ?
जवाब:
दरिद्राय कृतं दानं, शुन्यलिङ्गपूजनम्, अनाथप्रेतदहनं च अश्वमेधसमं विदुः ।

उ) सकलभाषाजन्मदात्री का ?
जवाब:
सकलभाषा जन्मदात्री अमरवाणी ।

ऊ) कोऽपि कपिः कथं मौनं बिभेद ?
जवाब:
जृम्भां विधाय, हस्तं व्यवधूय, पादं प्रसार्य, गात्रं परिवर्त्य, पुच्छाग्रम् आचुम्ब्य कोऽपि कपिः मौनं बिभेद |

VIII. त्रयाणां प्रश्नानां समाधानानि लिखत ।

अ) कपिञ्जलोपदेशं कस्मात् ग्रन्थात् स्वीकृतम् ?
जवाब:
कपिञ्जलोपदेशः कादम्बरी इति गद्यकाव्यात् स्वीकृतः । अस्य काव्यस्य कर्ता बाणभट्टः ।

आ) धनिनां धनं किम् ?
जवाब:
यत् ददाति यत् अश्नाति तदेव धनिनां धनम् ।

इ) तीक्ष्णदंष्ट्रस्य धर्मोपदेशं श्रुत्वा शशकः किमवदत् ?
जवाब:
तीक्ष्णदंष्ट्रस्य धर्मोपदेशं श्रुत्वा शशकः एवम् अवदत्-एष नदीतीरे तपस्वी धर्मवादी तिष्ठति । तदेनं पृच्छावः ।

ई) बालकाः स्थाने स्थाने कानि दृष्टवन्तः ?
जवाब:
बालकाः स्थाने स्थाने सुन्दराणि पुष्पाणि विकसितानि दृष्टवन्तः ।

उ) व्यासाश्रमद्वारा प्रकटिताः ग्रन्थाः के ?
जवाब:
व्यासाश्रमद्वारा प्रकटिताः ग्रन्थाः योगवासिष्ठम्, शङ्करविजयः, पातञ्जल योगदर्शनम्, विवेकचूडामणिः, अष्टावक्रगीता, बृहदारण्यकोपनिषद्, छान्दोग्योपनिषद् इत्यादयः ।

ऊ) स्वच्छताया विषये सर्वकारः किं करोति ?
जवाब:
स्वच्छताया विषये सर्वकारः बहुयन्नान् करोति । नगरपालिकासु, परिशुद्धकर्मचारिणः नियुक्ताः । आकाशवाणी-दूरदर्शनीद्वारा बहु प्रचारयति ।

IX. विरामग्रहणाय अभ्यर्थनपत्रं लिखत ?
जवाब:

गुन्टूर्
दिनाङ्कः 20-11-2018

सविधे –
प्रधानाध्यापकः / प्रांशुपालः
प्रभुत्व (जूनियर्) उच्चमाध्यमिक कलाशाला,
गुन्टूर्
मान्याः !
विषयः विरामस्य कृते अभ्यर्थनम् ।
मम सोदर्याः विवाहार्थम् अहं श्वः स्वग्रामं गमिष्यामि । अतः कृपया दिनत्रयस्य 21-11-2018 तः 23-11-2018 पर्यन्तं विरामं यच्छन्तु इति सादरप्रणामपूर्वकं विज्ञापयामि । मम अनुपस्थितिसमये पाठ्यमानान् पाठान् मित्रसकाशात् ज्ञात्वा अहं पठिष्यामि ।
सधन्यवादम् ।

भवतां विधेयः छात्रः
नाम: × × × × × × ×
अनुक्रमसंख्या × × × × × × ×
प्रभुत्व जूनियर् कलाशाला,
गुन्टूर् ।

(अथवा)

पुस्तकप्रेषणविषये पत्रं लिखत ?
जवाब:

कडप
दिनाङ्कः 20-01-2018

सविधे –
मान्यसञ्चालकाः,
सरस्वतीविद्याप्रकाशन्,
विजयवाटिकॉ
मान्याः !
भवद्भिः प्रकाशितेषु अधोनिर्दिष्टानि पुस्तकानि मया यथानिर्दिष्टम् अपेक्ष्यन्ते ।

क्र.सं पुस्तकनाम कविः प्रतय:
1 कुमारसम्भवम् कालिदासः 5
2 श्रीमद्रामायणम् वाल्मीकिः 4
3 पञ्चतन्त्रम् विष्णुशर्मा 6
4 भर्तृहरिसुभाषितानि भर्तृहरिः 3

कृपया एतानि पुस्तकानि वि.पि. पि. द्वारा अधोसूचितं सङ्केतं प्रति प्रेषयन्तु ।
सधन्यवादम् ।

भवदीयः / भवदीया
नाम …………
गृहसंख्या 2-11-18
गान्धीरोड़, कडप ।

X. द्वयोः शब्दयोः सविभक्तिकरूपाणि लिखत । (2 × 6 = 12 M)

अ) रवि
जवाब:
AP Inter 1st Year Sanskrit Model Paper Set 6 with Solutions 1

आ) नदी
जवाब:
AP Inter 1st Year Sanskrit Model Paper Set 6 with Solutions 2

इ) फल
जवाब:
AP Inter 1st Year Sanskrit Model Paper Set 6 with Solutions 3

ई) किम् (पुं.)
जवाब:
AP Inter 1st Year Sanskrit Model Paper Set 6 with Solutions 4

XI. द्वयोः धातुरूपाणि लिखत | (2 × 3 = 6 M)

अ) लिखेत्
जवाब:
AP Inter 1st Year Sanskrit Model Paper Set 6 with Solutions 5

आ) खादतु
जवाब:
AP Inter 1st Year Sanskrit Model Paper Set 6 with Solutions 6

इ) भवति
जवाब:
AP Inter 1st Year Sanskrit Model Paper Set 6 with Solutions 7

ई) मोदिष्यते
जवाब:
AP Inter 1st Year Sanskrit Model Paper Set 6 with Solutions 8

XII. त्रीणि सन्धिनामनिर्देशपूर्वकं विघटयत ।

अ) वाणीशः
जवाब:
वाणी + ईश = सवर्णदीर्घ सन्धिः

आ) विक्रमोर्वशीयम्
जवाब:
विक्रम + ओर्वशीयम् = गुण सन्धिः

इ) वसुधैव
जवाब:
वसुधा + एव = वृद्धि सन्धिः

ई) वाण्येका
जवाब:
वाणी + एका = यणादेश सन्धिः

उ) पावकः
जवाब:
पौ + अक: = अयवायाव सन्धिः

ऊ) सोऽपि
जवाब:
सः + अपि = पूर्वरुप सन्धिः

XIII. त्रीणि सन्धिनामनिर्देशपूर्वकं सन्धत्त ।

अ) पितृ + ऋणम्
जवाब:
पितॄणाम् = सवर्णदीर्घ सन्धिः

आ) महा + इन्द्राः
जवाब:
महेन्द्रः = गुण सन्धिः

इ) महा + ऐश्वर्यम्
जवाब:
महैश्वर्यम् = वृद्धि सन्धिः

ई) धातृ + अंश:
जवाब:
धात्रंशः = यणादेश सन्धिः

उ) भानो + ए
जवाब:
भानवे = अयवायाव सन्धिः

ऊ) का + अपि
जवाब:
कार्येऽपि = पूर्वरुप सन्धिः

XIV. आन्ध्रभाषायां वा आङ्ग्लभाषायां वा अनुवदत ।

अ) अहं ग्रामं गच्छामि ।
जवाब:
I go to village.

आ) ते पाठम् अपठन् ।
जवाब:
Where did they go.

इ) यूयं कदा खादिष्यथ ।
जवाब:
When will you eat ?

ई) सः नगरं गमिष्यति ।
जवाब:
He will go to city.

उ) सर्वे गुणाः काश्चनमाश्रयन्ते ।
जवाब:
All the virtues depend on gold.

XV. एकेन पदेन समाधत्त ।

अ) दशरथः कां नदीम् अन्वगात् ?
जवाब:
सरयूनदीम् ।

आ) “मायावटुः” इति पाठ्यभागः कस्मात् स्वीकृत: ?
जवाब:
कुमारसम्भवात् ।

इ) घनस्य तृतीया गतिः का ?
जवाब:
नाशः ।

ई) प्रत्यक्षे स्तुत्याः के ?
जवाब:
गुरवः ।

उ) अनरवाणी सर्वमानवान् शुभमार्गे कैः गमयति ?
जवाब:
धर्मसूत्रैः ।

XVI. एकेन पदेन समाधत्त ।

अ) कपिञ्जलः कः ?
जवाब:
पुण्डरीकस्य मित्रम् |

आ) भोजस्य मुख्यामात्यः कः ?
जवाब:
बुद्धिसागरः ।

इ) तीक्ष्णदंष्ट्रः कः ?
जवाब:
अरण्यमार्जारः ।

ई) तृणवत् कान् न गणयेम ?
जवाब:
श्रामिकान् ।

उ) मार्गाः काभिः कलुषिताः ?
जवाब:
धूलिभिः ।

TS Inter 1st Year Political Science Model Paper Set 3 with Solutions

Thoroughly analyzing TS Inter 1st Year Political Science Model Papers Set 3 with Solutions helps students identify their strengths and weaknesses.

TS Inter 1st Year Political Science Model Paper Set 3 with Solutions

Time: 3 Hours
Max. Marks:100

Section – A
3 x 10 = 30 Marks

Note: Answer any THREE of the following questions in not exceeding 40 lines each. Each question carries 10 Marks.

Question 1.
Discuss the relationship of Political Science with History and Economics.
Answer:
Political Science has intimate relation with other social sciences like History and Economics. Such inter-relation between political science and History as well as relation between political science and Economics can be detailed as below.
a. Political Science – History
b. Political Science – Economics.

a. Political Science – History: History describes the past. The development of man-kind and society can be known through History. History being the story of man, functions as a treasure house of human experiences. It is like a laboratory to all social sciences. The political, economic, social, cultural, religious and literary activity of man can be known only through History. It describes different associations of man from earliest times. History conveys information to the present society, the developments in the past in the areas like state, civilization, culture, religion and economic activity. History is a written record of different events, movements, their causes and interrelations. History provides information to study the political activity in the past. The birth and development of political ideas and institutions is known through history.

“History without Political Science has no fruit.
Political Science without History has no root’’

There has been continuous transformation and development of political institutions since the earliest period of History. The evolution of different political institutions through the ages is recorded in History.

History is the foundation of Political Science. A comparative study of the previous political institutions and the contemporary political activity provides a scope to find ideal and state political institutions in future. The knowledge of political activity is very much essential to understanding the events like founding of the Indian National Congress, the French Revolution, the Russian Revolution and the theories like the two Nation Theory of the Muslim League and also to estimate their impact.

In the same way it is possible to study the concepts proposed by Plato. Aristotle and other Philosophers in the light of the knowledge of history of Ancient Europe. Different political thinkers like Machiavelli, Montesquieu and Lord Bryce developed their respective theories basing on the information found in history. As Robbon opined, ills essential for a student to know about the history of his own race to study the constitution and foreign policy of his race.

The knowledge of Political Science is essential to history, in the same way as the knowledge of History to Political Science. History and Political Science can contribute for the development of any civilised society in the spirit of mutual co-operation.

b. Political Science – Economics: Economics studies the aspects like wealth, production, distribution and exchange of goods. It studies about various methods to accumulate wealth. Economics is a sociological study of the aspects like wealth, production and distribution. All the social institutions and political theories place the human life on a right track. A clearly defined political system is very essential for a man to become a good and ideal citizen. Economics helps in different ways to study the human welfare.

Economics tries to co-ordinate the methods of satisfying unlimited wants with limited resources. Lack of peace and dissatisfaction preiai1 in a society when the economic needs are not satisfied.

The Primary needs like food, clothing, shelter, education and medical aid are to be satisfied. Otherwise, life becomes sorrowful. If the basic needs are not satisfied, the individual has to spend all high energy for that purpose only. A poverty-stricken society gives scope for the prevalence of immorality and anti-social elements. An individual suffering from hunger, ignorance, ill-health cannot be in a position to assess his political aims and responsibilities. He resorts to many crimes to satisfy his hunger. Such an individual entertains a spirit to adopt illegal means for his progress. He cannot use his rights properly and discharge his duties. A citizen without basic needs cannot understand the value of right to vote. The communists feel that democracy cannot be successful without a socialist economy. Aristotle warned that economic inequalities lead to social revolutions.

Even though Political Science and Economics are two different disciplines, their common aim is the welfare of people. The policies related to the production, consumption, proper use of exchange units, removing inflation, contributing for the accumulation of national wealth, promotion of industrial development are very much a part of the activities of a modern State. The impact of economic policies is very much considerable on all the above policies. It is quite possible to solve many economic problems through a political system only.

TS Inter 1st Year Political Science Model Paper Set 3 with Solutions

Question 2.
What is sovereignty and explain its characteristic features.
Answer:
Introduction: Sovereignty is the most important characteristic of the modern state. Today all the modern states are the nation states. Sovereignty is their main element. The state is distinguished from other associations or institutions only by sovereign power. The state is qualified to exercise its power on account of sovereignty alone.

Definitions: The word sovereignty has been defined by different writers in different ways. Some definitions are mentioned as follows:
Willoughby: “Sovereignty is the supreme will of the state”.
Blackstone: “Sovereignty is the irresistible, absolute, uncontrolled and authority in which the supreme legal power. resides.”
Jean Bodi: “Sovereignty is the supreme power of the state over citizens and subjects unrestrained by law”. Characteristics of sovereignty:

The following are the main characteristics of sovereignty:

  1. Absoluteness
  2. Universality
  3. Permanence
  4. Inalienability
  5.  Indivisibility
  6. Exclusiveness

1. Absoluteness: Sovereignty is the absolute power of the state within the sphere of the state, there is no other power over and above sovereignty. It is subject to no legal limitations within or without. No other state can control the policies and actions of a particular state. If there are certain boundaries or limitations to the sovereignty of state, they are only self-imposed. But, the absolute sovereign power of the state is questioned by writers like Maitland raised some objections over this characteristic feature of sovereignty. They stated that the various customs and traditions of the people within the state and international obligations outside of the state will act as limitation on the sovereignty of the state.

2. Universality: The sovereignty is universal in character. It is applicable to all persons and organizations within the geographical boundaries of a state. The sovereign power makes no exception, and grants, no exemption to any individual or group. However, the state may provide certain safeguards or exemptions to the diplomats because of the need for promoting peace, friendship and goodwill among the nations on reciprocal basis.

Further, the foreign diplomats are subject to the laws of their own state, therefore, the diplomatic personnel, the foreign tourists, and the foreign heads of the states are given certain immunities. We must also note that the state is free to withdraw the privileges extended to these foreign dignitaries.

3. Permanence: Sovereignty is a permanent feature of a state. Sovereignty continues as long as the state continues in existence. Government may change periodically or frequently. but state will not change because state is permanent and the government is temporary.

To quote Garner, it does not cease with the ‘death or temporary dispossession of a particular bearer or the re organization of the state, but shifts from one point of a physical body to another, when it undergoes external change “According to Grilchrist, It is only a personal change in the government not a break in the continuity of the state”.

This is the reason why people in England used to say ‘The King is dead, long live the Kings’. It means the Kingship is immortal and the king is mortal.

4. Inalienability: Sovereignty is inalienable. It cannot be transferred to another state or organization. The obvious reason is that if the sovereign transfers its supreme power, it no longer remains the sovereign. It can be said that state and sovereignty go hand in hand, no state can claim to remain a state if it has parted and alienated its sovereignty. It is like a ‘soul’ that cannot be separated from body and vice-versa.

5. Indivisibility: Sovereignty cannot be divided. It is indivisible. Division of sovereignty amounts to the destruction of sovereignty. Each state can have only one supreme will. The concept of divided sovereignty is against to the notion of sovereignty. There will be only one sovereign authority in a state. All other organizations and institutions remain subordinate to the state.

Question 3.
Define the term equality? Write about different kinds of equality.
Answer:
Meaning and explanation of Equality: The concept of Equality is of great significance in the study of political science. The term ‘Equality became an important slogan and inspired the people of France, America, Russia and India during their struggle for freedom and independence.

The term Equality’ implies absolute equality of treatment. In Political science, the term ‘Equality’ refers to a state which grants its citizens equality before the law and equal opportunities to develop their personality. But it may be noted that individuals are not equal in many respects. While some of them are strong, some others may be weak. Similarly, some are more intelligent than others. In this way, men differ in many respects. [lence, equality of treatment is not possible. In other words, it implies that state should grant to its citizens equality before law and equal protection by law.

Definitions:

  1. “Equality means first of all the absence of special privilege. In the second place “It means that adequate opportunities are laid open to all”. ‘ – H.J. Laski
  2. “Equality implies equal rights for all the people and abolition of special rights and privileges”. – Barker
    Types of Equality: There are many types of equality. They may be analysed in the following.

1. Social Equality: Social equality stands for equality of status and absence of class distinctions and discrimination. It exists when no individual is made to suffer on account of his caste, class, colour,creed, race etc. For instance, the constitution of India does not recognize caste or class distinctions. In fact it has opposed the practice of untouchability When all citizens enjoy social equality, there will be no discrimination against anyone on the grounds of caste, class, colour, creed, race and place of birth and legal rights. Social equality cannot be achieved only through laws.

There must prevail a sentiment of equality among individuals. The following elements help in achieving and maintaining social equality.

  1. Special privileges should not be allowed by the state to any citizen.
  2. The government shall adopt and implement a uniform policy in the fields of education, employment, administration and legislation.
  3. People must have a broad outlook with the qualities of tolerance, sacrifice, honesty etc., in social and religious affairs.
  4. Citizens of a state should maintain harmonious relations with their fellow citizens.
  5. People should not use their name of the caste, sect or religion at the time of forwarding petitions.
  6. Inter-caste marriages should be encouraged.

2. Economic equality: This kind of equality is a precondition for the enjoyment of social and political equalities. Its absence leads to several social and political problems. Economic equality does not mean equal distribution of social wealth among all the people. It only means the elimination of inequities in wealth, income and property.

The basic needs like food, shelter and clothing must be available to all. It the opinion of Laski, “Economic equality means the abolition of unfettered and irresponsible will in the industrial world’. In Barker’s view, ‘Economic equality is partly a matter of status and partly a matter of property and income”.

Question 4.
What is Democracy? Explain its merits and demerits.
Answer:
Democracy is an important and most significant form of government. The term Democracy” is derived from two greek words namely ‘Demos’ and ‘Kratio’. In greek language Demos means the people and Kratio means power. Hence Democracy means power of the people.

Definitions of Democracy:
Abraham Lincoln: “Democracy is a government of the People, by the people and for the people’.
J.R. Seeley: ‘Democracy is a government in which everyone has a share.”

Merits of Democracy: In modern world almost every state has choose the Democracy because of its merits. So, here we mentioned the merits of the Democracy.
1. Efficient form of Government: In democracy, the representatives of the people will run the administration on efficient lines. As they are responsible and responsive to the people, they always make laws and implement them by consulting various administrative experts and eminent leaders. They also try to minimize the expenditure of the government by following simplicity and economy in administration.

The administrative and executive personnel will be recruited on the basis of merit, efficiency, experience and their service will be utilized in implementing various public welfare programmes.

2. Upholds Individual Liberties: It is only in democracy that the liberties of individuals can be guaranteed to a great extent. Citizens living in democracy enjoy a great amount of liberty. Their rights, liberties and freedoms are incorporated in and guaranteed by a written constitution. Any attempt made to take away or confiscate one’s liberties are forbidden. The various courts preserve, protect and promote individual liberties through their interpretation and analysis of different provisions of the Constitution.

3. Importance to Equality: Democracy gives preference to equality. Democracy próvides equal rights and liberties to all citizens alike in political, economic and social matters. It opposes the practice of untouchability, bonded labour and discrimination on the grounds of religion, region, language, birth etc.

4. Political awareness: Democracy is a large-scale experiment in Public education or political awareness. Election campaigns give education to the masses. At the time of election, the canvassing through press and platform enlightens the mind of the electorate.

5. Promotes Patriotism: Democracy promotes patriotic feelings among the people. People get several opportunities in selecting and electing their representatives to various bodies. They feel conscious and proud in participating in the working of the democratic institutions in a perfect manner. They show enthusiasm by extending their cooperation and support to the nation at the times of crisis. This is evident in the case of Indians during the Chinese and Pakistani invasions at the borders on several occasions.

6. Democracy serves as a school of citizenship: One more advantage of democracy is that it serves as a training school of citizenship. In democracy right to vote, right to contest, right to criticize the government and other political rights make the citizens politically trained in citizenship and other matters.

7. Democracy inculcates moral values: Democracy promotes a better and higher standard of natural character than any other government whatever. It teaches cooperation. responsibility toleration and self-sacrifice. According to Lowell “the best government in the long run is the one that nurtures a people strong in moral fiber, in integrity, industry, self-reliance and courage”.

8. Democracy gives Preference to public opinion: In fact, Democracy is the real representative of the Public opinion. According to Hearen Shaw, “Democracy ties a nerve to every individual; it makes a connection between him and the centre”.

9. It reduces the danger of revolution: Democracy is a government by discussion and persuasion. Every other form of government rests on force. Democracy gives every individual and every group of people a chance to express their views on public matters, to demonstrate their dissatisfaction and resentment against governmental measures and to convert the masses to their own point of view. They have a right to change the government by their vote. So, there is no chance to people opt for revolutionary methods.

10. Symbol of progress and welfare programmes: Democracy is the only form of government which ensures always the progress and welfare of the people.

11. Support World Peace: Democracy promotes world peace and opposed to war and violence. It seeks the change through evolutionary process with the purview of Constitution. Democracy believes in peaceful co-existence and cooperation. According to Burns, ‘Democratic movements have always been movements for peace’.

Demerits of Democracy:
1. Expensive Government: Democracy is the most expensive type of government. A lot of money is spent on election propaganda. Millions of money is spent on the sessions of the national and provincial legislatures. Carlyle said the parliament as a Talking shop’. italian dictator, Mussolini could, therefore, correctly say that democracy is not suited to a poor country.

2. Weak and unstable Government: Democracy is a weak and unstable type of government. In those countries where there are multiple party systems, the coalitions faif frequently and bring instability in the government. For ex; In India 1977 to 79; 1989 to 1991; 1996 to 1999 coalition governments failed to rule the government with stability.

3. Government by the minority: In practical Democracy does not represent majority opinion. Actually, Democracy is said to be government by the majority. This claim on behalf of democracy is not sustainable. In every state a large number of people never exercise their vote.

Besides, in most democratic states the first-past-the-post electoral system prevails (majority vote system). Under this system it often happens that the party securing a minority vote is returned to power. The number of votes secured by the party in power is less than those obtained by the opposition. Democracy thus ceases to be government by the majority.

4. Discouragement to culture, civilization and intellectual development: Democracy is the government by masses and always have a conservative outlook. They show no interest in the development of literature, art and architecture. In Democracy, intellectuals always keeps distance from elections. So many examples are in history defeat of intellectüals in direct elections.

5. Slow decision-making: Democracy is slow decision-making government. Because it takes lot of time to take decision. It delays the work. Legislature process, party system, opposition parties, coalition politics plays key role in decision-making in Democratic governments.

TS Inter 1st Year Political Science Model Paper Set 3 with Solutions

Question 5.
What is Executive? What are its functions?
Answer:
Of the three organs of government, the Executive occupies the most important place. Very often it is referred to as the government. It refers to that branch of government which executes or enforces the laws of the state those are framed by the legislature that is why the executive is identified with government in modern times. Broadly speaking, the executive has three parts:

  1. The Head of the state
  2. The Council of Ministers and
  3. The Employees.

Functions of the Executive:
Defence and Foreign Affairs: The executive takes suitable decisions and implements them in case of any threat to the national independence and sovereignty and protects the country from foreign aggression. The duties of the executive include establishing embassies in others countries, recognising newly formed states and concluding treaties and alliances with the foreign countries.

Administrative functions: The entire administration is carried on in the name of the executive. In this context, its functions include the appointment of the officers in highest ranks allotting their duties, providing training, giving directions to different administrative departments, changing the rules and regulations from time to time, to protect law and order, to appoint and advise the council of ministers in parliamentary system, to create new departments or reorganise the existing ones or abolish them altogether as deemed necessary.

Legislative functions: The executive prepares the draft bills to be presented to the legislature and gets them enacted. In the parliamentary government as the members are directly associated with the legislature the ministers present the draft bills to the legislature and get its acceptanœ. After acceptance by the legislature, the draft bills can become acts with the assent of the chief executive. In the presidential government, the president sends to the legislature for its acceptance certain messages embodying the various legislative measures considered necessary by him. All the bills passed by the legislature can become acts only with the assent of the president. When the legislature does not meet, the executive can issue ordinances.

Financial functions: It is the duty of the executive to prepare the annual budget containing income and expenditure of the government and gets the approval of the legislature. The legislature cannot levy new taxes without the consent of the executive. Levy or abolition of taxes, provision of capital funds, reduction of prices etc. come under the domain of executive responsibility.

Judicial functions: The executive has the certain functions like: To implement the judgements of the courts of law, to reduce or cancel the punishment, to appoint the Judges, to appoint special courts for the enquiry of certain special problems etc. In some of the democratic countries, the chief executive appoints judges of the highest courts of justice.

Welfare functions: It is the Prime duty of the executive in the modem times to undertake welfare measures. The executive has to work for the planned development and contribute for the improvement of the standards of living.

Section – B
8 × 5 = 40 Marks

Note: Answer any EIGHT of the following questions in not exceeding 20 lines each. Each question comes 5 Marks.

Question 1.
Write about the relationship of Political Science with sociology.
Answer:
The knowledge of Sociology is essential to a political scientist. It is essential to study the sociological roots to understand the nature of the State. The Greeks never conceived any difference between society and State. In the opinion of Greek philosophers, State is not only a political system but also a social system of commendable value.

Social customs help to systematise social set up and social life. The political scientists give importance to the trends of socialisation to understand the behaviour of the citizen as an individual and part of a community.

Of late, political sociology developed as a special science. This very factor shows how much functional is the impact of social institutions on political life. Political parties, groups and public opinion are subjected to the influence of social factors. The political changes in a State cannot be understood without the knowledge of the social conditions and institutions. For instance, an indepth study of political activity in India needs a study of the social factors like caste, religion, area and language and also processes related to them.

As Political Science discusses the organised groups of people, Sociology studies both organised and unorganised groups. Political Science deals with all the political institutions in the past, present and future. Sociology discusses the rise and growth of all institutions in a society in the past and present.

Question 2.
What are the differences between State and Association?
Answer:

State Associations
1. The state is permanent. 1. Associations are temporary.
2. The state has sovereign power. 2. Associations cannot have sovereignty.
3. The state has fixed boundaries. No state is universal or worldwide. 3. The associations cannot have fixed territorial boundaries. Some associations are international and universal in character. Ex: U.N.O. Red Cross Society.
4. The membership of state is compulsory. Every citizen naturally becomes the member of the state. 4. But the membership of an association is optical. It depends on the will and wish of the people.
5. A man can become a member of one state only at a time. 5. But they can be member of any number as associations as he desires.
6. The state has multifarious functions concerning almost the whole of man’s life. 6. The functions of an association are singular and common to its members only.
7. The state makes the laws, violation of which is visited by punishment. 7. The associations cannot make laws, but makes their own rules and regulations.

Question 3.
In what way do ‘Nation’ and ‘State’ differ from each other?
Answer:
Several Nation-States came into existence after the two world wars, on the basis of the principle of self-determination. The terms ‘Nation” and “State” were used synonymously. Even the political experts used both these words homogeneously and intermixingly as if both had same meaning. However, in practice both these terms are not same and identical.

Nation: “Nation is a nationality which has organized itself into a political body either Independent or desiring to be independent”.
State: “State is a people organized for law within definite term
Differences: The concepts of Nation and State differ from one another from the following points of view:

Nation State
1. Nation is an independent political community or an integral part of a multinational state. 1. State may consist people of the ‘same or many nations.
2. Nation preceeds the state. 2. State follows the nation. The final form of a nation is the accomplishment of statehood.
3. Nation is historical and cultural in its evolution. 3. State is political and legal structure.
4. Nation is the community of people who exist together for a common goal and who were united by psychological feeling of oneness. 4. State is a people organised by law in a definite territory.
5. Nation is the culmination of a long coexistence of the people. 5. State need not be evolutionary in nature. It may come into existence either by unification of the smaller independent political communities or by participation.

Question 4.
Explain about any three safeguards of Liberty.
Answer:
Introduction: The concept of Liberty is of great significance in the study of political science. Lihetty is an essential condition without which man cannot develop his personality. It became a source of inspiration to the millions of the people living all over the world.

Meaning: The term liberty is derived from the Latin word” LIBER’ which means free from restrictions.

Definition:

  1. ‘Liberty means the absence of restraints”. – J.R. Seely
  2. ‘Liberty means the positive power of doing or enjoying something worth doing or enjoying’. – T.H. Green

Safeguards of liberty: liberty is the most cherished ideal of human beings. Hence, it must be safeguarded in the larger interest of the society and state. In this context, the following safeguards of liberty, are worth mentioning.
1. Democratic rule: Democratic rule is considered as a heaven to liberty, liberty flourishes only in a democratic state. The reason is that democratic state extends protection to individual’s liberties through various laws. It creates a conducive atmosphere for the individuals to enjoy their liberties freely and impartially. It makes the people to participate in the government process directly or indirectly.

It makes the people to participate in the governmental process directly or indirectly. It makes the government answerable to the people. It allows the people the right to change the government through public opinion or ballot when the government acts improperly.

2. Written and rigid constitution: A written and rigid constitution is considered the most important safeguard of individual liberty. Such a constitution incorporates the various freedoms of individuals in several provisions. It acts as a custodian of peoples rights and liberties. It demarcates the spheres of govèrnmental activity.

It mentions about the various measures to be taken in case of peoples freedoms are infringed or confiscated by others including governmental authorities It also imposes restraints on the political parties by not allowing them to amend the constitutional provisions for furthering their partisan interests.

3. Independent judiciary: An independent and impartial judiciary is another safeguard of individual liberty. The judiciary will uphold the constitution and keeps the government accountable to the people. It prescribes various safeguards for protecting the fundamental rights of citizens. The judges in higher courts will deliver justice to the people on fair, free and impartial manner. Pro. Laski, while recognizing his safeguard, stated that good governance depends upon the effective functioning of judiciary.

TS Inter 1st Year Political Science Model Paper Set 3 with Solutions

Question 5.
Discuss Individualism.
Answer:
Meaning: Individualism means the state should leave the individual alone. This theory is also known as the laissez-faire theory. Laissez-faire is a French term which means ‘leave alone’. It regards the individual as the centre of social life. According to this theory, the individual freedom should be given maximum Scope and the state interference should be reduced to the minimum.

The individualists regard state as a ‘necessary evil’. It is necessary because it has to protect the individual from violence and fraud. It is an evil because its existence is a threat to individual freedom. So it is desirable to have state’s interference as little as possible. Lesser the functions performed by the state, the more is the liberty enjoyed by the individual.

The state should perform the following limited functions:

  1. Protection of the individual and of the state from foreign aggression.
  2. Protection of the individual against one another.
  3. Protection of property from robbery and damage.
  4. Protection of individual from false contracts and breach of contracts.

Question 6.
What are the features of Rights?
Answer:
Rights are the essential conditions for the development of the personality of individuals. They are upheld by the laws of the state. Individuals cannot achieve progress in the absence of rights.

Definition: Rights are those conditions of social life without which no man can seek in general to be himself at his best.

Features of Rights: Rights comprise the following features.

  1. Rights are possible only in society: Rights originate in society. They denote human social behaviour. They do not exist outside of the society.
  2. Rights are social in nature: Rights are the claims of individuals. These claims can be established only when the society or the state recognises and maintains them. So they are social in nature.
  3. Rights are inherent in nature: Rights are inherent in the social nature of men. The social contractualists stated that rights are inherent in nature. The in views are accepted to some extent in modern times.
  4. Rights are enforced and protected by the state: Rights are enforced and protected by the state. The various judicial organisations act as the custodians of the rights of individuals. In other words, rights are protected by the courts of law. Individuals enjoy several rights fully only in a democratic state.
  5. Rights are not absolute: Rights are not absolute. Society and state impose some restrictions on the enjoyment of rights by the individuals. These restrictions are meant for maintaining peace and other in the society Further, rights are meant for contributing social welfare and security.
  6. Every right has a corresponding responsibility: Rights and responsibilities are interdependent. Every right has a corresponding responsibility. It is the responsibility of every individual to see that his neighbours also enjoy the same rights. Rights without responsibilities or responsibilities without rights cannot exist. Both are essential for leading a peaceful social life.
  7. Rights are universal: Rights are universal in nature. They are applicable to all. They are given to all without any discrimination.
  8. Rights vary: Rights vary from time to time according to the needs of the people. They also grow with the changes in time and conditions. Some rights which were not found in the past may exist now. The socio-economic, political and cultural conditions will have an influence over the rights.
  9. Rights precede the state: Rights are the products of history. Rights originated in course of time. They were prevalent even before the origin of the state. However, they were guaranteed only after the origin of the state.
  10. Rights are meant for common good: Rights always exist and flourish as long as they are meant for common good. Only those rights which promote common good of the people are recognised by the society and the state. They are essential for leading a prosperous life by the individuals.

TS Inter 1st Year Political Science Model Paper Set 3 with Solutions

Question 7.
Explain any three conditions for securing naturalized citizenship.
Answer:

  1. Residence: An alien who resides in a state for a prescribed period automatically becomes its citizen. Residence in any part of the state is a must for an alien. The period of residence varies from state to state. For instance, it is 5 years in Britain and United States and 10 years in France respectively.
  2. Choice: The children of alien parents could receive citizenship of the state according to their option and choice.
  3. Application: An alien in a state may apply for the citizenship of that state. Then the government of that state considers his application on its merits. It grants citizenship to him with or without some conditions.

These prescribed conditions refer to a minimum period of residence, good moral character, financial capability and knowledge of one of the national languages. Besides, an alien must take an oath of allegiance before he assumes the citizenship of another state.

Question 8.
What is the role of public opinion in Democracy?
Answer:
The term public opinion was coined by philosopher John Locke in the 17th century. In the middle ages, the saying ‘Vox populi; Vox Dei which means voice of the people is the voice of God gained prominence.

Definitions of Public Opinion:

  1. Lord Biyce: “Public opinion is commonly used to denote the aggregate of the views men hold regarding the community’.
  2. R.G.Gettel: Public opinion is the opinion formed by a small group of leaders, which will be accepted by individuals as they have neither the knowledge nor the time nor the interest to enable them to form opinions of their own’.

Role of Public Opinion in Democracy: The term ‘public opinion was coined by philosopher John Locke in the 17th century However, the concept itself predates Locke. Vox Populi or voce of he people is’ a similar Latin concept. Today, public opinion is defined in the following way: collective evaluations expressed by people on political issues, policies, institutions and individuals.

Public opinion changes policy through passive observations that accumulate amongst citizens. Public opinion is important in a democracy because the people are the ultimate source of political power.

In theory, a government official has to take public opinion into account whilst deciding on a future course of action. Political parties must maintain a certain level of positive public opinion which is subject to a certain measure of manipulation through mass media and other sources and used to maintain the status quo. That is, however, not to say that public opinion, itself should be considered a ‘positive social function’.

it is important to understand •the factors that shape public opinion as a whole. These include social class, education, region, age, gender and ethnic, group. Society is not a homogeneous whole. It is made up of components. Each component faces different challenges in its functioning, therefore, members of each component view the world differently.

A state is built upon the consent of the members of society. A constitution is drafted by the collective will of the people. Just as the state and the constitution are vital organic constructs, so is the primary method of informing them: public opinion. Many governments, in one way or the other, try to influence the public by promoting its policies or actions through the mass media.

Question 9.
What are the features of Secular State?
Answer:
Features of Secular State: Secular State comprises the following features:

  1. No place for religion: Secular States does not assign significance to any particular religion. It will not make laws or implement them on religious grounds.
  2. Equal status: Secular Stafe accords equal status to its people. It makes no differentiation between individuals on the grounds of their caste, colour, community; religion, race, region, language etc. As a result, people will have satisfaction and extend co-operation to the government in the implementation of various policies and programmes. They live together with the fellow members of other religious denominations.
  3. No state religion: Secular state does not recognize any particular religion as the state religion. It adopts neutral policy in religious matters.’ It implements various laws and social welfare measures without basing on the religious feelings of the people. It will not assign special role to any particular religion in public activities. All public places like educational institutions, government offices and judicial organizations will carry on their activities without aligning to a particular religion.

Question 10.
Explain the merits and demerits of Unwritten Constitution.
Answer:
Unwritten Constitution: Unwritten constitution is one whose provisions are not written in a single document. It includes several customs and traditions which are manifested in the form of the laws. The Constitution of Britain is the best example of Unwritten constitution.
Merits

  1. An unwritten constitution paves the way for progressive legislation. It has development orientation.
  2. It always undergoes the process of evolution as it aims at ‘bettering the best’.
  3. It gives no scope for revolutions and such other agitations. It concedes to’the popular -demands.
  4. It can be amended according to the popular needs and aspirations.
  5. Its provisions are elastic in nature. So, changes in the constitution are easily made.

Dements:

  1. An unwritten constitution may be changed frequently by the party in power for its political gains. This affects the political stability of the nation.
  2. It fails to protect the rights and freedom of people.
  3. It is more informal in nature.
  4. It is also not suitable for federal states.
  5. An unwritten constitution is considered as a play tool of judges. This may lead to judicial manipulations.
  6. It is prone to Frequent amendments.
  7. It is not suitable to democratic states.

Question 11.
What are the reasons for the decline of the significance of legislature?
Answer:
The political thinkers opine that the legislature, which has wide powers theoretically, is losing its significance gradually in the democratic countries.

There are many reasons for this.

  1. The executive acquired a lot of constitutional and legal powers to achieve comprehensive Socio-economic development.
  2. The legislature has to depend on the executive as it does not possess even the fundamental capacity to understand the novel measures introduced in the administrative system due to the advancement of science and technology.
  3. Since the highest political offices are filled in through direct elections, the government gained importance by directly influencing the people. It could develop direct relations with the people lessening the importance of the legislature.
  4. Another reason for the reduction of the importance of the legislature is delegated legislation. The legislature, passes a law in a skeleton form and delegates power to the executive to fill in the details of it in the shape of rules and regulations. This is called delegated legislation. This power is used by the executive to implement plans and welfare schemes.
  5. Other reasons the political thinkers are worried about the fact that the significance of the legislature is further reduced due to certain reasons like lack of a powerful opposition, less duration of the sessions of the parliament, the irresponsible behaviour of the ruling party and also the misbehaviour of the legislators.

TS Inter 1st Year Political Science Model Paper Set 3 with Solutions

Question 12.
Write a note on modem classification of Governments.
Answer:
In modern times, the Governments have been classified into various forms by different political scientists of a different point of time based on nature of exercise of power.
The modem classification of governments broadly consists of two types.

  1. Despotic Government,
  2. Democratic Governments.

Democratic governments are further divided into limited monarchical form of government and republican form of Government. The Governments are either in the form of unitary or federal based on territorial division of powers and presidential or parliamentary form of Government based on division of powers between the organs of Government.
TS Inter 1st Year Political Science Model Paper Set 3 with Solutions 1
Despotic Government: Despotism is a form of government in which a single entity rules with absolute power and its other connotations are tyranny and dictatorship. The despotic ruler rules at his will and pleasure without any concern for public opinion and welfare.

Democratic Government: Democratic government is a government in which all the people participate in the decision making and it is run as per the aspirations of all the groups. It aims and prdvided for equality.

Section – C
15 × 2 = 30 Marks

Note: Answer any FWfEEN of the following questions in not exceeding 5 lines each. Each question carries 2 Marks.

Question 1.
Aristotle.
Answer:
The Ancient Greek philosopher Aristotle is regarded as the Father of political Science because of his objective and scientific study of the affairs of the government and politics. Aristotle used the term Politics’ to designate the science of the state. He called politics a Master Science. He was the disciple of plato.

Question 2.
Mention any four points on significance of Political Science.
Answer:

  1. Political Science explains concepts and theories.
  2. Political Science enlightens on Rights and Duties.
  3. Political Science provides knowledge of the political thinkers.
  4. Political Science deals with International Relations.

Question 3.
Internal Sovereignty.
Answer:
Internally sovereign has supremacy over all other institutions and associations within the Territorial limits of that state. It also implies that sovereign power is above all laws in the state. All Liberties and Rights of Individuals in the state are only due to sovereignty.

Question 4.
Write any two differences between Nations and States.
Answer:

Nation State
1. Nation is an independent political community or an integral part of a Multi-National state. 1. State may consist of the people of the same Nation or many nations.
2. Nation is Historical and cultural in its evolution. 2. State is a political and Legal structure.

Question 5.
Write any two sources of Law.
Answer:
Professor T.E. Holland mentioned six sources of law.
They are:

  1. Customs
  2. Religion
  3. Judicial decisions
  4. Scientific commentaries
  5. Equity and
  6. Legislature.

Question 6.
What is positive Liberty?
Answer:
It is understood as creating necessary conditions and removing all possible constraints for the development of human beings by the intervention of the state. It is called as positive liberty.

TS Inter 1st Year Political Science Model Paper Set 3 with Solutions

Question 7.
What is uni-polar world?
Answer:
Unipolarity in international politics is a distribution of power in which one state exercises most of the cultural, economic and military influence unipolar systems possess only one great power and face no competition. The post-Cold war international system is unipolar The United States defence – spending is close to half of global military expenditures. The United States of America (USA) is playing a dominant role in the affairs of the United Nations Organisation (UNO) and even in the world affairs since the end of the Cold War in 1991.

Question 8.
Capitalism.
Answer:
Capitalism is an economic system based on the private ownership of the means of production and their operation for profit. Characteristics central to capitalism include private property, capital accumulation, wage labour, voluntary exchange, a price system and competitive markets. In a capitalist market economy, decision-making an Investments are determined by every owner of wealth, property or production in ability in financial and capital markets.

Question 9.
Laissez-Faire.
Answer:
Laissez Faire is a French term which means leave alone It regards the Individual as the centre of social life. According to his theory, the Individual freedom Should be given maximum scope and the state interference should be reduced to the minimum.

Question 10.
What are Civil Rights?
Answer:
Civil rights aim at providing basic conditions for individuals to lead a happy and dignified social life. These rights are considered vital for a civilized society. Civil rights are described as the gift of democracy. Democracy flourishes when the citizens are provided these rights. Civil rights are those rights which provide opportunity to each person to lead a civilized social life. These fulfil basic needs of human life in society. Right to life, liberty and equality are civil rights. Civil rights are protected by the state.

Question 11.
What does the terms Jus Soil mean?
Answer:
Jus Soil means acquisition of citizenship by the principle of place of birth. According to this method, a child acquires the citizenship, of a State, where it borns. It is the place of birth which determines citizenship. This method is not more popular in modern times. At present, this method is observed exclusively in Argentina.

Question 12.
What is initiative?
Answer:
An initiative is a request made by the people to the legislature in framing a law on certain national problem or policy as such. After make in the law, the same shall be presented for referendum: In this aspect at, people in a specified number present a petition in written form to the legislature proposing a legislation. It is also of two kinds. They are

  • Formulative initiative
  • Non-formulative initiative.

Question 13.
What is plebiscite? .
Answer:
A plebiscite means ascertaining public opinion on certain important issues. This is not applicable to the laws and the Constitution. People’s verdict is sought on certain public problems and policies of the government. This method was first used in 1804 by Napoleon in France.

Question 14.
What is the meaning of Secular State?
Answer:
The term ‘Secular’ in Latin language means “of this world”. It denotes the meaning “the opposite of religion”. The english word ‘Secular’ comes from the Latin ‘Saecbllum’ which means “An age” or ‘‘the spirit of an age”. According to E.S. Waterhouse, “Secularism is an Ideology which provides a theory of Life and conduct as against one provided in religion.

TS Inter 1st Year Political Science Model Paper Set 3 with Solutions

Question 15.
What is a Written Constitution?
Answer:
A written constitution is formulated and adopted by a constituent Assembly. It comprises several principles and rules of the government in a written form or document. The constitution of India is an example of written constitution. The American Constitution is the first written constitution in the world.

Question 16.
Senate.
Answer:
Senate is the Upper House of United States Congress, which is a small group of elected people who decide the laws of the country. It consists of 100 members. Every U.S. state elects two people to represent them in the U.S. Senate. These people are called senators.

Question 17.
Independent Judiciary.
Answer:
Independence of Judiciary implies an opportunity to the Judges to perform their duties without fear or favour and act impartially. The Judges should have no relation with the Legislature and the Executive. Both the organs should not interfere in the functioning of the Judiciary. It should be protected from political pressure and influence. Otherwise the very purpose of Judiciary stands defeated.

Question 18.
Parliamentary Executive.
Answer:
A parliamentary government is one in which the executive ie., the council of Ministers headed by the Prime Ministër owes to the legislature for Its Formation. Continuation; and Survival in office. It is also known as Responsible government, cabinet government, Prime Ministerial government etc., Britain is a classical example of this form of government. Besides many states like Australia, Canada, india, Japan etc. have been following this system.

Question 19.
Unitary form of Government.
Answer:
The word Unitary consists of two words, namely, ‘Uni’ and Tary’, uni means one and tary means ‘rule’. A unitary Government is a single integrated government with all executive powers. The Constitution vests all powers in the Central Government.

Question 20.
What is public opinion?
Answer:
Public opinion occupies an important place in democratic states. People express their opinion in times of elections or through the legislatures. They obey and follow those laws which are formulated in accordance with the public opinion. If the government acts against the wishes of people it has to face the consequences. Public opinion keeps the government responsible and responsive. It keeps the government alert and vigilant in its functioning.

AP Inter 1st Year Zoology Question Paper March 2016

Varied difficulty levels in AP Inter 1st Year Zoology Model Papers and AP Inter 1st Year Zoology Question Paper March 2016 cater to students with diverse academic strengths and challenges.

AP Inter 1st Year Zoology Question Paper March 2016

Time : 3 Hours
Max. Marks: 60

General Instructions:
Note : Read the following instructions carefully.

  1. Answer All questions of Section – A. Answer any six questions in Section – B and answer any two questions in Section – C.
  2. In Section – A. questions from Sr. Nos. 1 to 10 are of Very Short Answer Type. Each question carries two marks. Every answer may be limited to 5 lines. Answer all these questions at one place in the same order.
  3. In Section – B, questions from Sr. Nos. 11 to 18 are of Short Answer Type. Each question carries four marks. Every answer may be limited to 20 lines.
  4. In Section – C, questions fromSr. Nos. 19 to 21 are of Long Answer Type. Each question carries eight marks. Every answer may be limited to 60 lines.
  5. Draw labelled diagrams wherever necessary in Sections – B and C.

Section – A (10 × 2 = 20)

Note : Answer all the questions in 5 lines each.

Question 1.
What is meant by tautonymy ? Give two examples.
Answer:
The practice of naming the animals, in which the generic name and species name are the same, is called tautonymy. So the name is called tautonymy. Eg : Naja naja (Indian Cobra), Axis axis (spotted deer)

Question 2.
Why is the true coelom considered a secondary body cavity ?
Answer:
During the embryonic development of the eucoelomates the blasto coel is replaced by coelom derived from the mesoderm vitrue. So, the true coelom is also called ‘Secondary bodycavity”.

Question 3.
Mention any two substances secreted by mast cells with their functions.
Answer:

  1. Heparin – an anticoagulant (prevents blood clotting).
  2. Histamine – vasodilators (cause inflammation in response to injury and infection).

Question 4.
What is the strongest cartilage ? In which regions of the human body, do you find it ?
Answer:
Fibrous cartilage is the strongest of all types of cartilage. It occurs in the intervertebral discs and public symphysis of the pelvis.

AP Inter 1st Year Zoology Question Paper March 2016

Question 5.
Which arthropod, you have studied, is called a “living fossil”? Name its respiratory organ.
Answer:
Limulus (king crab) belonging to class xiphisura of sub phylum Chelicerata is called living fossil. Its respiratory organs are book gills.

Question 6.
Name two poisonous and nonpoisonous snakes found in south India.
Answer:

  1. Two poisonous snakes found in south India are a) Naja naja (cobra) b) Bungarus (krait)
  2. Two non poisonous snakes found in south India are a) Ptyas (rat snake) b) Tropidonotus (pond snake).

Question 7.
Distinguish between lobopodium and filopodium. Give an example of each.
Answer:
a) Blunt finger like pseudopodia are called lobopodium and are seen in Amoeba and Entamoeba.
b) Fibre like pseudopodia are called filopodium and are seen in Euglypha.

Question 8.
What is a kinety ?
Answer:
A longitudinal row of kinetosomes together with kinetodesmata constitute a unit called kinety.

Question 9.
From which substances ‘Smack’ and ‘Coke’ are obtained ?
Answer:
a) Smack is chemically di acetyl morphine obtained by the acetylation of morphine extracted from dried latex of unripe seed capsule of poppy plant.
b) Coke or crack is obtained from coca plant Erythroxylum cocacommonly called cocaine.

Question 10.
Which air pollutants are mainly responsible for acid rains ?
Answer:
Sulphur dioxide and nitrogen oxides are the major causes of acid rains.

AP Inter 1st Year Zoology Question Paper March 2016

Section – B (6 × 4 = 24)

Note : Answer any six questions in 20 lines each.

Question 11.
What is the ‘evil quartet’ ?
Answer:
The following are the four major causes for accelerated rates of species extinction in the world. These causes are referred to as evil quartet.
a) Habitat loss and fragmentation : These are most important reasons for the loss of biodiversity.
b) Over exploitation : When need turns to greed, it leads to over exploitation.
c) Invasion of Alien species : When Alien species are introduced into a habitat, they turn invasive and establish themselves at the cost of indigenous species.
d) Co – extinctions : In an obligate association between a plant and an animal, if a plant becomes extinct, the animal also becomes extinct as seen in a parasitic and host association.

Question 12.
Explain Haversian system.
Answer:
In a bone between the outer and inner circumferential lamellae, there are many Haversian systems. The spaces between the Haversian systems are filled with interstitial lamellae. Haversian system consists of a Haversian canal that runs parallel to the marrow cavity. It contains an artery, a vein and a lymphatic vessel. Haversian canal is surrounded by concentric lamellae. Small fluid filled spaces called ‘lacunae’ provided with minute canaliculi lie in between the lamellae. Canaliculi connect the lacunae with one another and with Haversian canal. Each lacuna encloses one osteocyte (inactive form of osteoblast).

The cytoplasmic processes of osteocytes extend through canaliculi. A Haversian canal and the surrounding lamellae and lacunae are collectively called a Haversian system or osteon. The Haversian canals communicate with one another, with the periosteum and also with the marrow cavity by transverse or oblique canals called Volkmanns canals. Nutrients and gases diffuse from the vascular supply of Haversian canals.
AP Inter 1st Year Zoology Question Paper March 2016 1

Question 13.
Write the salient features of the anthozoans.
Answer:
Class – Anthozoa or Actinozoa :

  1. They are commonly referred to as sea anemones.
  2. They are sedentary and only polypoid in form.
  3. Coelenteron is divided into several compartments by vertical septa called mesenteries. Adamsia (sea anemone)
  4. Mesoglea contains connective tissue.
  5. Cnidocytes occur both in the ectoderm and endoderm.
  6. Germ cells are derived from the endoderm.
    Examples: Adamsia (sea anemone), Corailium rubrum (precious red stone coral), Gorgonia (sea fan), Pennatula (sea pen).

AP Inter 1st Year Zoology Question Paper March 2016 2

AP Inter 1st Year Zoology Question Paper March 2016

Question 14.
Compare and contrast cartilaginous and bony fishes.
Answer:

Cartilaginous fishes Bony fishes
1) Caudal fin is heterocercal 1) Caudal fin is diphycercal or homocercal.
2) Scales if present placoid scales. 2) Scales are ganoid, cycloid or ctenoid scales.
3) Endoskeleton is entirely cartilage. 3) Endoskeleton is bony.
4) Mouth and nostrils are ventral. 4) Mouth is usually terminal.
5) Digestive tract opens into cloaca, If present. 5) Digestive tract opens out by anus.
6) Air bladder is absent. 6) Air bladder Is often present.
7) Fertilization is internal. Mostly viviparous, e.g : Scoliodon. 7) Fertilization is external. Mostly oviparous, e.g: Catla catla

 

Question 15.
What are the adverse effects of tobacco ?
Answer:
Effect : Smoking increases the carbon monoxide (CO) level and reduces the oxygen level in the blood. Nicotine stimulates the adrenal gland to release adrenaline and nor-adrenajine into blood.

These hormones raise the blood pressure and increase the heart rate. Smoking is associated with bronchitis, emphysema, coronary heart disease, gastric ulcer and increases the incidence of cancers of throat, lungs, urinary bladder etc. Smoking also paves the way to hard drugs. Yet smoking is very pravalent in society, both among young and old. Tobacco chewing is associated with increased risk of cancer of the oral cavity.

Question 16.
Discuss the causes and effects of “Global Warming”. What measures need to be taken to control “Global Warming” ?
Answer:
Global warming is causing climate changes and is “ilso responsible for the melting of polar ice caps and other snow caps of mountains such as the Himalayas. Over many years, this will result in a rise in sea levels all over the world that can submerge many coastal areas. The total spectrum of changes that global warming can bring about is a subject that is still under active research.
Global warming – Control measures :

  1. The measures include cutting down use of fossil fuels
  2. Improving efficiency of energy usage .
  3. Planting of trees, and avoiding deforestation
  4. Slowing down the growth of human population.

Section – C (2 × 8 = 16)

Note : Answer any two questions in 60 lines each.

Question 17.
Describe the life cycle of plasmodium vivax in mosquito.
Answer:
Life cycle of Plasmodium in mosquito (The mosquito phase) Ross cycle : When a female Anopheles mosquito bites and sucks the blood of a malaria patient, the gametocytes along with the other stages of the erythrocytic cycle reach the crop of mosquito. Here all the stages are digested except the gametocytes. Further part of the life cycle consists of:
i) Gametogony
ii) Fertilization
iii) Formation of ookinete & oocysts
iv) Sporogony

i) Gametogony : The formation of male and female gametes from the gametocytes is called gametogony. It occurs in the lumen of the crop of mosquito.

Formation of male gametes : During this process, the nucleus of microgametocyte divides into eight daughter nuclei called pronuclei which reach the periphery. The cytoplasm is pushed out in the form of eight flagella like processes. Into each flagellum like process, one pronucleus enters and forms a micro gamete or male gamete. These male gametes show lashing movements like flagella and get separated from the cytoplasm of microgametocyte. This process is called exflagellation.

Formation of female gamete: The female gametocyte undergoes a few changes and transforms into a female gamete. This process is called maturation. The nucleus of the female gamete moves towards the periphery and the cytoplasm at that point forms a projection. This projected region is called the fertilization cone.

ii) Fertilization: The fusion of male and female gametes is called fertilization. It also occurs in the lumen of the crop of the mosquito. When an actively moving male gamete comes into contact with the fertilization cone of the female gamete, it enters it. The pronuclei and cytoplasm of these two gametes fuse with each other, resulting in the formation of a synkaryon. Since the two gametes are dissimilar in size, this process is known as anisogamy. The female gamete that bears the synkaryon is called the zygote which is round and non – motile.
AP Inter 1st Year Zoology Question Paper March 2016 3
iii) Formation of ookinete and oocysts : The zygote remains inactive for some time and then transforms into a long, slender, motile, vermiform ookinete or vermicule within 18 to 24 hours. It pierces the wall of the crop and settles beneath the basement membrane. It becomes round and secretes a cyst around its body. This encysted ookinete is now called oocyst. About 50 to 500 oocysts are formed on the wall of the crop and appear in the form of small nodules. (Sir Ronald Ross identified these oocysts for the first time).
AP Inter 1st Year Zoology Question Paper March 2016 4
iv) Sporogony : The formation of sporozoites in the oocysts is called sporogony. According to Bano, the nucleus of the oocyst first undergoes reduction division followed by repeated mitotic divisions resulting in the formation of about 1,000 daughter nuclei. Each bit of nucleus is surrounded by a little bit of the cytoplasm and transforms into a sickle shaped sporozoite. Oocyst with such sporozoites is called sporocyst. When this sporocyst ruptures, the sporozoites are liberated into the haemocoel of the mosquito. From there, they travel into the salivary glands and are ready for infection. The life cycle of Plasmodium in mosquito is completed in about 10 to 24 days.

AP Inter 1st Year Zoology Question Paper March 2016

Question 18.
Describe the digestive system of periplaneta in detail with a neat and labelled diagram.
Answer:
The digestive system of cockroach consists of an alimentary canal and the associated glands. The preoral cavity, surrounded by the mouth parts, is present in front of the mouth. The hypopharynx divides it into two chambers called cibarium (anterior) and salivarium (posterior).

Alimentary canal : The alimentary canal of cockroach is a long tube and is coiled at some places. lt extends between the mouth and the anus. It is divided into three regions, namely, foregut or stomodaeum, midgut or mesenteron and hindgut or proctodaeum. The foregut and hindgut are internally lined by ectoderm. The mesenteron is lined by the endodermal cells.

Foregut of stomodaeum : The foregut includes pharynx, oesophagus, crop, and gizzard. It is internally lined by a chitinous cuticle. Mouth opens into the pharynx, which in turn leads into a narrow tubular oesophagus. The oesophagus opens behind into a thin walled distensible sac called crop. The crop serves as a reservoir for storing food. Its outer surface is covered by a network of tracheae.
AP Inter 1st Year Zoology Question Paper March 2016 5
Behind the crop there is a thick walled muscular proventriculus, or gizzard. The chitinous inner lining of the gizzard has six powerful teeth, which form an efficient grinding apparatus. Behind each tooth is a hairy pad, which bears backwardly directed bristles. Among these plates, food is thoroughly ground into fine particles. These food particles are filtered by the bristles. The gizzard thus acts both as a grinding mill and also as a sieve. There Is a membranous projection of the gizzard into the mesenteron in the form of a funnel called stomodeai valve. This valve prevents the entry (regurgitation) of food from the mesenteron back into the gizzard.

Midgut (mesenteron or ventriculus) : The midgut is a short and narrow tube behind the gizzard. It is also called mesenteron or ventriculus. Between the ventriculus and the gizzard, arising from ventriculus, there are six to eight finger like diverticula called hepatic caecae. They are helpful in digestion and absorption of the digested food materials. Ventriculus is functionally divided into an anterior secretory part and a posterior absorptive part.

The secretory part of the ventriculus has many gland cells and it secretes several enzymes. The ‘bolus’ of food in the mesenteron is enveloped by a chitinous and porous membrane called peritrophic membrane, which is secreted by the funnel like stomodeal valve of the gizzard.

Digested food is absorbed into the blood through the peritrophic membrane in the posterior absorptive region of the ventriculus. The peritrophic membrane protects the wall of the ventriculus from hard food particles in the food. The opening of the ventriculus into the hindgut is controlled by a sphincter muscle. It prevents entry of undigested food and uric acid from the hindgut into the midgut.

Hindgut or proctodaeum : The hindgut is a long coiled tube, consisting of three regions namely ileum, colon and rectum, it is internally lined by chitinous cuticle. The ileum that lies behind the mesenteron is a short tube. Six bundles of fine yellow, blind tubules called malpighian tubules open into the ileum near the junction of mesenteron and ileum. Malpighian tubules are excretory in function. Ileum collects uric acid from the malpighian tubules and undigested food from the mesenteron. Ileum opens behind into a long coiled tube called colon. Colon leads into a short and wide rectum, which opens out through the anus. Rectum bears on its inner side six longitudinal chitinous folds called rectal papillae. They are concerned with the reabsorption of water from the undigested food.

Digestive glands : The digestive glands associated with the alimentary canal of cockroach are salivary glands, hepatic caecae and glandular cells of the mesenteron.

Salivary glands : There is a pair of salivary glands attached to the ventrolateral sides of the crop, one on each side. Each salivary gland has two lobes. Each lobe of salivary gland has many lobules called acini. Each acinus is a group of secretory cells called zymogen cells with a small ductule. The ductules of both the lobes of a salivary gland unite to form a common salivary duct on each side.
AP Inter 1st Year Zoology Question Paper March 2016 6
The two common salivary ducts are joined to form the median salivary duct. Between the two lobes of a salivary gland of each side is a sac called salivary receptacle that stores saliva. It leads into a receptacular duct, or ‘reservoir duct’. The receptacular ducts of both the sides are united to form a common receptacular duct, or ‘common reservoir duct’. The median salivary duct opens into the common receptacular duct. Later these two form an efferent salivary duct. The efferent salivary duct opens at the base of the hypopharynx. Acinar cells secrete saliva, which contains starch digesting enzymes such as amylase.

AP Inter 1st Year Zoology Question Paper March 2016

Question 19.
Describe the different types of food chains that exist in an ecosystem.
Answer:
Energy flows into biological systems (ecosystems) from the Sun. The biological systems of environment include several food levels called trophic levels. A trophic level is composed of those organisms which have the same source of energy and having the same number of steps away from the sun. Thus a plant’s trophic level is one, while that of a herbivore – two, and that of the first level carnivore – three. The second and third levels of the carnivores occupy fourth and fifth trophic levels respectively.
AP Inter 1st Year Zoology Question Paper March 2016 7
A given organism may occupy more than one trophic level simultaneously. One must remember that the trophic level represents a functional level. A given species may occupy more than one trophic level in the same ecosystem at the same time; for example, a sparrow is a primary consumer when it eats seeds, fruits, and a secondary consumer when it eats insects and worms.

The food energy passes from one trophic levels to another trophic level mostly from the lower to higher trophic levels. When the ‘path of food energy is ‘linear’, the components resemble the ‘links’ of a chain, and it is called ‘food chain’. Generally a food chain ends with decomposers. The three major types of food chains in an ecosystem are Grazing Food Chain, Parasitic Food Chain and Detritus Food Chain.

I. Grazing Food Chain (GFC): It is also known as predatory food chain. It begins with the green plants (producers) and the second, third and fourth trophic levels are occupied by the herbivores, primary carnivores and secondary carnivores respectively. In some food chains there is yet another trophic level – the climax carnivores. The number of trophic levels in food chains varies from 3 to 5 generally. Some examples for grazing food chain (GFC) are given below.
AP Inter 1st Year Zoology Question Paper March 2016 8
AP Inter 1st Year Zoology Question Paper March 2016 9
II. Parasitic food chain : Some authors included the ‘parasitic Food Chains’ as a part of the GFC. As in the case of GFCs, it also begins with the producers, the plants (directly or indirectly). However, the food energy passes from large organisms to small organisms in the parasitic chains. For instance, a tree which occupies the 1st trophic level provides shelter and food for many birds. These birds host many ectoparasites and endo-parasites. Thus, unlike in the predator food chain, the path of the flow of energy includes fewer, large sized organisms in the lower trophic levels, and numerous, small sized organisms in the successive higher trophic levels.

III. Detritus Food Chain : The detritus food chain (DFC) begins with dead organic matter (such as leaf litter, bodies of dead organisms). It is made up of Decomposers which are heterotrophic organisms, mainly the ‘fungi’ and ‘bacteria’. They meet their energy and nutrient requirements by degrading dead organic matter or detritus. These are also known as saprotrophs (sapro : to decompose)

Decomposers secrete digestive enzyme that breakdown dead and waste materials (such as faeces) into simple absorbable substances. Some examples of detritus food chains are :

      1. Detritus (formed from leaf litter) – Earthworms – Frogs – Snakes.
      2. Dead animals – Flies and maggots – Frogs – Snakes.

In an aquatic ecosystem, GFC is the major ‘conduit’ for the energy flow. As against this, in a terrestrial ecosystem, a much larger fraction of energy flows through the detritus food chain than through the GFC. Detritus food chain may be connected with the grazing food chain at some levels. Some of the organisms of DFC may form the prey of the GFC animals. For example, in the detritus food chain given above, the earthworms of the DFC may become the food of the birds of the GFC. It is to be understood that food chains are not ‘isolated’ always.

AP Inter 1st Year Hindi Model Paper Set 9 with Solutions

Exploring a variety of AP Inter 1st Year Hindi Model Papers Set 9 is key to a well-rounded exam preparation strategy.

AP Inter 1st Year Hindi Model Paper Set 9 with Solutions

Time : 3 Hours
Max Marks : 100

सूचनाएँ :

  1. सभी प्रश्न अनिवार्य हैं ।
  2. जिस क्रम में प्रश्न दिये गये हैं, उसी क्रम से उत्तर लिखना अनिवार्य है ।

खण्ड – ‘क’
(60 अंक)

1. दुःख में सुमिरन सब करे, सुख में करै न कोय ।
जो सुख में सुमिरन करे, तो दुख काहे होय ॥
उत्तर:
प्रसंग :- यह दोहा कबीरदास के द्वारा लिखी गयी ‘साखी’ नामक रचना से लिया गया है | वे निर्गुणशाखा के अन्तर्गत ज्ञानमार्ग शाखा से संबंधित सन्त कवि थे ।
सन्दर्भ :- हमेशा ईश्वर का स्मरण करने का उपदेश कवि दे रहे हैं ।
व्याख्या :- कवि का कहना है कि सभी लोग दुःख मे ईश्वर का स्मरण करते है, पर सुख में ईश्वर को भूल जाते हैं । लेकिन यदि सुख मे भी ईश्वर का स्मरण करे तो ऐसे व्यक्ति को कभी भी दुख की प्राप्ति नही होती ।

विशेषताएँ :-

  1. इसमें हमेशा ईश्वर के प्रति विश्वास रखने का सन्देश दे रहे हैं ।
  2. उनकी भाषा सदुक्कडी है ।

(अथवा)

बिगरी बात बन नहीं, लाख करो किन कोय |
रहिमन फाटे दूध को, मभे न माखन होय ॥
उत्तर:
प्रसंग :- यह दोहा रहीम के द्वारा लिखी गयी दोहावली से लिया गया है । वे भक्तिकाल से सम्बन्धित कृष्ण भक्त कवि थे ।
सन्दर्भ :- सब के साथ अच्छी तरह व्यवहार करने का सन्देश दे रहे है |
व्याख्या :- रहीम का कहना है कि एक बार किसी से बात या व्यवहार बिगड जाता है तो अनेक प्रयत्न करने पर भी उनके बीच समस्या ठीक नही होता । जिस प्रकार एक फटे दूध को मधने से मक्कन नहीं मिलाता है उसी प्रकार बात बिगड जाने से आपस में सम्बन्धं टूट जाता है और फिर नहीं बनता ।

विशेषताएँ :-

  1. इससे कवि मित्रता की भावना पर जोर देते हैं ।
  2. उनकी भाषा, व्रजभाषा है ।

2. किसी एक कविता का सारांश लिखिए ।

(1) फल की चाह
उत्तर:
कवि परिचय :- माखनलाल चतुर्वेदी का जन्म सन् 1889 मे बाबई, होशंगाबाद मध्यप्रदेश मे हुआ | वे एक भारतीय आत्मा के रूप मे प्रसिद्ध थे । उनकी रचनाओं मे राष्ट्रीयता और देश भक्ति स्पष्ट झलकती है । उनहें कवि और राष्ट्र सेवी के रूप मे सम्मान प्राप्त हुआ । हिमकिरीटिनी, हिमतरंगिणी, माता, युगचरण, मरण ज्वर आदि उनकी प्रसिद्ध रचनाएँ है । उनकी मृत्यु सन् 1968 मे हुई । प्रस्तुत ‘फूल की चाह’ कविता मे कवि फूलों के द्वारा अपनी देश भक्ति और जीवन के लिए कुछ लक्ष्य रखने का सन्देश दे रहे हैं ।

सारांश :- कवि फूलो के द्वारा अपनी इच्छा व्यक्त कर रहे है कि मुझे देवताओं के गले मे गहनों के बीच गूंध जाने की इच्छा नहीं है । प्रियतम के हाथो मे माला बनकर प्रेयसी को ललकारने की इच्छा नही है । बडे-बडे लोगों के शवों पर माला बनकर पडने की इच्छा भी नही है । ईश्वर के सिर पर चढकर अपने भाग्य पर घमंड करने की भी इच्छा नही है पर हे वनमाली ! मुझे अवश्य तोड लो | देश के लिए बलिदान देने वाले वीर जिस मार्ग मे चलते है उनके जाने के रास्ते पर मुझे फेंक दो । क्योंकि मै उनके चरणो के स्पर्श से पवित्र हो जाऊँगी और उनके चरणों को आराम पहुँचा दूँगी ।

इस प्रकार कवि फूलो के द्वारा अपनी देशभक्ति भावना, देश के लिए मर मिटने वाले वीरो के प्रति गौरव और जीवन के लिए कुछ-न-कुछ लक्ष्य रखने का सन्देश दे रहे हैं । उनकी भाषा सरल खडी बोली है।

(2) अकाल और उसके बाद
उत्तर:
कवि ने इसके अकाल के समय और अकान के बाद की स्थिती को दो पद्यों के द्वारा विस्तार रूप मे वर्णन किया | अकाल के समय मे घर पर खाने के लिए अनाज का अभाव है इसलिए कई दिनो से जला और अनाज के अभाव से आठा न पीसने के कारण चक्की के भी काम नही किया | चूहे चूल्हा न जलने से घर का एक का अंख वाला कुत्ता उसी के पास सो रही है । खाना न मिलने से सारा घर और आदी जन्तुएँ भी उदास है । घर के लोग उदास से एक बैठे हुए है । और दीवार पर छिपकलियाँ गस्ती देखे हैं । कई दिनों से अकाल से पीडित होते वाले घरों में अनाज न मिलने से चूहो की स्थिति भी बडी दयनीय थी ।

अब अकाल चल गया । बहुत दिनों के बाद घर मे अनाज आया । चूल्हे जलने से घर के आंगन मे धुँआ उठा । घर के सभी लोगो की आँखों मे चमक उठी अर्थात सब लोगों मे असाह भर गया । भोजन के बाद केंके हुए अन्न से अपने पेट भरने की आशा से काँठा भी पंख खुजलाकर इत्तर इन्तजार कर रही हैं ।

इस्प्रकार कवि ने अकाल से पीडित जनता की दयनी स्थिति और बाद की स्थिति का मार्मिक रूप से चित्रण किया है । अकल की दुस्थिति केवल लोंगों पर ही नहीं बल्कि उनके चारों ओर वातारिण को किस प्रकार प्रभावित करती है, उस्का स्पश्ट चित्रण किया है । उनकी भाषा सरल खडी बोली है।

AP Inter 1st Year Hindi Model Paper Set 9 with Solutions

3. किसी एक पाठ का सारांश लिखिए ।

(1) पर्यावरण और जीवन
उत्तर:
मानव एक सामाजिक प्राणी है । व्यक्ति और परिवार से समाज का निर्माण होता है । वह अपनी आवश्यकताओं की पूर्ति के लिए एक दूसरे भर निर्भर रहता है । मानव समाज के विकास में विज्ञान, वैज्ञानिक आविष्कारों, परिवाहन तथा संचार के साधनों और विविध यंत्रों आदि की महत्वपूर्ण भूमिका रही । इस प्रकार जहाँ मानव सभ्यता के विकास के साथ ही प्रकृति का शोषण आरम्भ हुआ, वहीं जनसंख्या की वृद्धि के साथ पृथ्वी पर प्रदूषण भी बढने लगा । इसके लिए प्रकृति की रक्षा और सुरक्षा को मानव जीवन के लिए अनिवार्य अंग बनाना होगा ।

पर्यावरण हमारे चारों ओर के वातावरण से सम्बंध है । पर्यावरण समस्त जीव-जन्तुओं, प्राणियों और मनुष्य के जीवन का आधार और अधिरचना अर्थात् पर्यावरण सभी के जीवन का अभिन्न अंग है, यही कारण कि आदिकाल से प्रकृति के सान्निध्य मे मानव ने अपने जीवन को विकसित किया है । जब जैविक तथा अजैविक पदार्थों के बीच संतुलन को पर्यावरण संतुलन कहा जाता है । आजकल पर्यावरण संतुलन बिगड़ रहा है ।

मनुष्य अपनी सुख-सुविधा के लिए प्राकृतिक सम्पदाओं को क्षति पहुँचा रहा है । पर्यावरण संकट प्रदूषण और असंतुलन का परिणाम है । यह संकट भूमि, जल वायु, ताप, ऊर्जा, खनिज, ध्वनि तथा वनस्पति आदि सभी क्षेत्रों मे उत्पन्न हो चुका है । यहा प्रदूषण विभिन्न रूपों मे देखा जा सकता है। जैसे भूमिप्रदूषण, वायुप्रदूषण, जल प्रदूषण, ध्वनि प्रदूषण आदि । इन प्रदूषणों ने न केवल मानव जीवन को संकटमय बना दिया है अपितु उसके लिए कई प्रकार की बीमारियों को भी उत्पन्न कर दिया । जैसे कम उम्र के बच्चों और वृद्धों के लिए स्वाँस की समस्या, ब्रोंकाइटिस, फेफडो की टी.बी. कैंसर, त्वचा का रोग आदि समस्याएँ बढती जा रही है । प्रदूषित पानी पीने से उदर संबंधी रोग बढ रहे हैं । ध्वनि प्रदूषण से महानगरों मे बहरेपन की समस्या बढ रही है | मोबाइल फोन से निकलने वाले रेडिएशन से मनुष्य मे चिड़चिड़ापन, बेचैनी, हाई ब्लडप्रेशर एवं डिप्रेशन आदि मानसिक बीमारियाँ होने लगी है इसके इलावा कारखानों, अस्पतालों, प्लास्टिक से निकलने वाली जहरीली राख और केमीकल्स मिट्टी की उवराशक्ति कम हो रही है और कीटनाशक केमिकल्स स्वास्थ्य को प्रभावित कर रहे है |

प्रदूषण निवारण के लिए उपाय : सबसे पहले आबादी का नियन्त्रण करना चाहिए । जंगलों को न काटना, औद्योगीकरण के समय में नियमों का पालन, जल की रक्षा, स्वच्छ भारत कार्यक्रम, पेड़ों को रोपना, कारखानों के विष पदार्थ को पानी में फेकने पर पाबंद रखना, प्रदूषण फैलाने वालों को दण्ड देना इन सभी के पालन करने से पर्यावरण की रक्षा और प्रदूषण से मुक्त हो जाएंगे ।

(2) अभातो धुमक्कड जिज्ञासा
उत्तर:
शास्त्रों मे व्यक्ति और समाज के लिए जिज्ञासा को हितकारी माना गया है । लेकिन लेखक के अनुसार घुमक्कडी से अधिक सर्वश्रेष्ठ वस्तु समाज के लिए और कोई नही है । प्राकृतिक आदिम मनुष्य परम घुमक्कड था जिस ने आज की दुनिया को बनाया है । आदिम घुमक्कडों में से आयों, शको और हूणों ने अपने शूनी पथों द्वारा मानवता के पथ को प्रशस्त किया । मंगोल घुमक्कडों के द्वारा वैज्ञानिक युग का आरम्भ हुआ ।

कोलम्बस और वास्को-डि-गामा अपनी घुमक्कडी प्रकृति से अमेरिका पर झंडी गाडकर पश्चिमी देशो को आगे बडाया । सदियों पहले चीन और भारत घुमक्कड धर्म से विमुख रहने से ही आस्ट्रेलिया की अपार संपत्ति और अपार भूमि से वंचित रहे ।

दुनिया के अधिकांश धर्मनायक घुमक्कड थे । धर्माचार्यों में आचार-विचार, बुद्धि और तर्क तथा सहृदयता से सर्वश्रेष्ट बुद्ध घुमक्कड राजा थे । बुद्ध ने सिर्फ पुरुषों के लिए घुमक्कडी करने का आदेश नहीं दिया, बल्कि स्त्रियों के लिए भी यही उपदेश उन्होंने दिया ।

जैन धर्म भी प्राचीन धर्म है जिसके श्रमण महावीर भी प्रथम श्रेणी के घुमक्कड थे। वे आजीवन । घूमते ही रहे । शंकराचार्य जो साक्षात् ब्रह्मस्वरूप थे, जिन्हे बडा बनानेवाला धर्म यही घुमक्कडी धर्म था । अपने थोडे से जीवन मे उन्होंने तीन भाष्य भी लिखे और अपने आचरण से अनुनायियों को घुमक्कडी पाठ भी पढाया । रामानुज, माध्वाचार्य जैसे धार्मिक अनुयायी अपनी धार्मिक पाखण्डता से दूसरी श्रेणी के घुमक्कड बन गए । इसलिए शैव हो या वैष्णव, वेदान्ती हो या सदान्ती, सभी को केवल घुमक्कडी धर्म ने ही आगे बढाया ।

गुरूनानक, स्वामी दयानन्द, अपनी घुमक्कडी धर्म से ही महान बन गए । बीसवी शताब्दी में भारत देश में अनेक धार्मिक सम्प्रदायों का आना-जाना हो गया । जैसे यहूदी, मारवाडी जैसे लोग अपनी हस घुमक्कडी धर्म से केवल व्यापार कुशल, उद्योग-निष्णात ही नही बल्कि विज्ञान, दर्शन, साहित्य, संगीत सभी क्षेत्रों को आगे बढाया ।

इस प्रकार घुमक्कड होना आदमी के लिए परम सौभाग्य की बात है । घुमक्कडी के लिए चिन्ताहीन होना और चिन्ताहीन के लिए घुमक्कडी होना आवश्यक है । जाति का भविष्य घुमक्कडी पर निर्भर करता है | घुमक्कडी की गति को रोकनेवाला इस दुनिया में कोई नहीं है । सभी को घुमक्कड की दीक्षा लेनी चाहिए ।

इस प्रकार निबन्धकार इसमें कहते है कि दुनिया मे मनुष्य जन्म एक ही बार होता है और जवानी भी केवल एक ही बार आती है। इसलिए साहसी स्त्री और पुरुष दोनों को घुमक्कड धर्म को स्वीकारना चाहिए । उनकी भाषा शुद्ध खडी बोली है।

4. किसी एक कहानी का सारांश लिखिए ।

(1) चीफ का दावात
उत्तर:
मिस्टर शामनाथ अपने घर में ‘चीफ’ को दावत पर बुलाते है । अपनी धर्म पत्नी के साथ मिलकर सारी तैयारियाँ करने लगता है । चीफ अमेरिकन है । उसको खुश करने से शामनाथ को नौकरी में तरक्की होगी । इसलिए दावत की शाम घर की सभी चीजों को चीफ के अनुरूफ बनाने की चेष्टा की जाती है । ड्रिंक का इंतजाम बैठक में कर दिया गया । घर का फालतू सामान अलमारियों के पीछे और पलंगों के नीचे छिपाया जाने लगा । तब उसको अपनी बूढ़ी माँ की याद आती है । अब प्रश्न उठता है कि उसको कहाँ छिपाना है । उस बुढ़िया लोगों के सामने आना- जाना, नींद में उठे खर्राटे की आवाज ये सब किसी के सामने प्रस्तुत होना शामनाथ के लिए पसंद नही है । इसलिए माँ को अनेक चेतन चेतावनियाँ देते है कि ठीक से कुर्सी पर बैठकर रहना है, पार्टी खतम होने तक नही सोना है । माँ सब के लिए राजी होकर उसके कहे अनुसार अच्छी साडी पहनकर अपने घर में बैठ जाती है ।

चीफ अपनी पत्नी के साथ दावत पर आते है । सभी अतिथि लोग आते है । दावत शुरु हो जाती है। ड्रिंक होने के बाद बरामदे में आते तो वहाँ उसकी माँ कुर्सी पर पैर रखकर सो रही है । यह दृश्या देखकर शामनाथ कृद्ध हो उठे । पर चीफ और उसकी पत्नी उस पर बडी सहानुभूति देखने है । चीफ माँ को वहाँ से जाने नही देते तथा पंजाबी लोक गीत सुनाने का आग्रह करते है | माँ शामनाथ की सोच के विरुद्ध अच्छी तरह गाकर चीफ एवं मेहमानों को प्रश्न करती है । माँ के हाथों में बनी ‘फुलकारी’ चीफ को बहुत पसंद आता हैं । उसके हाथों से बनी फुलकारी के लिए वापस आने का वादा करता है । इस प्रकार उसकी बूढ़ी माँ अचानक ही मूल्यवान सिद्ध हो उठती है । जो बेकार थी वही तरक्की का जरिया बन जाती है। उससे फुलकारी बनाने की ताकत नही होने पर भी अपने बेटे की तरक्की के लिए खुद बनाना चाहती है ।

इस प्रकार आजकल माँ-बेटे के संबंध में प्रेम आदर भावनाएँ लुप्त हो रही है। शामनाथ अपनी नौकरी के लिए माँ को आतंक मानता है । लेकिन उसी के द्वारा उसकी इच्छा पूर्ण हो जाती है । माँ ́के पाश्चात्य सभ्यता का प्रभाव, माँ की ममता का सजीव चित्रण उसमें किया गया है ।

(2) बदला
उत्तर:
धम्मपट्टनं एक ऐसा जगह है, वहाँ पर एक अस्पताल है जहाँ कदम-कदम पर भ्रष्टाचार, मामूल वसूल किया जाता है । वहाँ पर कुछ दूरी पर एक गाँव था वहाँ पर कोटय्या एक मामूली किसान, मजदूर था । वह खास पढा लिखा नहीं होने पर भी अखबार पढ लेता है । उसकी पत्नी गर्भवती थी। नौ महीने मे तबियत खराब होने से अपने मालिक की बैलगाडी लेकर धर्मपट्टनं अस्पताल पहुँचता है अस्पताल के अन्दर जाने के लिए भी चवन्नी हाथ मे रखना पडता है । अन्दर जाने के बाद उसकी पत्नी बेहोश होने पर भी कोई भी इसके प्रति ध्यान नहीं देते । शायद वहाँ बेहोशी होना भी मामूली बात है। कुछ देर बाद डाक्टर पद्मा वहाँ आती है । कोटय्या भय, विवशता, अज्ञान अन्धार मे पडकर वही बाहर खड़ा हो जाता है ।

बाद में उस पता चलना है कि प्रसव के पहले ही उसकी पत्नी चली गयी। उसकी लाश को लेकर जाने के लिए भी उसे मामूल देना पडता है | पत्नी का दहन संस्कार होने के बाद कोटय्या धम्मपट्टनं अस्पताल के सामने सात्याग्रह करना शुरू करता है । पर किसी ने इस पर ध्यान नहीं देता । पाँच दिन बाद डाक्टर पद्मा उसको समझाने आता है। धीरे-धीरे सब का ध्यान इसकी ओर आकृष्ट होता है । डाक्टर पद्मा पर शिकायत रखी जाती है और इस के द्वारा सभी कर्मचारियों के विषय बाहर आने लगते है । सत्याग्रह करते-करते कोटय्या का स्वास्थ्य बिगड़ जाता है और उसी अस्पताल मे दाखिल हो जाता है । इन समस्याओं के बीच डा. पद्मा की हत्या अस्पताल मे हो जाती हैं इसकी जिम्मेदारी कोटय्या पर पडती है और पुलीस उसको जेल लेजाता है । वह निर्दोष होकर भी अन्त मे उसे दण्ड भोगना पड़ता है ।

इस प्रकार इस कहानी का नायक एक गरीब मजदूर मामूली व्यक्ति था । आत्याचारों के विरूद्ध स्वर उठाने पर भी उसका कोई नहीं सुनता । निर्दोष होकर भी सजा भुगतनी पड़ती है । तत्कालीन सामाजिक स्थिति को मार्मिक रूप से कहानीकार ने चित्रण किया । उनकी भाषा सरल है |

5. निम्नलिखित दो पद्यांशों की संदर्भ सहित व्याख्या कीजिए :

(1) कई दिनों तक चूल्हा रोपा, चक्की रही उदास ।
कई दिनों तक कानी कुतिया सोई उनके पास ॥
कई दिनों तक लगी भीत पर छिपकलियों की गरत ।
कई दिनों तक चूहों की भी हालत रही शिकस्त ॥
उत्तर:
प्रसंग :- यह पद्य को नागार्जुन द्वारा लिखे गयी अकाल और उसके बाद कविता से लिया गया है । वे प्रगतिवादी कवि थे और आधुनिक कबीर से प्रसिद्ध थे !
सन्दर्भ :- इस्में कवि अकाल के समय देश की स्थिति और लोगों की दयनीय स्थिति की ओर सकेत कर रहे है ।
व्याख्या :- अकाल के समय अनाज न मिलने के कारण घर में चूलहा नही जल रहे हँ । आटा न पिसने के कारण चक्की भी बन्द भी । चूलहे न जलने से उसे घर का कुत्ता सो रहा है । घर के लोगों की उदासी से दीवार के छिपकलियां मस्तसे गस्ती दे रहे है और अनाज न मिलने से घर के चूहों की स्थिति भी दयनीय हो गयी ।

विशेषताएँ :-

  1. कवि की प्रगतिवादी धारणा का चित्रण हो रहा है ।
  2. उनकी भाषा सरल खडीबोली है ।

(2) जग पीडित अति से – दुख से
जग पीडित रे अति – सुख
मानव – जग में बँद जाएँ
दुख सुख से औ सुख दुख से
उत्तर:
प्रसंग :- यह पद्य सुमित्रानंदन पंत के द्वारा लिखी गयी ‘सुख-दुख’ नामक कविता से लिया गया है । वे प्रकृति का सुकुमार कवि कहे जाते है ।
सन्दर्भ :- इसमें कवि सुख-दुख को समान रूप में स्वीकार करने की बात कह रहे हैं ।
व्याख्या : – कवि का कहना है कि यह संसार हमेशा अति दुख से और अति सुख से पीडित होता रहता हैं | लेकिन मानव जीवन मे सुख और दुख समान रूप मे बाँट जाना चाहिए ।

विशेषताएँ :-

  1. जीवन के लिए सुख और दुख होने का आवश्यकता के बारे में कवि कह रहे हैं ।
  2. उनकी भाषा खडीबोली हैं ।

(3) मुझे तोड लेना वनमाली ।
उस पथ पर देना तुम फेंक |
मातृ-भूमि पर शीश चढाने,
जिस पभ जावें वीर अनेक ।
उत्तर:
प्रसंग :- यह पद्य ‘फूल की चाह’ नामक कविता से लिया गया है । यह कविता माखनलाल चतुर्वेदी के द्वारा लिखी गई है । वे भारतीय आत्मा के रूप से प्रसिद्ध है ।
सन्दर्भ :- कवि फूल की चाह के द्वारा अपनी देशभक्ति भावना को व्यक्त कर रहे हैं ।
व्याख्या :- कवि फूलों के लो । पर तोड़कर उस के लिए वीर जाते है। को मै राहत दूँगी ।
द्वारा अपना विचार व्यक्त कर रहे है कि हे वनवाली ! मुझे अवश्य तोड़ रास्ते मे मुझे फेंक दो जिस रास्ते पर मातृभूमि के लिए बलिदान करने ताकि उनके चरणों के नीचे पड़कर मैं पवित्र हो जाऊँगी और उनके पैरों

विशेषताएँ :-

  1. देश के लिए बलिदान करने वाले वीरो के प्रति कवि का गौरव स्पष्ट हो रहा है ।
  2. उनकी भाषा सरल खडीबोली है ।

(4) वह आता –
दो टूक कलेजे के करता पछताता पथ पर आता ।
पेट – पीठ दोनों मिलकर है एक
चल रहा लकुटिया टेक
मुट्ठी भर दाने को भूख मिटाने को
मुँह फटी – मुरानी झोली का फैलाता –
दो टूक कलेजे के करता पछताता पथ पर आता ।
उत्तर:
प्रसंग :- यह पद्य निराला जी के द्वारा लिखी गयी ‘भिक्षुक’ नामक कविता से लिया गया है वे छायावादी कवि है ।
सन्दर्भ :- इसके एक भिक्षुक की दयनीय स्थिति का वर्णन किया गया है ।
व्याख्या : – कवि एक भिक्षुक के जीवन का वर्णन कर रहा है कि एक भिक्षुक अपनी दयनीय स्थिति पर टूटे हृदय से उस पथ पर आ रहा है। भूख के कारण उसका पेट और पीट दोनों मिले हुए है । अपनी भूख मिटाने एक मुट्ठी भर अन्न के लिए लकडी टेकता हुआ आ रहा है । वह लकडी के सहारे खडे होकर अपने फटे हुए झोले का मुँह फैलाता है । अपनी दयनीय स्थिति से वह टूटे हृदय से मन ही मन रो रहा हैं ।

विशेषताएँ :-

  1. शोषित वर्ग के प्रति कवि की सहानुभूति व्यक्त होती है ।
  2. इसके कवि की प्रगतिवादी धारणा स्पष्ट होती है ।
  3. उनकी भाषा शुद्ध खडीबोली है ।

AP Inter 1st Year Hindi Model Paper Set 9 with Solutions

6. निम्नलिखित किन्हीं दो गद्यांशों की संदर्भ सहित व्याख्या कीजिए :

(1) “भारतीय संस्कृति का सौन्दर्यबोध नारी रूप से अविच्छिन्न सम्बन्ध से जुडा हुआ है |
उत्तर:
प्रसंग :- यह उद्धरण महादेवी वर्मा के द्वारा लिखी गयी “भारतीय संस्कृति और नारी” नामक निबन्ध से लिया गया वे छायावाद से सम्बन्धित प्रमुख साहित्यकार है ।
सन्दर्भ :- इसमें भारतीय संस्कृति और नारी के बीच के अविच्छिन्न सम्बन्ध के प्रति जोर दिया गया है ।
व्याख्या :- भारतीय संस्कृति मे प्राचीनकाल से ही नारी का महत्वपूर्ण स्थान है । भारतीय संस्कृति का सौन्दर्य बोध नारी के रूप से अविच्छिन्न सम्बन्ध से जुडा हुआ है। नारी को देवी, माता जैसे रूपों में दर्शाया गया है जो प्राचीन आर्य काल से पहले ही मातृसत्ता से जुडा हुआ है ।

विशेषताएँ :-

  1. भारतीय संस्कृति मे नारी की महानता के बारे में स्पष्ट किया गया है ।
  2. उनकी भाषा खडीबोली है ।

(2) हिन्दू होते हुए भी शिवा के लिए इस्लाब धर्म पुज्य है, इस्लाम के पवित्र स्थान, उसके पवित्र ग्रन्थ सम्मान की वस्तुएँ है |
उत्तर:
प्रसंग :- यह उद्धरण शिवाजी का सच्चा स्वरूप नामक एकांकी से लिया गया है । इसके लेखक सेठ गोविन्ददास जी है । आप गाँधी जी से प्रभावित होकर स्वतंत्रता सग्राम मे भी भाग लेकर जेल भी गये ।

सन्दर्भ :- हिन्दू होते हुए भी शिवाजी के मन में हिन्दू और मुसलमान धर्मों के प्रति गौरव स्पष्ट होती है ।

व्याख्या :- हिन्दू होते हुए भी शिवाजी हिन्दू और मुसलमान प्रजा में कोई भेद नहीं समझता । उसकी दृष्टि मे सारी जनता बराबर है । वह स्वयं हिन्दू होकर भी इस्लाम धर्म की पूजा करता है । उनके पवित्र स्थान और पवित्र धर्म के प्रति भी आदर व्यक्त करता है ।

विशेषताएँ :-

  1. इसमें शिवाजी की धार्मिक सहिष्णुता स्पष्ट होती है ।
  2. उनकी भाषा सरल है ।

(3) शंकर को शंकर किसी ब्रझा ने नहीं बनाया, उन्हे बडा बनानेवाला था यही घूमक्कडी – धर्म ।
उत्तर:
प्रसंग :- यह उद्धरण राहुल सांस्कृत्यायन के द्वारा लिखी गयी अथात घुमक्कड जिज्ञासा नामक यात्रा वृत्तांत है । वे पुरातत्व इतिहास के विशेष ज्ञाता रहे हैं और उनका यात्रा साहित्य आयन्त महत्वपूर्ण रहा है ।
सन्दर्भ :- लेखक इसमें घुमक्कडी प्रवृत्ति को सर्वश्रेष्ठ माना है और उसकी महानता को इसमें स्पष्ट करते हैं ।
व्याख्या :- अनेक धर्मावलम्बी बुद्ध, महावीर के साथ-साथ शंकराचार्य जी ने भी घुमक्कडी प्रवृत्ति का अनुकरण करने से ही महान बन गए । शंकराचार्य इतने महान थे कि उनको महान बनाने का श्रेय ब्रह्मा को नहीं घुमक्कडी धर्म को दिया जाता है। इसी प्रवृत्ति से उन्होंने अपने छोटे उम्र में ही अपने अनुयायियों को भी घुमक्कडी का पाठ पढाया ।

विशेषताएँ :-

  1. घुमक्कड धर्म को अपनाने का सन्देश लेखक देते हैं ।
  2. उनकी भाषा सरल खडीबोली है ।

(4) आंध्र प्रदेश धार्मिक रुप से संपन्न राज्य है ।
उत्तर:
प्रसंग :- यह वाक्य ‘आन्ध्र संस्कृति नामक पाठ से लिया गया है इसमें आन्ध्र संस्कृति पर विभिन्न रूपों में दृष्टि डाला गया है ।
सन्दर्भ :- आन्ध्र प्रान्त विभिन्न धर्मों का मिश्रित रूप है । इन सभी धर्मों से विकसित आन्ध्र राष्ट्र का चित्रण किया गया है ।
व्याख्या :- संस्कृति का मूल रूप धर्म में है । आन्ध्र प्रदेश धार्मिक रूप से एक संपन्न राज्य है यहाँ पर वैदिक, बौद्ध, जैन, अद्वैत, विशिस्टाद्वैत इस्लाम, ईसाई धर्म आदि धर्मों का विकास हुआ है। यहाँ पर अनेक जातियों और अलग-अलग संस्कृतियों का सहयोग दिखाई पडता है ।

विशेषताएँ :-

  1. आन्ध्र संस्कृति के विकास के प्रति ध्यान दिया गया है ।
  2. उनकी भाषा खडीबोली है ।

7. एक शब्द में उत्तर लिखिए (पद्यभाग ) ।

(1) कौन अपना पानी खुद नही पीती ?
उत्तर:
सरोवर

(2) माखनलाल चतुर्वेदी जी किसके जरिए अपनी देश भक्ति प्रकट कर रहे है ?
उत्तर:
फूल के जरिए

(3) ‘म्यान’ शब्द का अर्थ बताइए ?
उत्तर:
तलवार रखने का खाचा

(4) अकाल के समय घर पर खाने के लिए क्या नहीं आता ?
उत्तर:
अनाज

(5) भिक्षुक और उसके बालक किसके लिए भीख माँग रहे है ?
उत्तर:
भूख मिटाने के लिए

8. एक शब्द में उत्तर दीजिए (गद्यभाग) ।

(1) संस्कृति और सभ्यता शब्दों में अर्थ भेद बताइए ?
उत्तर:
संस्कृति अंतरिक आचरण से सम्बन्ध रखती है तो सभ्यता बाह्य आचरण से सम्बन्ध रखती है ।

(2) शिवाजी धन, धर्म तुलना में किसको अधिक महत्व देते है ?
उत्तर:
शील चरित्र को

(3) दुनिया के अधिकांश धर्मनायक कौन थे ?
उत्तर:
घुमक्कड थे ।

(4) पर्यावरण किन दो शब्दों से बना है ?
उत्तर:
परि, आवरण

(5) आन्ध्र प्रान्त के प्रसिद्ध वाग्गोयकार कौन थे ?
उत्तर:
अन्नमाचार्य, रामदास, त्यागराज

खण्ड – ‘ख’

9. निम्नलिखित में से कोई एक पत्र लिखिए :

नौकरी के लिए आवेदन पत्र लिखिए |
उत्तर:

नरसराव पेट,
दिनांक 17.07.2018.

प्रेषक :
वी. सहदेवी,
मकान नं. बी 185
एन.जी.वो. कॉलोनी,
नरसराव पेट – 522601.

सेवा में,
प्रधानाचार्य जी,
एस.के.बीर. आर. कॉलेज,
माचल

विषय : हिन्दी प्राध्यापक के लिए आवेदन पत्र |

संदर्भ : दैनिक ईनाडु में 20.6.2018 को प्रकाशित विज्ञापन |

दैनिक ईनाडु में प्रकाशित आपके विज्ञापन के द्वारा मुझे पता चला कि आपके कॉलेज में हिन्दी प्राध्यापक की नौकरी खाली है। इसके उत्तर में मैं आपना आवेदन पत्र आपकी सेवा में विचारार्थ भेज रही हूँ । आपसे प्रार्थना है कि मेरा आवेदन स्वीकार करें मेरे संबंध में विवरण साथ संलग्न
हैं ।

भवदीय,
वी. सहदेवी

संलग्न :

  1. दसवी कक्षा प्रमाण पत्र ।
  2. इंटरमीडियट प्रमाण पत्र |
  3. बी. ए. प्रमाण पत्र |
  4. एम. ए. प्रमाण पत्र ।
  5. चिकित्सा प्रमाण पत्र |
  6. अनुभव प्रमाण पत्र |
  7. यू.जी.सी. नेट प्रमाण पत्र |

अनुभव- मैं स्थानीय प्रभुत्व जूनियर कलाशाला में तीन वर्ष से हिन्दी प्राध्यापिका का काम कर रही हूँ | अंग्रेजी माध्यम से पढने वाले छात्रों के आवश्यकतानुसार मैं अंग्रेजी में भी समझा सकती हूँ ।

धन्यवाद !

हस्ताक्षर
वी. सहदेवी

(अथवा)

पुलिस स्टेशन के नाम शिकायती पत्र लिखिए ।

पिडुगुराला,
दिनांक 25.11.2018.

प्रेषक :
बी. सुरेश,
मकान नं. – 177
कस्तूरी नगर,
पिडुगुराल्ला – 522413.

सेवा में,
पुलिस इंसपेक्टर,
पुलिस थाना,
पिडुगुराल्ला – 522413,
महोदय,

निवेदन है कि कल रात हमारे घर में चोरी हुई है । आज सुबह हम गुंटुर से लौटे तो देखा कि घर का दरवाजा खुला पड़ा है हम बहुत भयभीत हुए घर के अंदर जाकर देखे तो पूरा सामान बिखरे पड़े हुए हैं । अलमारी का भी दरवाजा खुला पड़ा है । अलमारी में रखे पाँच हजार रूपये, सोने का हार तथा कुछ प्रमाण पत्र की चोरी की गयी है । इसलिए आपसे प्रार्थना है कि आप शीघ्रातिशीघ्र चोरों का पता लगाएँ और हमारी चीजों को दिलवाने की कृपा करें ।

आपका,
बी. सुरेश

10. किन्हीं पाँच (5) शब्दों के विलोम शब्द लिखिए ।

(1) अपराध × निरपराध
(2) एक × अनेक
(3) क्रय × विक्रय
(4) कर्म × अकर्म
(5) इधर × उधर
(6) ऊँचा × नीचा
(7) क्रोध × हँसी
(8) कुपुत्र × सुपुत्र
(9) घर × बाहर
(10) पक्ष × विपक्ष

(अथवा)

पुलिस स्टेशन के नाम शिकायती पत्र लिखिए ।
उत्तर:

पिडुगुराला,
दिनांक 25.11.2018.

प्रेषक :
बी. सुरेश,
मकान नं. – 177,
कस्तूरी नगर,
पिडुगुराल्ला – 522413.

सेवा में,
पुलिस इंसपेक्टर,
पुलिस थाना,
पिडुगुराल्ला – 522413,
महोदय,

निवेदन है कि कल रात हमारे घर में चोरी हुई है । आज सुबह हम गुंटुर से लौटे तो देखा कि घर का दरवाजा खुला पड़ा है हम बहुत भयभीत हुए घर के अंदर जाकर देखे तो पूरा सामान बिखरे पड़े हुए हैं । अलमारी का भी दरवाजा खुला पड़ा है। अलमारी में रखे पाँच हजार रूपये, सोने का हार तथा कुछ प्रमाण पत्र की चोरी की गयी है । इसलिए आपसे प्रार्थना है कि आप शीघ्रातिशीघ्र चोरों का पता लगाएँ और हमारी चीजों को दिलवाने की कृपा करें ।

आपका,
बी. सुरेश

10. किन्हीं पाँच (5) शब्दों के विलोम शब्द लिखिए ।

(1) अपराध × निरपराध
(2) एक × अनेक
(3) क्रय × विक्रय
(4) कर्म × अकर्म
(5) इधर × उधर
(6) ऊँचा × नीचा
(7) क्रोध × हँसी
(8) कुपुत्र × सुपुत्र
(9) घर × बाहर
(10) पक्ष × विपक्ष

AP Inter 1st Year Hindi Model Paper Set 9 with Solutions

11. किन्हीं पाँच शब्दों के समानार्थी शब्द लिखिए ।

(1) अलि = भौंश, भ्रमर, मधुप, मधुकर ।
(2) इन्द्र = देवराज, सुरेश, सुरेंद्र, सुरपति ।
(3) क्रोध = गुस्सा, रोष, आक्रोश, अमर्ष ।
(4) गंगा = देवनदी, सुरसरि, भगीरथी, जाह्नवी ।
(5) उन्नति = प्रगत, वकास, उत्थान, उत्कर्ष ।
(6) किरण = कर, मरीचि, रश्मि, अंशु ।
(7) राजा = नृप, नुपति, साम्राट, नरेश ।
(8) सेना = कंटक, दल, वाहिनी, फौज |
(9) नदी = सरिता, तटिनी, तरंगिणी, निर्भरिणी ।
(10) समुद्र = सागर, सिन्धु, रत्नाकर, जलधि ।

12. किन्हीं पाँच शब्दों की शुद्ध वर्तनी लिखिए ।

(1) भगीरथी – भागीरथी
(2) चरन – चरण
(3) पँख – पंख
(4) देनिक – दैनिक
(5) श्रंगार – श्रृंगार
(6) बन – वन
(7) दर्शनीयता – दर्शनीय
(8) नये – नए
(9) प्रात – प्रात
(10) अतह – अत:

13. किन्हीं पाँच शब्दों का अनुवाद हिन्दी में कीजिए ।

(1) Seminar – संगोष्टी
(2) Manager – प्रबंधक
(3) Cashier – रोकड़िया
(4) Arts – कला
(5) Maths – गणित
(6) Play ground – मैदान
(7) Peon – चपरासी
(8) Junior – कनिष्ठ
(9) Award – पुरस्कार योग्यता
(10) Accountant – खाता

14. कारक चिह्नों की सहायता से रिक्त स्थानों की पूर्ति कीजिए ।

(1) रावण पर राम ने विजय प्राप्त की ।
(2) करण की बहन सुशीला है ।
(3) शेर जंगल में रहता है ।
(4) हम देश केलिए मर मिटने को तैयार हैं ।
(5) रहीम ने दोहे लिखे

15. सूचना के अनुसार वाक्य में परिवर्तन कीजिए ।

(1) आज माली नहीं आया । (रेखांकित शब्द का लिंग बदलकर लिखिए |)
उत्तर:
आज मालिन नहीं आथी ।

(2) दिल्ली की सड़क साफ है । (रेखांकित शब्द का वचन बदलकर लिखिए |)
उत्तर:
दिल्ली की सड़के साफ है ।

(3) राम सुपुत्र है | (रेखांकित शब्द में उपसर्ग क्या है ?)
उत्तर:
सु

(4) घर की बनावट सुन्दर है । (रेखांकित शब्द में प्रत्यय क्या है ?)
उत्तर:
आवट

(5) वे मैदान में खेलते हैं । (भविष्यत् काल में लिखिए |)
उत्तर:
वे मैदान में खेलेंगे ।

16. सुचना के अनुसार भाषा विभाग को पहाचानिए :

(a) प्रेमचन्द उपन्यास लिखते हैं । (इस वाक्य में संज्ञा क्या है)
उत्तर:
प्रेमचन्द्र

(b) आप इधर आइए । (इस वाक्य में सर्वनाम पहचानो)
उत्तर:
आप

(c) होशियार लड़का प्रथम आया । (इस वाक्य में विशेषण क्या है)
उत्तर:
होशियार

(d) शिवम नहा रहा है । (इस वाक्य में क्रिया शब्द क्या है)
उत्तर:
नहारहा

(e) वह काफी धनवान है । (इस वाक्य में क्रिया विशेषण पहचानकर लिखिए)
उत्तर:
काफी

AP Inter 1st Year Sanskrit Model Paper Set 6 with Solutions

Self-assessment with AP Inter 1st Year Sanskrit Model Papers Set 6 allows students to take charge of their own learning.

AP Inter 1st Year Sanskrit Model Paper Set 6 with Solutions

Time : 3 Hours
Max Marks : 100

सूचना : प्रथम, द्वितीय, तृतीयप्रश्नान् अनुवादप्रश्नं च विहाय अन्ये प्रश्नाः संस्कृतभाषायामेव समाधातव्याः ।

अ) दानं भोगो ………….. तृतीयगतिर्भवति ॥
जवाब:
दानं भोगो नाशः तिस्रो गतयो भवन्ति वित्तस्य ।
यो न ददाति न भुङ्क्ते तस्य तृतीयागतिर्भवति ॥
Three are the ways to money Charity, enjoyment and loss. The money not given in charity or spent for enjoyment will be lost.

आ) अपूज्या यत्र ………………. मरणं भयम् II
जवाब:
अपूज्या यत्र पूज्यन्ते पूज्यानां च व्यतिक्रमः ।
त्रीणि तत्र भविष्यन्ति दुर्भिक्षं मरणं भयम् ||
Where the unworthy are honoured and the worthy are not honoured, three things happen there famine, death and fear.

II. एकं निबन्धप्रश्नं समाधत्त ।

अ) “मायावदृ” इति पाठ्यभागस्य सारांश संक्षेपेण लिखत ।
जवाब:
Introduction : The lesson “Mayavatu” is taken from the work Kumarasambhava written by Kalidasa. The fifth canto of the work describes the arrival of Siva in the guise of a celibate to test Parvati.

The false celibate: A celibate, shining with spiritual glow, wearing deerskin and holding a staff of Palasa entered the hermitage of Parvati like the embodiment of Brahmacharya. After receiving the hospitality of Parvati he asked her whether fuel sticks and kusa grass were easily available. He further said that physical body is the primary means of dharma. शरीरमाद्यं खलु धर्मसाधनम् ।

The celibate’s query: Then he asked her the reason for her penance saying that friendship with the good happens in seven steps or words. मनीषिभिः साप्तपदीनमुच्यते | She was born in a noble family, beautiful, wealthy and young. She could not face any humiliation in her parental house. Her father’s abode was divine. She should not search for a husband as a gem does not search, it is searched for. न रत्नमन्विष्यति मृग्यते हि तत् । He offered her half of his penance to get her desired husband. The reason for Parvati’s penance: The friend of Parvati told the celibate that having rejected Indra and others, Parvati wanted Siva as her husband. He was not to be won over by beauty as he destroyed Manmatha. When Parvati confirmed it, the celibate said that he would not support her as he knew Siva as one indulged in inauspicious practices.

Censure of Siva by the celibate: The celibate said that it would be inappropriate to let ashes take the place of sandal paste on the bosom of Parvati. People would smile seeing her riding a bull. Siva had deformed body. His birth was unknown. His wealth was indicated by his nakedness.

Parvati’s reply: Then Parvati replied angrily that he did not know Siva properly. Stupid people despise the acts of the noble. Though penniless, Siva was the source of riches. He was the lord of the three worlds. No one knew whether his body shone with ornaments or snakes, elephant hide or fine garments, and whether he held a skull or sported the crescent moon on the crest. Indra, who rode an elephant bowed to him. How could anyone know the origin of Siva, who was considered the cause of the creator ? She said that her mind was fixed on him with the feeling of love.

When Parvati was about to go from there, Siva revealed himself to her and said that he became her slave. Fruition makes exhaustion fresh again. क्लेशः पुनर्नवतां विधत्ते ।

आ) उपवासदीक्षिताः कपयः कथं बुभबक्षापीडिताः ?
जवाब:
Introduction: The lesson: was written by Srisailam Tatacharya, who was known as D. T. Tatacharya. He authored Kavyalankara and Mugdhanjali. In this satirical work he advises that imitating others is not good.

The greatness of the monkey race: Once the lord of the monkeys assembled all his followers and told them that previously their forefathers became great by their valour, strength and learning. Hanuman, who crossed the ocean, torched the city of Lanka and mastered the nine grammars belonged to their race only. He asked them that by what quality the humans became superior to them.

The decision of the monkeys: Then he asked them to find out why the monkeys should not have the same authority as the humans had. He told them that they should follow the customs or otherwise their animalhood would be well established. He suggested fasting on the day after as it was the Ekadasi day. The monkeys agreed to it.

Fasting of the monkeys on the Ekadasi day: The next day the monkeys took bath in the river and started their fast. But who could control his natural impulses? The monkeys became restless in a moment. They yawned, shook their hands, stretched their legs and kissed the tips of their tails. Then one of them suggested that they should go to the Jambu tree so that the next day breaking the fast would be easier.

The others agreed to it and moved to the tree. As they sat under the tree in a row, one of them suggested that they should climb the tree, and all of them did so. The juicy Jambu fruits: The monkeys passed the minutes like aeons. As the juicy fruits fell down making sounds when the branches were moved by the wind, their mouths watered. Then one of them said that they could as well keep the ripe fruits in the bag between their chins so that there would be no need to search for fruits the next day. Agreeing to it the monkeys jumped down and started to fill their mouths with the fruits.

The poet ends the story saying that what should be done afterwards need not be told to them.

III. एकं निबन्धप्रश्नं समाधत्त ।

अ) कामावेशात् निवर्तयितुं पुण्डरीकं प्रति कपिञ्जलः किमुवाच ।
जवाब:
Introduction : The lesson Kapinjalopadesa is an extract from the Kadambari, written by Bana. Pundarika, an ascetic fell in love with a celestial nymph Mahaswetha, His friend Kapinjala advised him that it was not proper for him to lose his selfcontrol. He asked him to condemn the Love-god.

Kapinjala’s questions :
Kapinjala told Pundarika that it was not proper for Punda-rika to behave in such a way as the common men would behave. The ascetics had the wealth of courage. धैर्यधना हि साधवः । He asked Pundarika why he did not restrain himself. How he was attacked by the senses. Where had his courage, self-restraint, calmness, celibacy, learning, disinterestedness and distaste for pleasures gone? If people like him were also sullied by passion then there was no use of intellect, study of the scriptures, culture, knowledge received from the teachers, and awakening. निरर्थकः संस्कारः |

He asked him how did he not notice the crystal rosary that fell from his hand and taken away by that evil woman. He exclaimed that Pundarika was absent minded, and thought of rescuing him before his heart was stolen by that evil woman. अहो विगत चेतनत्वम् ।

When Pundarika accepted that he was attracted by the beauty of Mahaswetha, Kapinjala further advised thus.

Kapinjala’s advice :
He asked Pundarika whether it was taught by his teachers of he studied it in the scriptures. Was it a means to acquire Dharma, a new kind of penance, a mystic vow or path leading to heavens? Could he even imagine this, let alone tell or see ? Why did he allow the Cupid make him a laughing-stock? Only fools are tormented by the Cupid मूढो हि मदनेन आयास्यते Could he have happiness in something condemned by the virtuous and honoured only by the vile ?

He said that he was a fool, who watered a garden of poisonous plants, embraced a sword creeper, held a black serpent or touched a burning coal thinking them to be something else. If he could not restrain his senses or control his mind, what was the use of his knowledge like that of the light of a firefly? And he advised him to condemn the stupid Love-god.

आ) शबरी रामाय को कथामुक्तवती ?
जवाब:
The Teacher’s Query:
Once a teacher, while going to the forest along with his students asked them which water was the best in the world. The students gave different answers such as the water of the Gangas, rain water and tears. The teacher then told them a story.

Sabari’s Hospitality :
While searching for Sita in the forest, Rama and Lakshmana met Sabari. She worshipped Rama, and offered him sweet fruits. There were beautiful and fragrant flowers in front of the place where Sabari lived. Rama asked her about those flowers.

Matanga and his students :
Sabari told him that the hermitage of Matanga used to be there at that place. Many students came for their studies to the hermitage. Once it was the end of the summer, and monsoon was about to set in. But no arrangement to store fuel sticks was made.

Sage Matanga and his students went to the forest, and cut wood. The young and the old carried the load on their heads and returned to the hermitage. Every- one was drenched in sweat. Beads of sweat dropped from the bodies on the ground.

The fragrance :
The next day everyone at the hermitage was surprised at the fragrance that spread there. The students found that the fragrance came from the direction of the forest where they went the day before. They went there and saw beautiful flowers blossomed here and there. Those flowers were not there the day before. The students ran back to Matanga, and reported to the teacher that strange inci- dent. Matanga went there, and touched those flowers affectionately.

The flowers of sweat :
Matanga told his students that the flowers were born from their sweat. Sweat born of labour was pure. The heart of Mother Earth seemed to have bloomed up on seeing their effort. He told them that there was nothing purer than physical labour. If the farmer and the weaver did not work, there would be no crop or clothes. The world moved on because of the labourers. He advised them to respect the workers. The students promised Matanga that they would respect the labourers and see that they were comfortable.

Rama and Lakshmana bowed to those flowers and went away.

The real sweat :
Having told the story the teacher explained to his students that the sweat from which any useful thing was produced was the sacred and true sweat. He advised his students to make the lives of the workers happy when they grew up.

IV. चतुर्णां प्रश्नानां समाधानानि लिखत । (4 × 2 = 8 M)

अ) चाणक्यः कस्य राज्ञः अमात्यः ?
जवाब:
चाणक्यः चन्द्रगुप्तस्य राज्ञः अमात्यः । सः नन्दवंशम् उन्मूल्य चन्द्रगुप्तं सिंहासने अध्यारोपयत् ।

आ) राजसान्नवाच्याः के भवन्ति ?
जवाब:
अतिकषायपदार्थः, अतिलवणम्, अत्युष्णापदार्थः अतिक्षारपदार्थः, अत्याम्लपदार्थः, अतिक्षातिदाहपदठार्थाः राजसान्नवाच्या भवन्ति ।

इ) बोपदेवविरचितः व्याकरणशास्त्रग्रन्थः कः ?
जवाब:
बोपदेवविरचितः व्याकरणशास्त्रग्रन्थः मुग्धबोधः ।

ई) कृपणस्य अग्रे गगनं दृष्ट्वा सर्वे किं कृतवन्तः ?
जवाब:
कृपणस्य अग्रे गमनं दृष्ट्वा सर्वे अट्टहासं कृतवन्तः ।

उ) नागार्जुनः कः ?
जवाब:
नागार्जुनः रसायनशास्त्रज्ञः प्रसिद्धः चिकित्सकः च ।

ऊ) पेटिकायां व्यापारी किम् अपश्यत् ?
जवाब:
पेटिकायां व्यापारी एवं जीर्णं धौतवस्त्रम् एकं प्राचीनं करांशुकम् एकं पुरातनं पादत्राणम् च अपश्यत् ।

V. द्वयोः संदर्भ व्याख्यानं लिखत ।

अ) अचक्षुर्विषये घोषं वारणस्येव नर्दतः ।
जवाब:
परिचयः – एतत् वाक्यं दशरथस्य पश्चात्तापः इति पाठ्यभागात् सवीकृतम् । अयं पाठ्यभागः
वाल्मीकिरामायणे अयोद्याकाण्डतः स्वीकृतः ।
सन्दर्भ: – दशरथः कौसल्यां प्रति मुनिशापवृत्तान्तम् कथयन् एवम् उक्तवान् ।
भावः – अदृष्टिगोचरे रात्रौ गजस्य शब्दः इव श्रुतः ।
विवरणम् – दशरथः कौसल्याम् उक्तवान् यत् सः एकदा रात्रौ मृगयार्थं सरयूतीरं गतवान् । तत्र रात्रौ जलेन पूर्यमाणस्य कुम्भस्य शब्दः गजस्य शब्दः इव श्रुतः ।

आ) न रत्नमन्विष्यति मृग्यते हि तत् ।
जवाब:
परिचयः एतत् वाक्यं मायावटुः इति पाठ्यभागात् स्वीकृतम् । एषः भागः कालिदासस्य कुमारसम्भवे पञ्चमसर्गात् स्वीकृतः ।
सन्दर्भः – मायावटुः पार्वतीम् एवं वदति ।
भावः – रत्नं अन्वेषणं न करोति । तत् अन्विष्यते ।
विवरणम् : पार्वत्याः तपोवनं एकः जटिलः प्रविवेश । सः अवदत् यत् यदि पार्वती पतिमिच्छति तपसा अलम् । रत्नं न अन्विष्यति । तत् अन्वेष्यते ।

इ) भवति सकलभाषाजन्मदात्री यतस्त्वम् ।
जवाब:
परिचयः – एतत् वाक्यं अमरवाणीप्रशस्तिः इति पाठ्यभागात् स्वीकृतम् । अस्य कविः श्रीमान् ई. शठकोपाचार्यः ।
सन्दर्भः – कविः अमरवाणीं प्रशंसन् एवं वदति ।
भावः – त्वम् सकलभाषाणां जननी ।
विवरणम् – जगति सर्वाः भाषाः संस्कृतसंपर्केण विराजन्ते । संस्कृतभाषा सकलभाषाजननी ।

ई) यतः प्रकृष्येत मनुष्यवर्गः ।
जवाब:
परिचयः – एतत् वाक्यं कपीनामुपवासः इति पाठ्यभागात् स्वीकृतम् । अस्य कविः श्रीशैलं ताताचार्यः ।
सन्दर्भः – कपिसार्वभौमः अन्यान् कपीन् एवं वदति ।
भावः – मनुष्याः कथं प्रकर्षम् आप्नुवन्ति ।
विवरणम् – कपिसार्वभौमः वदति यत् कपयः लोकत्रये प्रसिध्दाः बभूवुः । परन्तु केन मनुष्याः प्रकर्षम् वति ।

VI. द्वयो: संदर्भ व्याख्यानं लिखत ।

अ) अहो विगतचेतनत्वम् ।
जवाब:
परिचयः एतत् वाक्यम् ‘कपिञ्जलोपदशः इति पाठ्यभागात् स्वीकृतम् । अस्य मूलग्रन्थः कादम्बरी । अस्य कविः बाणः ।
सन्दर्भः महाश्वेतायाम् अनुरक्तम् पुण्डरीकम् सन्मार्गे प्रवर्तयितुं कपिञ्जलः एवम् उपदिशति ।
अर्थः – अहो, अचेतनत्वम् ।
विवरणम् – कपिञ्जलः पुण्डरीकं वदति यत् पुण्डरीकस्य स्थितिः तस्य अनुरूपा न । सः अचेतनः इवास्ति । करतलात् भ्रष्टाम् अक्षमालाम् अपि न लक्षयति ।

आ) कुम्भकारोऽपि यो विद्वान् स तिष्ठतु पुरे मम ।
जवाब:
परिचयः – एतत् वाक्यं भोजस्य औदार्यम् इति पाठ्यभागात् स्वीकृतम् । अस्य कविः बल्लालः ।
सन्दर्भः – भोजः मुख्यामात्यम् एवम् अवदत् ।
भावः – विद्यावान् चेत् कुम्भकारः अपि मम नगरे निवसतु ।
विवरणम् – कदाचित् भोजः मुख्यामात्यम् एवम् अवदत् – अविद्वान् चेत् ब्राह्मणः अपि नगरात् बहिः गच्छेत् । विद्वान् चेत् कुम्भकारः अपि नगरे वसतु ।

इ) पुण्यरूपाणि एतानि पुष्पाणि वन्दे |
जवाब:
परिचयः – एतत् वाक्यं स्वेदस्य पुष्पाणि इति पाठ् यभागात् स्वीकृतम् । अस्य रचयित्री डा. माधवी जोषी ।
सन्दर्भः – रामः शबरीम् एवं वदति ।
भावः – मातङ्गस्य पुण्यरूपाणि एतानि पुष्पाणि नमस्करोमि ।
विवरणम् – शबरी स्वेदपुष्पाणां कथां रामाय अकथयत् । तत् श्रुत्वा रामः तानि पुष्पाणि नमस्कृतवान् ।

ई) स्वच्छतायै प्रयतामहे ।
जवाब:
परिचयः – एतत् वाक्यम् वर्षापरिदेवनम् इति पाठ्यभागात् स्वीकृतम् । अस्य रचयिता आचार्य गुल्लपल्लि श्रीरामकृष्णमूर्तिः ।
सन्दर्भः – स्वच्छतापरिरक्षणार्थं प्रयत्नं करणीयमिति वदन्ती वर्षादेवी एवं प्रबोधयति ।
भावः – स्वच्छतार्थं प्रयत्नं कुर्मः ।
विवरणम् – विश्वं कलुषितम् इति दुःखित्वा वर्षादेवी स्वच्छतापरिरक्षणार्थं जनान् प्रबोधयति । स्वच्छतार्थं प्रयत्नं कुर्मः । यः क्रियावान् स पण्डितः । इति ।

VII. त्रयाणां प्रश्नानां समाधानानि लिखत ।

अ) मुनिः दशरथं किमिति शशाप ?
जवाब:
मुनिः दशरथं राजन् पुत्रशोकेन त्वं कालं करिष्यसि इति शशाप |

आ) धर्मसाधनेषु आद्यं किम् ?
जवाब:
धर्मसाधनेषु आद्यं शरीरम् ।

इ) पुराकृतानि पुण्यानि मानवान् कथं रक्षन्ति ?
जवाब:
पुराकृतानि पुण्यानि मानवान् वने रणे शत्रुजलाग्निमध्ये, समुद्रे, पर्वते, सुप्तमपि, प्रमत्तमपि, विषमस्थितमपि रक्षन्ति ।

ई) किम् अश्वमेधेन समं विदुः ?
जवाब:
दरिद्राय कृतं दानं, शुन्यलिङ्गपूजनम्, अनाथप्रेतदहनं च अश्वमेधसमं विदुः ।

उ) सकलभाषाजन्मदात्री का ?
जवाब:
सकलभाषा जन्मदात्री अमरवाणी ।

ऊ) कोऽपि कपिः कथं मौनं बिभेद ?
जवाब:
जृम्भां विधाय, हस्तं व्यवधूय, पादं प्रसार्य, गात्रं परिवर्त्य, पुच्छाग्रम् आचुम्ब्य कोऽपि कपिः मौनं बिभेद |

VIII. त्रयाणां प्रश्नानां समाधानानि लिखत ।

अ) कपिञ्जलोपदेशं कस्मात् ग्रन्थात् स्वीकृतम् ?
जवाब:
कपिञ्जलोपदेशः कादम्बरी इति गद्यकाव्यात् स्वीकृतः । अस्य काव्यस्य कर्ता बाणभट्टः ।

आ) धनिनां धनं किम् ?
जवाब:
यत् ददाति यत् अश्नाति तदेव धनिनां धनम् ।

इ) तीक्ष्णदंष्ट्रस्य धर्मोपदेशं श्रुत्वा शशकः किमवदत् ?
जवाब:
तीक्ष्णदंष्ट्रस्य धर्मोपदेशं श्रुत्वा शशकः एवम् अवदत्-एष नदीतीरे तपस्वी धर्मवादी तिष्ठति । तदेनं पृच्छावः ।

ई) बालकाः स्थाने स्थाने कानि दृष्टवन्तः ?
जवाब:
बालकाः स्थाने स्थाने सुन्दराणि पुष्पाणि विकसितानि दृष्टवन्तः ।

उ) व्यासाश्रमद्वारा प्रकटिताः ग्रन्थाः के ?
जवाब:
व्यासाश्रमद्वारा प्रकटिताः ग्रन्थाः योगवासिष्ठम्, शङ्करविजयः, पातञ्जल योगदर्शनम्, विवेकचूडामणिः, अष्टावक्रगीता, बृहदारण्यकोपनिषद्, छान्दोग्योपनिषद् इत्यादयः ।

ऊ) स्वच्छताया विषये सर्वकारः किं करोति ?
जवाब:
स्वच्छताया विषये सर्वकारः बहुयन्नान् करोति । नगरपालिकासु, परिशुद्धकर्मचारिणः नियुक्ताः । आकाशवाणी-दूरदर्शनीद्वारा बहु प्रचारयति ।

IX. विरामग्रहणाय अभ्यर्थनपत्रं लिखत ?
जवाब:

गुन्टूर्
दिनाङ्कः 20-11-2018

सविधे –
प्रधानाध्यापकः / प्रांशुपालः
प्रभुत्व (जूनियर्) उच्चमाध्यमिक कलाशाला,
गुन्टूर्
मान्याः !
विषयः विरामस्य कृते अभ्यर्थनम् ।
मम सोदर्याः विवाहार्थम् अहं श्वः स्वग्रामं गमिष्यामि । अतः कृपया दिनत्रयस्य 21-11-2018 तः 23-11-2018 पर्यन्तं विरामं यच्छन्तु इति सादरप्रणामपूर्वकं विज्ञापयामि । मम अनुपस्थितिसमये पाठ्यमानान् पाठान् मित्रसकाशात् ज्ञात्वा अहं पठिष्यामि ।
सधन्यवादम् ।

भवतां विधेयः छात्रः
नाम: × × × × × × ×
अनुक्रमसंख्या × × × × × × ×
प्रभुत्व जूनियर् कलाशाला,
गुन्टूर् ।

(अथवा)

पुस्तकप्रेषणविषये पत्रं लिखत ?
जवाब:

कडप
दिनाङ्कः 20-01-2018

सविधे –
मान्यसञ्चालकाः,
सरस्वतीविद्याप्रकाशन्,
विजयवाटिकॉ
मान्याः !
भवद्भिः प्रकाशितेषु अधोनिर्दिष्टानि पुस्तकानि मया यथानिर्दिष्टम् अपेक्ष्यन्ते ।

क्र.सं पुस्तकनाम कविः प्रतय:
1 कुमारसम्भवम् कालिदासः 5
2 श्रीमद्रामायणम् वाल्मीकिः 4
3 पञ्चतन्त्रम् विष्णुशर्मा 6
4 भर्तृहरिसुभाषितानि भर्तृहरिः 3

कृपया एतानि पुस्तकानि वि.पि. पि. द्वारा अधोसूचितं सङ्केतं प्रति प्रेषयन्तु ।
सधन्यवादम् ।

भवदीयः / भवदीया
नाम …………
गृहसंख्या 2-11-18
गान्धीरोड़, कडप ।

X. द्वयोः शब्दयोः सविभक्तिकरूपाणि लिखत । (2 × 6 = 12 M)

अ) रवि
जवाब:
AP Inter 1st Year Sanskrit Model Paper Set 6 with Solutions 1

आ) नदी
जवाब:
AP Inter 1st Year Sanskrit Model Paper Set 6 with Solutions 2

इ) फल
जवाब:
AP Inter 1st Year Sanskrit Model Paper Set 6 with Solutions 3

ई) किम् (पुं.)
जवाब:
AP Inter 1st Year Sanskrit Model Paper Set 6 with Solutions 4

XI. द्वयोः धातुरूपाणि लिखत | (2 × 3 = 6 M)

अ) लिखेत्
जवाब:
AP Inter 1st Year Sanskrit Model Paper Set 6 with Solutions 5

आ) खादतु
जवाब:
AP Inter 1st Year Sanskrit Model Paper Set 6 with Solutions 6

इ) भवति
जवाब:
AP Inter 1st Year Sanskrit Model Paper Set 6 with Solutions 7

ई) मोदिष्यते
जवाब:
AP Inter 1st Year Sanskrit Model Paper Set 6 with Solutions 8

XII. त्रीणि सन्धिनामनिर्देशपूर्वकं विघटयत ।

अ) वाणीशः
जवाब:
वाणी + ईश = सवर्णदीर्घ सन्धिः

आ) विक्रमोर्वशीयम्
जवाब:
विक्रम + ओर्वशीयम् = गुण सन्धिः

इ) वसुधैव
जवाब:
वसुधा + एव = वृद्धि सन्धिः

ई) वाण्येका
जवाब:
वाणी + एका = यणादेश सन्धिः

उ) पावकः
जवाब:
पौ + अक: = अयवायाव सन्धिः

ऊ) सोऽपि
जवाब:
सः + अपि = पूर्वरुप सन्धिः

XIII. त्रीणि सन्धिनामनिर्देशपूर्वकं सन्धत्त ।

अ) पितृ + ऋणम्
जवाब:
पितॄणाम् = सवर्णदीर्घ सन्धिः

आ) महा + इन्द्राः
जवाब:
महेन्द्रः = गुण सन्धिः

इ) महा + ऐश्वर्यम्
जवाब:
महैश्वर्यम् = वृद्धि सन्धिः

ई) धातृ + अंश:
जवाब:
धात्रंशः = यणादेश सन्धिः

उ) भानो + ए
जवाब:
भानवे = अयवायाव सन्धिः

ऊ) का + अपि
जवाब:
कार्येऽपि = पूर्वरुप सन्धिः

XIV. आन्ध्रभाषायां वा आङ्ग्लभाषायां वा अनुवदत ।

अ) अहं ग्रामं गच्छामि ।
जवाब:
I go to village.

आ) ते पाठम् अपठन् ।
जवाब:
Where did they go.

इ) यूयं कदा खादिष्यथ ।
जवाब:
When will you eat ?

ई) सः नगरं गमिष्यति ।
जवाब:
He will go to city.

उ) सर्वे गुणाः काश्चनमाश्रयन्ते ।
जवाब:
All the virtues depend on gold.

XV. एकेन पदेन समाधत्त ।

अ) दशरथः कां नदीम् अन्वगात् ?
जवाब:
सरयूनदीम् ।

आ) “मायावटुः” इति पाठ्यभागः कस्मात् स्वीकृत: ?
जवाब:
कुमारसम्भवात् ।

इ) घनस्य तृतीया गतिः का ?
जवाब:
नाशः ।

ई) प्रत्यक्षे स्तुत्याः के ?
जवाब:
गुरवः ।

उ) अनरवाणी सर्वमानवान् शुभमार्गे कैः गमयति ?
जवाब:
धर्मसूत्रैः ।

XVI. एकेन पदेन समाधत्त ।

अ) कपिञ्जलः कः ?
जवाब:
पुण्डरीकस्य मित्रम् |

आ) भोजस्य मुख्यामात्यः कः ?
जवाब:
बुद्धिसागरः ।

इ) तीक्ष्णदंष्ट्रः कः ?
जवाब:
अरण्यमार्जारः ।

ई) तृणवत् कान् न गणयेम ?
जवाब:
श्रामिकान् ।

उ) मार्गाः काभिः कलुषिताः ?
जवाब:
धूलिभिः ।

AP Inter 1st Year Zoology Question Paper June 2015

Varied difficulty levels in AP Inter 1st Year Zoology Model Papers and AP Inter 1st Year Zoology Question Paper June 2015 cater to students with diverse academic strengths and challenges.

AP Inter 1st Year Zoology Question Paper June 2015

Time : 3 Hours
Max. Marks: 60

General Instructions:
Note : Read the following instructions carefully

  1. Answer All questions of Section A. Answer ANY SIX questions in Section B and answer ANY TWO questions in Section C.
  2. In Section A questions from Sr. Nos. 1 to 10 are of “Very Short Answer Type”. Each question carries TWO marks. Every answer may be limited to 5 lines. Answer all these questions at one place in the same order.
  3. In Section ‘B’, questions from Sr. Nos. 11 to 18 are of “Short AnswerType”. Each question carries FOUR marks. Every answer may be limited to 20 lines.
  4. In Section ‘C’, questions from Sr. Nos. 19 to 21 are of “Long Answer Type”. Each question carries EIGHT marks. Every answer may be limited to 60 lines.
  5. Draw labelled diagrams wherever necessary in Sections ‘B’ and ‘C’.

Section – A (10 × 2 = 20)

Note : Answer all the questions in 5 lines each.

Question 1.
Differentiate between Protostomia and Deuterostomia.
Answer:
a) Protostomia: The eumetazoans in which blastopore develops into mouth are referred to as the protostomians. Eg : Annelida.
b) Deuterostomia : These are eucoelomates in which anus is formed from or near the blastopore. Eg : Echinodermata.

Question 2.
Distinguish between holocrine and apocrine gland.
Answer:

  1. Holocrine glands : In which the entire cell disintegrates to discharge the contents. Eg : sebaceous glands.
  2. Apocrine glands : In which the apical part of the cell is pinched off alpng with the secetory product. Eg : mammary gland.

Question 3.
What is haematocrit value ?
Answer:
The percentage of total volume occupied by RBCs is called haematocrit value.

AP Inter 1st Year Zoology Question Paper June 2015

Question 4.
What is cephalization ? How is it useful to its possessors ?
Answer:
Cephalization is concentration of nerve and sensory cells at the anterior end. As a result of cephalization, bilaterally symmetrical animals can sense the new environment into which they enter and respond more efficiently and quickly.

Question 5.
What is Aristotle’s Lantern ? Give one example of an animal possessing it.
Answer:
In the mouth of a sea-urchin a complex five jawed masticatory apparatus called Aristotle’s lantern is present.
Ex : Echinus sea urchin.

Question 6.
What are claspers ? Which group of fishes possesses them ?
Answer:
In the group chondrichthys males possess claspers on pelvic fins to facilitate internal fertilization Ex : shark.

Question 7.
Distinguish between proter and opisthe.
Answer:
During Transverse binary fission of paramoecium, two daughter paramoecia are formed. The upper or anterior one is proter which receive upper contractile vacuole cytopharynx and cytostome of its parent. The lower or posterior daughter is opisthe which receives the posterior contractile vacuole only.

Question 8.
Distinguish between synchronous and metachronous movements.
Answer:

  1. Synchronous movement : Cilia in a transverse row beat simultaneously in one direction. It is called synchronous movement.
  2. Metachronous movement : The sequential movement of cilia, in a longitudinal row, one after the other in one direction is called metachronous movement.

Question 9.
Define neoplasia. Give one example.
Answer:
Neoplasia: Some cause an abnormal growth of the host cells in a tissue to form new structures. This effect is called Neoplasia which leads to cancers. Eg: some viruses.

AP Inter 1st Year Zoology Question Paper June 2015

Question 10.
Define entropy.
Answer:
Energy lost or not available for work in a system is called ENTROPY.

Section – B (6 × 4 = 24)

Note : Answer any six questions in 20 lines each.

Question 11.
What is the evil quartet ?
Answer:
The following are the four major causes for accelerated rates of species extinction in the world. These causes are referred to as evil quartet.
a) Habitat loss and fragmentation : These are most important reasons for the loss of biodiversity.
b) Over exploitation : When need turns to greed, it leads to over exploitation.
c) Invasion of Alien species : When Alien species are introduced into a habitat, they turn invasive and establish themselves at the cost of indigenous species.
d) Co – extinctions : In an obligate association between a plant and an animal, if a plant becomes extinct, the animal also becomes extinct as seen in a parasitic and host association.

Question 12.
Describe the structure of a multipolar neuron.
Answer:
A neuron usually consists of a “cell body” with one to many dendrites and a single axon.
AP Inter 1st Year Zoology Question Paper June 2015 1

Cell body: It is also called perikaryon, cyton or soma. It contains abundant granular cytoplasm and a large spherical nucleus. The cytoplasm has Nissl bodies (they represent RER, the sites of protein synthesis), neurofibrils and lipofuscin granules (the products of cellular wear and tear, accumulating in lysosomes with age). A group of cell bodies in the central nervous system is called a ‘nucleus’, and in the peripheral nervous system, it is called a ‘ganglion’.

Dendrites :
Several short, branched processes which arise from the cyton are called dendrites. They also contain Nissl bodies and neurofibrils. They conduct nerve impulses towards the cell body (afferent processes).

Axon :
An axon is a single, long, cylindrical process that originates from a region of the cyton called axon hillock. Plasmalemma of an axon is called axolemma, and the cytoplasm is called axoplasm, which contains neurofibrils. However, Nissl bodies are absent. An axon may give rise to collateral branches. Distally it branches into many fine filaments called telodendria, (axon terminals), which end in bulb like structures called synaptic knobs or terminal boutons. Synaptic knobs possess ‘synaptic vesicles’ containing chemicals called neurotransmitters. Axon transmits nerve impulse away from the cyton (efferent process) to an interneuronal or neuromuscular junction called synapse.

Question 13.
What are the chief characters of crustaceans ?
Answer:

  1. They are aquatic.
  2. Head and thorax fuse forming the cephalothorax (covered by chitinous carapace). In some the exoskeleton is hardened by calcium carbonate (crabs and lobsters).
  3. Cephalic region bears two pairs of antennae (antennules and antennae – unique feature), one pair of mandibles and two pairs of maxillae.
  4. Thoracic and abdominal appendages are ‘biramous’.
  5. Respiratory organs are gills (branchiae).
  6. Excretory organs are green glands or antennary glands.
  7. Sense organs include antennae compound eyes, statocysts, etc.
  8. Development is indirect and includes different larval forms. Examples : Palaemon (freshwater prawn), Cancer (crab), Balanus (rock barnacle), Sacculina (root-headed barnacle), Astacus (crayfish), Daphnia (waterflea).

AP Inter 1st Year Zoology Question Paper June 2015 2

AP Inter 1st Year Zoology Question Paper June 2015

Question 14.
What are the features peculiar to Ratitae birds ? Give two examples of Ratitae birds.
Answer:
Ratitae / Palaeognathae : They are modern flightless running birds. They are ‘discontinuous’ in their distribution like the lung fishes and marsupials. They are characterized by the presence of reduced wings, a raft like sternum without keel and males with penis. They do not possess syrinx, clavicles and usually preen gland.
E.g. Struthio camelus (African ostrich), Kiwi (National bird of New Zealand), Rhea (American ostrich), Dromaeus (Emu), Casuarius.

Question 15.
Give an account of pseudopodia.
Answer:
Pseudopodia : These are found in rhizopods. The pseudopodia are temporary extensions of cytoplasm that develop in the direction of the movement. These temporary structures are useful to move on the substratum as our legs do, hence the name ‘pseudopodia’. There are four kinds of pseudopodia namely lobopodia (blunt finger-like) as in Amoeba and Entamoeba, filopodia (fibre-like) as in Euglypha, reticulopodia (net-like) as in Elphidium and axopodia or heliopodia (sun ray-like) as in Actionophrys.
AP Inter 1st Year Zoology Question Paper June 2015 3

Question 16.
What is the need for parasites to develop special adaptations ? Mention sfome special adaptations developed by the parasites.
Answer:
Parasites have evolved special adaptations to meet the requirements and lead successful life in the hosts.

  • In order to live in the host, some parasites have developed structures like hooks, suckers, rostellum, etc., for anchoring. E.g. Taenia solium.
  • Some intestinal parasites have developed protective cuticle to withstand the action of the digestive enzymes of the host. E.g. Ascaris lumbricoides.
  • Some intestinal parasites produce anti enzymes to neutralize the effect of host’s digestive enzymes, e.g. Taenia solium.
  • Some parasites live as obligatory anaerobes as the availability of oxygen is very rare for them. e.g. Entamoeba histolytica, Taenia solium, etc.
  • Some intestinal parasites live as facultative anaerobes i.e., if oxygen is not available, they live anaerobically and if oxygen is available, they respire aerobically, e.g. Ascaris lumbricoides.
  • The morphological and anatomical features are greatly simplified while emphasizing their reproductive potential. For example, an Ascaris lays nearly two lakh eggs per day. In Taenia solium the body is divided into 700 to 900 proglottids of which each proglottid acts as a unit of reproductive system and releases approximately 35,000 eggs.
  • The life cycles of endoparasites are more complex because of their extreme specialization. For example, life cycle of certain parasites like Fasciola hepatica (sheep liver fluke) is very complex involving many developmental stages and two intermediate hosts to increase the chances of reaching a new definitive host.
  • Certain parasites like Entamoeba develop cysts to tide over the unfavourable conditions like desiccation while reaching the new host.
  • Some parasites elude production of vaccines against them (smart*parasites!) as they keep changing their surface antigens form time to time, e.g. Plasmodium, HIV, etc.

Question 17.
Draw a neat labelled diagram of the salivary apparatus of cockroach.
Answer:
AP Inter 1st Year Zoology Question Paper June 2015 4

AP Inter 1st Year Zoology Question Paper June 2015

Question 18.
What is summer stratification ? Explain.
Answer:
Summer stratification : During summer in temperate lakes, the density of the surface water decreases because of increase in its temperature (21 – 25°C). This ‘upper more warm layer’ of a lake is called epilimnion. Below the epilimnion there is a zone in which the temperature decreases at the rate of 1°C per meter in depth, and it is called thermocline or metalimnion. The bottom layer is the hypolimnion, where water is relatively cool, stagnant and with low oxygen content (due to absence of photosynthetic activity).
AP Inter 1st Year Zoology Question Paper June 2015 5
During autumn (also called Fall), the epiiimnion cools down, and the surface water becomes heavy when the temperatures 4°C, and sinks to the bottom of the lake. Overturns bring about ‘uniform temperature’ in lakes during that period. This circulation during the autumn is known as the fall or autumn overturn. The upper oxygen rich water reaches the hypoliminion and the nutrient rich bottom water comes to the surface. Thus there is uniform distribution of nutrients and oxygen in the lake.

Question 19.
Describe the life cycle of Plasmodium vivax in man.
Answer:
Life cycle of Plasmodium in man (The human phase) : In man, the Plasmodium reproduces by asexual reproduction called schizogony. It occurs in liver cells (hepatocytes) as well as in RBC. In liver cells, it is called hepatic schizogony and in RBC it is called erythrocytic schizogony.

Hepatic schizogony : This was discovered by Shortt and Garnham. Whenever, a mosquito infected by Plasmodium bites a man, nearly 2000 sporozoites are released into the blood of man through its saliva. Within half an hour, they reach the hepatocytes where they undergo pre-erythrocytic and exo- erythrocytic cycles.

Pre-erythrocytic cycle : Whenever the sporozoites reach the liver cells, they transform into trophozoites. Theyfeed on the contents of the hepatic cells, assume spherical shape and attain the maximum size. This stage is called schizont stage. Its nucleus divides several times mitotically, followed by the cytoplasmic divisions resulting in approximately 12,000 daughter individuals called cryptozoites or the lstgeneration merozoites. They enter the sinusoids of the liver by rupturing the cell membrane of the schizont and the liver cells. This entire process is completed approximately in 8 days. Now these first generation merozoites have two options i.e., they can enter either fresh liver cells and continue exo-erythrocytic cycle or they can enter RBC and continue erythrocytic cycle.

Exo-erythrocytic cycle : If the cryptozoites enter the fresh liver cells, they undergo the changes similar to that of the pre-erythrocytic cycle and produce the second generation merozoites called metacryptozoites. These are of two types – the smaller micro-metacryptozoites and larger macro-metacryptozoites. This entire process is completed approximately in two days. The macro-metacryptozoites attack fresh liver cells and continue another exo – erythrocytic cycle, whereas the micro-metacryptozoites always enter blood stream and attack fresh RBC to continue erythrocytic cycle.

Prepatent period : The interval between ‘the first entry of Plasmodium into the blood in the form of sporozoites and the second entry of Plasmodium into the blood in the form of cryptozoites is called prepatent period. It lasts approximately 8 days. During this period, the host does not show any clinical symptoms of the disease. It is only a means of multiplication.

Erythrocytic cycle : It was first described by Camillo Golgi. Hence it is also called Golgi cycle. This cycle is initiated either by the cryptozoites of pre-erythrocytic cycle or the micro- metacryptozoites of exp – erythrocytic cycle. In the fresh RBC, these stages assume spherical shape and transform into trophozoites. It develops a small vacuole which gradually enlarges in size, pushing the cytoplasm and nucleus to the periphery. Now the Plasmodium looks like a finger ring. Hence this stage is called signet ring stage. Soon it loses the vacuole, develops pseudopodia and becomes amoeboid stage. With the help of pseudopodia, it actively feeds on the contents of the RBC and increases in size. As a result, the RBC grows almost double the size.

This process is called hypertrophy. The malaria parasite digests the globin part of the ingested haemoglobin and converts the soluble haem into an insoluble crystalline haemozoin. It is called the ‘malaria pigment’ which is a disposable product. During this stage, small red coloured dots appear in the cytoplasm of the RBC known as Schuffner s dots. These are believed to be the antigens released by the parasite.

Now the Plasmodium loses the pseudopodia, further increases in size, occupies the entire RBC and becomes a schizont. It undergoes schizogony similar to that of the pre-erythrocytic cycle and produces 12 to 24 erythrocytic merozoites. They are arranged in the form of the petals of a rose in the RBC. Hence, this stage is called the rosette stage. Finally the erythrocyte bursts and releases the merozoites along with haemozoin into the blood. This cycle is completed approximately in 48 hours.

Incubation Period : The period between ‘the entry of Plasmodium into the blood in the form of sporozoite and the first appearance of symptoms of malaria in man’ is called incubation period. It is approximately 10 to 14 days.

Formation of gametocytes : After repeated cycles of erythrocytic schizogony, when the number of fresh RBC decreases, some merozoites enter the RBC and transform into gametocytes instead of continuing the erythrocytic cycle. This process generally takes place when the RBCs are present in spleen and bone marrow.

The gametocytes are of two types namely, smaller microgametocytes or male gametocytes and larger macrogametocytes or female gametocytes. The gametocytes cannot undergo further development in man as the temperature and pH of the blood of man are not suitable for further development. These gametocytes reach the blood circulation and wait to reach the next host. They degenerate and die if they are not transferred to mosquito within a week.
AP Inter 1st Year Zoology Question Paper June 2015 6

AP Inter 1st Year Zoology Question Paper June 2015

Question 20.
Describe the respiratory system of cockroach with the help of neat and labelled diagrams.
Answer:
Respiratory System of Periplaneta : Due to the absence of respiratory pigment, the blood of cockroach is colourless and it cannot carry oxygen to different tissues. Therefore a tracheal system is developed to carry the air directly to the tissues. The respiratory system of cockroach consists of stigmata, tracheae and tracheoles.

Stigmata or spiracles : The tracheal system communicates with the exterior by ten pairs of openings called stigmata or spiracles. The first two pairs of spiracles are present in the thoracic segments, one pair in mesothorax and one pair in the
AP Inter 1st Year Zoology Question Paper June 2015 7
metathorax. The remaining eight pairs are present in the first eight abdominal segments. Spiracles are located in the pleura of their respective segments. The respiratory system in insects is classified on the basis of number and nature of spiracles. The spiracles of cockroach are polypneustic (as they are more than 3 pairs) and holopneustic (as all of them are functional). All spiracles are valvular and each of them is surrounded by a chitinous ring called peritreme. All spiracles bear small hair like structures called trichomes to filter the dust particles. Each spiracle opens into a small chamber called atrium.

Tracheae : From the atrium of each thoracic spiracle several horizontal tracheae run inside. They join with each other in the thorax to form many tracheal trunks like dorsal cephalic, ventral cephalic trunks and their branches. These branches enter all organs of the head. The thoracic region also contains lateral longitudinal trunks. The abdominal spiracles lead into atria.

From the atrium of each abdominal spiracle three tracheal tubes arise. All these tracheal tubes of one side open into three separate longitudinal tracheal trunks. They are lateral, dorsal and ventral longitudinal trunks. Lateral longitudinal trunks are the longest tracheal trunks. The three pairs of longitudinal tracheal trunks of both the sides are interconnected by many commissural tracheae. From all the tracheal trunks several branches are given out, which enter different organs. All tracheal branches entering into an organ end in a special cell called tracheole cell.

The wall of the tracheae is made of three layers. They are an outer basement membrane, a middle one cell thick epithelium and an inner layer of cuticle called intima. The intima is produced into spiral thickenings called taenidia. The taenidia keep the tracheae always open and prevent it from collapsing.

Tracheoles : The terminal cell of trachea is called tracheoblast or tracheole cell. It has several intracellular tubular extensions called tracheoles. Tracheoles are devoid of intima and taenidia. They are formed of a protein called trachein. Tracheolar fluid is present inside the tracheoles. The level of the tracheolar fluid varies with the metabolic activity of the insect. It is more when the insect is inactive and completely reabsorbed into the tissues, when the insect is more active. Tracheoles penetrate the cell and are intimately associated with mitochondria (to supply oxygen to them).

Mechanism of respriation : Respiration includes two events, viz., inspiration and expiration. The muscles helpful are dorsoventral muscles and ventral longitudinal muscles. Dorsoventral muscles are the principal muscles of respriation.
AP Inter 1st Year Zoology Question Paper June 2015 8
Inspiration : Taking in of air is inspiration. It is effected by the relaxation of the dorsoventral muscles and ventral longitudinal muscles. Due to the relaxation of the dorsoventral muscles, tergal plates are elevated and the volume of the body cavity increases. Due to the relaxation of the ventral longitudinal muscles, the telescoped segments come to the normal position. So the volume of the body cavity increases in the longitudinal axis. As air is drawn in due to the relaxation of the muscles, the process is a ‘passive’ process. During inspiration the thoracic spiracles are kept open and the abdominal spiracles are kept closed.

Expiration : Sending out air from the body is called expiration. On contraction the dorsoventral muscles depress the tergal plates. Body cavity decreases in size and pressure increases. Due to the contraction of the ventral longitudinal muscles, the segments are telescoped and the volume of the body cavity decreases in the longitudinal axis increasing the pressure further. As this process involves the contraction of muscles, expiration is described as active process. During expiration thoracic spiracles are closed and abdominal spiracles are kept open.

Exchange of gases : As air enters the tracheoles, oxygen from the air is taken into the cells and CO2 is released into haemolymph. The CO2 from the haemolymph mostly goes out through the inter- segmental membranes of the body wall. Cockroach and some other insects such as grasshoppers and beetles exhibit the phenomenon of discontinuous ventilation. In this mode of respiration continuous exchange of gases is interrupted by extended periods during which spiracles remain closed.

The expulsion of CO2 from the body occurs in bursts, when the spiracles are open. The exchange of gases depends on the metabolic rate and temperature. When air enters the tracheoles, oxygen diffuses faster into the tissues due to its high partial pressure. At the same time the carbon dioxide of tissues, instead of passing into the tracheal system, goes into the haemolymph. Carbon dioxide is carried more quickly into the haemolymph due to its greater solubility in it. This CO2 accumulates near the spiracles and diffuses into the artial chambers near the spiracles and goes out in bursts through the abdominal spiracles. Opening and closing of spiracles is influenced by CO tension in haemolymph and oxygen tension in the trachea.

AP Inter 1st Year Hindi Model Paper Set 8 with Solutions

Exploring a variety of AP Inter 1st Year Hindi Model Papers Set 8 is key to a well-rounded exam preparation strategy.

AP Inter 1st Year Hindi Model Paper Set 8 with Solutions

Time : 3 Hours
Max Marks : 100

सूचनाएँ :

  1. सभी प्रश्न अनिवार्य हैं ।
  2. जिस क्रम में प्रश्न दिये गये हैं, उसी क्रम से उत्तर लिखना अनिवार्य है ।

खण्ड – ‘क’
(60 अंक)

1. काल करे सो आज कर, आज करे सो अब ।
पल में परलै होयगी, बहुरि करोगे कब ॥
उत्तर:
प्रसंग : यह दोहा कबीरदास के द्वारा लिखी गयी ‘साखी’ नामक रचना से लिया गया है | वे निर्गुणशाखा के अन्तर्गत ज्ञानमार्ग शाखा से संबंधित सन्त कवि थे ।
संदर्भ : समय का सदुपयोग करने का सन्देश कवि दे रहे हैं ।
व्याख्या : कवि का कहना है कि कल का काम आज करो और आज का काम अभी करो । क्यों कि एक क्षण मे कुछ भी हो सकता है । तब यह काम हम करेंगे या नहीं, कह नही सकते ।

विशेषताएँ :

  1. जीवन की क्षणिकता के बारे में कवि कह रहे हैं ।
  2. उनकी भाषा सदुक्कडी हैं ।

(अथवा)

रहिमन जिहवा बावरी, कहि गई सरग पताल ।
आपु तो कहि भीतर गयी, जूती खात कपाल ||
उत्तर:
प्रसंग : यह दोहा रहीम के द्वारा लिखी गयी दोहावली से लिया गया है। वे भक्तिकाल से सम्बन्धित कृष्ण भक्त कवि थे ।
संदर्भ : इसमें रहीम सब के साथ अच्छी तरह बातें करने की चेतावनी दे रहे हैं ।
व्याख्या : रहीम का कहना है कि मनुष्य की जिह्वा एक पागल जैसी है। कुछ न – कुछ फिसलकर बुरी बात कह देती हैं । लेकिन वह भला बुरा कहकर हमारे मुंह के अन्दर चली जाती हैं। लेकिन उन बुरी बातों का फल खोपडी (सिर) को लेना पडता हैं ।

विशेषताएँ :

  1. कवि इसमें व्यक्ति को सबसे अच्छी तरह व्यवहार करने का सन्देश दे रहे हैं ।
  2. उनकी भाषा व्रज भाषा हैं ।

2. किसी एक कविता का सारांश लिखिए ।

1) सुख – दुःख
उत्तर:
सारांश : कवि का कहना है कि हमेशा सुख और दुख भी जीवन के लिए अच्छा नही | सुख और दुख दोनों के साथ खेल मिचौनी करना चाहिए । अर्थात् जीवन में सुख और दुख एक दूसरे के साथ आना ही चाहिए । सुख और दुख होने के साथ ही जीवन परिपूर्ण होता है । जिस प्रकार आकाश में घने बादलों के बीच चन्द्रमा और चाँदनी से बादल घेरे जाते हैं उसी प्रकार सुख और दुख एक दूसरे के बाद आते जाते है ।

सारा जगत कभी कभी अति दुख से और अति सुख से पीडित होता रहता है । लेकिन मानव जीवन में सुख और दुख दोनों को समान रूप में स्वीकारना चाहिए । हमेशा सुख और हमेशा दुख दोनो भी जीवन के लिए दुखदायक है । जिस प्रकार जीवन में दिन और रात का आना जाना स्वाभाविक है । उसी प्रकार जीवन में सुख और दुख का आना जाना भी स्वाभाविक है ।

जिस प्रकार सायंकाल सूर्योदय का आगमन विरह के बाद मिलन जीवन के लिए आनन्ददायक होता है | आनन्द और दुख हमेशा जीवन मे आता जाता रहता है । यही मानव जीवन है ।

इस प्रकार कवि इसमें जीवन मे स्वाभाविक और प्राकृतिक नियमों के बारे मे चित्रण करते हुए जीवन के लिए सुख और दुख जितना स्वाभाविक होते है उनके बारे मे व्यक्त कर रहे है । सुख और दुख दोनो को समान रूप मे स्वीकारने का सन्देश दे रहे हैं । उनकी भाषा सरल खडीबोली है ।

AP Inter 1st Year Hindi Model Paper Set 8 with Solutions

2) भिक्षुक
उत्तर:
एक भिखारी की दयनीय स्थिति का वर्णन करते हुए निराला जी कह रहे हैं कि एक भिखारी टूटे हुए हृदय से अपनी दयनीय स्थिति पर पछताता हुआ उस पथ पर आ रहा है । उसका पेट और पीठ दोनों मिले हुए दिखाई पड रहे हैं । अर्थात भूख के कारण उसका पेट पीछे चला जाकर पीठ से मिल गया जैसा दिखाई पड रहा है । उसने चलने के लिए भी शक्ति नही है । इसलिए हाथ में डंडा लेकर धीरे-धीरे अपनी भूख मिटाने एक मुट्ठी भर अन्न के लिए फटे हुआ होल को मुँह फैलाता है । उसके साथ दो बच्चे भी हाथ फैलाकर चल रहे हैं। वे अपने बाँए हाथ से भूखे पेट को मल रहे है और दाहिनी हाथ से दया की भीख माँग रहे हैं। भूख के कारण उनके ओठ सूखे जा रहे हैं । दाताओं से भीख माँगकर अपने भाग्य परखने के लिए उनके पास शक्ति भी नही है। यदि कही सडक पर जूठी पत्तल चाटने के लिए मिले तो उनसे पहले ही उन जूठे पत्तलों को झपटने के लिए कुत्ते वहाँ खडे थे। इतनी दयनीय स्थिति उस भिखारी परिवार की थी ।

इस प्रकार कवि की प्रयोगवादी दृष्टिकोण इसमें दिखायी पड रहा है। भिखारी की दयनीय स्थिति के द्वारा शोषित वर्ग की ओर कवि संकेत दे रहे है । शोषक और शोषित वर्ग भिन्नता का स्पष्ट चित्रण वे दे रहे है । उनकी भाषा शुद्ध खडीबोली है ।

3. किसी एक पाठ का सारांश लिखिए ।

1) चन्द्रशेखर वेंकटरमन्
उत्तर:
प्रस्तुत निबन्ध में भारतीय वैज्ञानिक श्री चन्द्रशेखर वेंकटरमन के व्यक्तित्व और उनके अनुसंधानों पर प्रकाश डाला गया है । उनका जन्म तमिलनाडु के तिरूचिरापल्ली में सन् 1888 मे हुआ । उनके पिता चन्द्रशेखर अय्यर जो भौतिक और गणित के प्राध्यापक थे और उनकी माता पार्वती अम्मल थी । उनकी आरम्भिक शिक्षा विशाखपट्टनम् मे हुई। उन्होंने ग्यारह वर्ष की आयु मे मेट्रिक्युलेशन की परीक्षा, तेरह वर्ष में एफ. ए. की परीक्षा उत्तीर्ण की। वो मद्रास के प्रेसिडेन्सी कॉलेज में बी. ए. प्रथम श्रेणी में उत्तीर्ण हुए और गणित में एम. ए. प्रथम श्रेणी में उत्तीर्ण हुए ।

उन्होंने अपने शिक्षार्थी जीवन में कई महत्वपूर्ण कार्य किए । सन् 1906 मे प्रकाश विवर्तन पर उनका पहला शोध पत्र लंडन के फिलासाफिकल पत्रिका मे प्रकाशित हुआ । उसका शीर्षक था – आयताकृत छिद्र के कारण उत्पन्न असीमित विवर्तन पट्टियाँ । उन्हे वैज्ञानिक सुविधाएँ ठीक से न मिलने के कारण वित्त विभाग के असिस्टेंट एकांउटेंट जनरल बनकर कलकत्ता चले गए । इसके बाद वैज्ञानिक अध्ययन के लिए भारती परिषद की प्रयोगशाला मे प्रयोग के लिए अनुमति प्राप्त की । अपने घर में भी उन्होंने प्रयोगशाला बनाया । उसका उनका अनुसंधान क्षेत्र था – ध्वनि के कम्पन और कार्यों का सिद्धांत इसपर जर्मनी मे प्रकाशित भौतिक विश्वकोश में भी एक लेख लिखा । इसके बाद उन्होंने उच्च सरकारी पद त्याग कर कलकत्ता विश्व विद्यालय चले गए । वहाँ पर उन्होंने वस्तुओं मे प्रकाश चलने पर अध्ययन किया । और बाद में पार्थिव वस्तुओं को प्रकाश के बिखरने का नियमित अध्ययन किया जो रामन प्रभाव के नामसे जाना जाता है ।

इस `पर विशेष प्रयोग करके के उन्होंने पारद आर्क के प्रकाश को स्पेक्ट्रम स्टेक्ट्रोस्कोप ने निर्मित किया और प्रमाणित किया कि यह अन्तर पारद प्रकाश की तरंग लंबाइयों मे परिवर्तित होने के कारण पडता है इसके बाद उन्होंने रायल सोसयटी, लंदन के फेलो बने और रामन प्रभाव के लिए उन्हे सन् 1930 ई में नोबेल पुरस्कार दिया गया । सन् 1948 मे बंगलूर मे रामन शोध संस्थान की स्थापना की गई | उन्हें सन् 1954 ई. में भारत रत्न की उपाधि और सन् 1957 ई. में लेनिन शान्ति पुरस्कार प्राप्त हुई। 28 फरवरी, 1928 को उन्होंने रामन प्रभाव की खोज की । इसलिए प्रत्येक वर्ष इस दिन को राष्ट्रीय विज्ञान दिवस से रूप में मनाया जाता है । सन् 1970 में उनकी मृत्यु हो गई ।

इस प्रकार इसमें चन्द्र शेखर वेंकटरामन के व्यक्तित्व के साथ-साथ उनके अनुसंधान, भौतिकी के क्षेत्र में उनकी उपलब्धियों और अनुसंधान के क्षेत्र में उनकी खोजों पर विशेष महत्व दिया गया है । उन्हे प्राप्त राष्ट्रीय और अन्तर्राष्ट्रीय सम्मानों पर भी रेखांकित किया गया है ।

2) आन्ध्र संस्कृति
उत्तर:
संस्कृति अर्जित आचरणों की एक व्यवस्था है । संस्कृति मानव की जीवन पद्धति है और विचारों, आचरणों और जीवन के मूल्यों का सामूहिक नाम है। भारतीय संस्कृति के बारे में दिनकर जी का कहना है कि संस्कृति जिंदगी का एक तरीका है और यह तरीका सदियों से जमा होकर एक उस समाज मे छाया रहता है जिसमें हम जन्म लेते है ।

भारतीय संस्कृति वैदिक संस्कृति है । उसका प्रादेशिक रूप तेलुगु संस्कृति है और यही आन्ध्र संस्कृति कहलाती है । आन्ध्र राज्य का इतिहास शातवाहनों से आरंभ होता है। इनके समय मे आंध्र मे आर्य व द्रविड संस्कृतियों का अपूर्व संगम हुआ था । शातवाहनों के बाद आन्ध्र संस्कृति के विकास में इक्ष्वाकु, चोल, चालुक्य, पल्लव, काकतीय, विजयनगर राजाओं का महत्वपूर्ण योगदान रहा । काकतीयों के समय 14 वी शताब्दी में आंध्र मे मुसलमानों का प्रवेश हुआ। जिससे एक और नयी संस्कृति का समावेश हो गया । ऐतिहासिक व राजनीतिक रूप से आंध्र प्रदेश दो भागों में विभक्त है – कोस्ता तटीयान्ध्र तथा रायलसीमा । गोदावरी, कृष्ण, आन्ध्र की प्रमुख जीव- नदियाँ है इनके अलावा छोटी-छोटी नदियों भी प्रवाहि पायी जाती है । आन्ध्रप्रदेश कृषि प्रधान राज्य है और अनाज मुख्य फसल है इसके अलावा मकई, मिर्च, कपास, मूंगफली, तम्बाकू व जूट अन्य फसल है | आंगेलु पशुओं की भारतभर प्रसिद्धि है । आन्ध्र का एक विशेष उद्योग है – नौका निर्माण उद्योग |

आन्ध्र प्रदेश धार्मिक रूप से एक संपन्न राज्य है । यहाँ पर वैदिक, बौद्ध, जैन, अद्वैत, विशिष्टाद्वैत, इस्लाम, सिख, ईसाईधर्म, नास्तिक धर्म आदि विराजमान हैं । बौद्ध संस्कृति और जैन धर्म से संबंधित मन्दिर और स्तूप और अनेक विहार यहाँ पर व्याप्त है । हिन्दू देवी-देवताओं के मंदिर भी निर्मित हुए जैसे द्राक्षारामम्, हंपी, ताडिपत्रि, लेपाक्षी आदि। कला और संस्कृति का भी विकास यहां पर हुआ । यहाँ पर नाग, यक्ष जातियों के साथ-साथ अनेक पर्वत और जंगलों जातियों भी विकास हुआ ।

संस्कृति मानव जीवन की आदर्श आचार सहित है । संगीत, नृत्य, शिल्प, चित्रकलाओं के साथ हरिकथा, बुर्राकथा, चेंचु नाटक भी प्रचार मे है । अन्नमाचार्य, रामदास, त्यागराज और क्षेत्रच्या के साथ 3 बीसवी राती के मंगलंपल्लि बालमुरली कृष्ण भी प्रसिद्ध वाग्गेयकार थे । कूचिपूडि, भरतनाट्यम, कथकली, कथक नृत्यों के साथ कलंकारी, कोंडपल्लि गुडियाँ, एटिकोप्पाका गुडिया, मंगलगिरि, उप्पाडा, पोंडूरू, वेंकटगिरि वस्त्र आदि प्रसिद्धि है ।

आन्ध्र प्रान्त मे अनेक पर्व और त्योहार मनाये जाते है जैसे संक्रांति, महाशिवरात्रि, उगादि, श्रीरामनवमी एरूवाका पूर्णिमा, विनायक चविति, दशहरा, दीपावली रमजान क्रिसमस आदि बनाया जाता है | विवाह तो जीवन मे सबसे महत्वपूर्ण संस्कार है । यहाँ के स्त्री-पुरुष कईतरह के आभूषण पहना करते थे । अनेक तरह के खेल खेला करते थे । यहाँ के व्यंजन भी सांप्रदायिक और प्रसिद्ध है | चावल प्रधान भोजन है । अमरावती, अन्नवरम् तिरूपति, कनकदुर्गम्मा नन्दिर, पंचारामम यहाँ के प्रसिद्ध मन्दिर है ।

आंध्रसंस्कृति का आरंभ ही भारतीय संस्कृति की सुरक्षा के उद्देश्य से हुआ । संस्कृत के प्रायः सभी इतिहास, पुराण, काव्य व नाटक तेलुगु मे अनुदित हुआ है। आंध्र में अष्टावधान नामक एक विशिष्ट साहित्य प्रक्रिया विकसित हुआ । आन्ध्र की राजभाषा तेलुगु है । नन्नया, तिक्कना, एर्राप्रगडा ने महाकाव्य महाभारत का तेलुगु मे अनुवाद किया । प्राचीनकाल के रचनाकारों में पालकुरिक सोमनाथ, श्रीनाथ, पोतना और आधुनिक साहित्यकारों मे गुरजाडा, कंदुकूरी, कृष्णाशास्त्री, श्री श्री गुर्रम जाषुआ, चिन्नयसूरी जैसे और भी अनेक है ।

इस प्रकार आन्ध्र संस्कृति विभिन्न जाति, धर्म, जाति, व वर्ण के लोगों से मिश्रित है । फिर भी राज्य मे सामाजिक, सांस्कृतिक व धार्मिक एकरूपता का आभास स्पष्ट झलकता है ।

4. किसी एक कहानी का सारांश लिखिए ।

1) परदा
उत्तर:
चौदरी पीरबख्श गरीब मुंशी था । दो रूपये के किराये पर एक मकान में रहता था । बस्ती में केवल चौदरी पीरबख्श ही पढ़ा-लिखा आदमी था । उसको ड्योढ़ीपर परदा रहता था । इसलिए सब लोग उसे चौदरी साहब कहकर सलाम करते थे । नौकरी में केवल बारह रूपये मासिक वेतन मिलते थे । संताने पाँच थी ।

नौकरी से आमदनी कम इसलिए परिवार के लोगों को पेट भर खाना, शरीर को ढ़कने के लिए कपडे नही मिलते थे । परदे के पीछे रहनेवालों के शरीरों पर केवल चिथडे ही रहते थे ।

पैसों की जरूरत अधिक होने के कारण सूदखोर बाबर अली खाँ से ब्याज पर उधार लेता है । चौदरी पीरबख्श किश्तों में चुकाना वादा करता है। लेकिन चुका नही पाता । खान बार-बार पैसे मांगता है, घर आकर गालियाँ देता है ।

किश्तो के बदले ‘परदा’ लेजाने के लिए खान तैयार हो जाता है । लेकिन वह परदा ही घर की इज्जत की रक्षा करने वाला है । पीरबख्श परदे को न ले जाने के लिए प्रार्थना करता है, लेकिन खान गुस्से में आकर परदे को खींच लेता है तो घर की दयनीय स्थिति दिखायी देती है । अपनी इज्जत की रक्षा के लिए घर में इधर उधर भागते है, सब लोग । चौदरी पीरबख्श बेहोश होकर गिर जाता है । होश आने के बाद समझता है कि अब परदा लटकाने जरूरत नही । क्यों कि घर की इज्जत चली गयी ।

इस कहानी से मध्यवर्गीय लोगों के मिथ्याभिमान और उससे जुडी विवशता दयनीय स्थिति की जानकारी हो जाती है ।

2) पूस की रात
उत्तर:
हल्कू एक निर्धन किसान था । मुन्नी उसकी पत्नी थी । हल्कू खेती करता था । किन्तु उसकी उपज इतनी नहीं होती कि उसे बेचकर साहूकार का ऋण चुका सके। सर्दी से बचने के लिये वह एक कम्बल खरीदना चाहता था । इसके लिये उसने पत्नी के पास तीन रूपये जमाकर रखे ।

लेकिन साहूकार से तुरंत बचने के लिए उसने उन तीन रूपयों को उसे दे दिया । पूस की रात थी । हल्कू खेती की रखवाली करने गया था ।

कड़ी सर्दी थी । वह काँपने लगा । उसके पास का चादर उसे बचा न सकता था । सर्दी से बचने या गर्माई पाने वह बार बार तमाखू पीता रहा । आखिर विवश होकर अपने कुत्ता ‘जबरा’ को उठाकर गोद में सुलाया । इससे कुछ गर्मी मिल रही थी ।

इस बीच नील गायों ने चरकर खेत साफ कर दिया । किन्तु वह उठ न सका । दूसरे दिन वह खुश इसलिए रहा कि अगले दिन से रखवाली केलिये आने की जरूरत नहीं थी । साहूकारों से, एवं जमींदारों से पीडित किसानों की दयनीय स्थिति, आर्थिक विषमता, शोषण और उसके दुष्परिमाणों को प्रकाश में लाना ही उनका उद्देश्य है ।

AP Inter 1st Year Hindi Model Paper Set 8 with Solutions

5. निम्नलिखित दो पद्यांशों की संदर्भ सहित व्याख्या कीजिए:

1. दाने आए घर के अंदर कई दिनों के बाद ।
धुआँ उठा आँगन के ऊपर कई दिनों के बाद ||
चमक उठीं घर भर की आँखे कई दिनों के बाद ।
कौए ने खुजलाई पाँखे कई दिनों के बाद ||
उत्तर:
प्रसंग : यह पद्य को नागार्जुन द्वारा लिखे गयी अकाल और उसके बाद कविता से लिया गया है । वे प्रगतिवादी कवि थे और आधुनिक कबीर से प्रसिद्ध थे !
सन्दर्भ : अकाल समाप्त होने के बाद जिसप्रकार की रोशनी घर के चारो ओर दिखाई पडती है, उसका मार्मिक चित्रण इसमें मिलता है |
व्याख्या : अकाल के बाद अनाज घर को आया । खाना पकाने के लिए घर मे चूलहा जलाया गया । इस खुशी मे घर के सारे लोंगों के मन उत्साह से भर गया । भोजन के बाद केंके हुए अन्न से पेट भरने की आशा मे कौए भी पँस खुजलाकर इन्तजार कर रही है ।

विशेषताए :

  1. कवि की प्रगतिवादी धारणा का चित्रण हो रहा है ।
  2. उनकी भाषा सरल खडीबोली है ।

2. साथ दो बच्चे भी हैं सदा हाथ फैलाए,
बाएँ से वे मलते हुए पेट चलते है,
और दाहिना दया- दृष्टि पाने की ओर बढाए ।
भूख से सूख ओठ जब जाते,
दाता – भाग्य – विधाता से क्या पाते !
घूँट आँसूओं के पीकर रह जाते ।
चाट रहे है जूठी पत्तल कभी सड़क पर खड़े हुए
और झपट लेने को उनसे कुत्ते भी हैं अडे हु ।
उत्तर:
प्रसंग : यह पद्य निराला जी के द्वारा लिखी गयी ‘भिक्षुक’ नामक कविता से लिया गया है वे छायावादी कवि है ।
सन्दर्भ : इसमें एक भिखारी और उसके दो बच्चों की दयनीय स्थिति का वर्णन किया गया है ।
व्याख्या : एक भिक्षुक रास्ते पर आ रहा है और उसके साथ दो बच्चे भी है । वे अपने बाए हाथ से अपने भूखे पेट को मल रहे है और दाहिने हाथ से दया की भीख माँग रहे है। भूख के कारण उनके ओठ सूख गए है । अपने भाग्य के लिए वे तडप रहे है । सडक पर जूठी पत्तल चाटने के लिए भी उनके पहले ही कुत्ते उन पत्तलों को लपटने के लिए वहाँ खडे हो रहे है ।

विशेषताएँ :

  1. शोषित वर्ग के प्रति कवि की सहानुभूति व्यक्त होती है ।
  2. इसके कवि की प्रगतिवादी धारणा स्पष्ट होती है ।
  3. उनकी भाषा शुद्ध खडीबोली है ।

3. मै नहीं चाहता चिर – सुख
मै नहीं चाहता चिर – दुःख
सुख दुःख की खेल मिचौनी
खेल जीवन अपना मुख !
उत्तर:
प्रसंग : यह पद्य सुमित्रानंदन पंत के द्वारा लिखी गयी ‘सुख-दुःख’ नामक कविता से लिया गया है । वे प्रकृति का सुकुमार कवि कहे जाते है ।
सन्दर्भ : इसमें कवि सुख-दुख को समान रूप में स्वीकार करने की बात कह रहे हैं ।
व्याख्या : कवि का कहना है कि जीवन मे हमेशा सुख और हमेशा दुख का रहना भी अच्छा नहीं है । सुख और दुख आंख मिचौनी खेल की तरह हमारे जीवन में आते जाते रहना चाहिए |

विशेषताएँ :

  1. जीवन के लिए सुख और दुख होने की आवश्यकता के बारे में कवि कह रहे हैं ।
  2. उनकी भाषा खडीबोली हैं ।

4. चाह नहीं, मैं सुरबाला के गहनों में गूँथा जाऊँ,
चाह नहीं प्रेमी – माला में बिंध प्यारी को ललचाऊँ,
चाह नहीं सम्राटों के शव पर हरि डाला जाऊँ,
चाह नहीं देवों के सिर पर चढूँ, भाग्य पर इठलाऊँ,
उत्तर:
प्रसंग : यह पद्य ‘फूल की चाह’ नामक कविता से लिया गया है । यह कविता माखनलाल चतुर्वेदी के द्वारा लिखी गई है । वे भारतीय आत्मा के रूप से प्रसिद्ध है ।
सन्दर्भ : इस में कवि एक फूल की इच्छा के द्वारा अपनी भावनाओं को व्यक्त कर रहे हैं ।
व्याख्या : एक फूल के माध्यम से कवि अपनी इच्छा व्यक्त कर रहा है कि मुझे देवताओं के गहनों में गूँथ जाने की इच्छा नहीं है । अर्थात देवताओं के गले में माला बनने की इच्छा नही हैं। प्रियतम के हाथो में माला बनकर प्रेयसी को ललकारने की इच्छा नही है । सम्राटों के शवो पर पुष्प माला बनने की चाह नही है और देक्ता ईश्वर के सिर पर चढकर अपने भाग्य पर घमंड करने की इच्छा भी मूझे नही है । अर्थात यहां फूल अपने जीवन के लिए कुछ-न-कुछ विशेषता होने की मांग कर रहा है ।

विशेषताएँ :

  1. इसमें जीवन के लिए कुछ ऊँचे उमंग रखने की इच्छा फूल के द्वारा व्यक्त कर रहे है ।
  2. उनकी भाषा सरल खड़ीबोली है ।

6. निम्न लिखित किन्हीं दो गद्यांशों की संदर्भ सहित व्याख्या कीजिए :

1) ब्रह्मा ने एकाकी न होकर अपने आपको दो मे विभक्त कर लिया जिसके दक्षिण अंश को पुरुष तथा वाम को नारी की संज्ञा दी गई।
उत्तर:
प्रसंग : यह उद्धरण महादेवी वर्मा के द्वारा लिखी गयी “भारतीय संस्कृति और नारी” नामक निबन्ध से लिया गया वे छायावाद से सम्बन्धित प्रमुख साहित्यकार है ।
सन्दर्भ : सृष्टि में स्त्री और पुरुष की समानता के बारे में विवरण दिया गया है ।
व्याख्या : सृष्टि मे पुरुष तथा नारी की उत्पत्ति का एक ही केन्द्र है । वृहदारण्यक उपनिषद मे कहा गया है कि ब्रह्मा ने अपने आप को दो भागों में विभक्त कर लिया । अपने शरीर को दक्षिण अंश को पुरुष तथा वाम अंश को नारी की संज्ञा दी गयी । इसलिए दोनों की स्थिति समान कर दी । इसीकारण शिव को अर्द्ध नारीश्वर रूप कहा गया है ।

विशेषताएँ :

  1. संस्कृति के विभिन्न रूपों पर ध्यान दिया गया है ।
  2. उनकी भाषा सरल खडी बोली है ।

2) घुमक्कडी के लिए चिन्ताहीन होना आवश्यक है और चिन्ताहीन होने के लिए घुमक्कडी भी आवश्यक है ।
उत्तर:
प्रसंग : यह उद्धरण राहुल सांस्कृत्यायन के द्वारा लिखी गयी अथात घुमक्कड जिज्ञासा नामक यात्रा वृत्तांत है । वे पुरातत्व इतिहास के विशेष ज्ञाता रहे हैं और उनका यात्रा साहित्य अत्यन्त महत्वपूर्ण रहा है ।
सन्दर्भ : लेखक इसमें घुमक्कडी प्रवृत्ति को सर्वश्रेष्ठ माना है और उसकी महानता को इसमें स्पष्ट करते हैं ।
व्याख्या : घुमक्कड होना आदमी के लिए परम सौभाग्य की बात है। जो निश्चिंत होता है, वही घुमक्कड बन सकता है क्यों कि घुमक्कड के लिए चिन्ता से दूर रहना आवश्यक होता है और साथ-साथ जो चिन्ता से दूर रहता है वही घुमक्कड बन सकता अर्तात घुमक्कडी में कष्ट रहने पर भा अत्यन्त सुख मन को मिलता है ।

विशेषताएँ :

  1. घुमक्कड धर्म को अपनाने का सन्देश लेखक देते हैं।
  2. उनकी भाषा सरल खडीबोली है ।

3) प्रकृति के दोहन और शोषण के स्थान पर प्रकृति के पोषण और रक्षण का दायित्व अपनाना होगा ।
उत्तर:
प्रसंग : यह संदर्भ पर्यावरण और प्रदूषण नामक लेख से लिया गया है।
सन्दर्भ : इसमें पर्यावरण प्रदूषण से होने वाले नष्टों के बारे कहकर उसका निवारण के लिए सुझाव दे रहे हैं ।
व्याख्या : प्रकृति ही हमारे जीवन का आधार और पोषित करने वाली शक्ति है जिसके बिना पृथ्वी पर किसी भी जीव-जन्तु और मानव और पशु के जीवन की कल्पना ही संभव नही है । इसलिए पर्यावरण को सोषित करना और प्रदूषण मुक्त रखना आवश्यक है । इसलिए पर्यावरण प्रदूषण से प्रकृति की हानी को रोककर उसकी सुरक्षा करने का दायित्व हम सब पर है ।

विशेषताएँ :

  1. इसमें पर्यावरण प्रदूषण के बारे मे कहा गया है ।
  2. उनकी भाषा सरल है ।

4) सम्पूर्ण भौतिक कोश में आप अकेले जर्मन लेखक नहीं थे ।
उत्तर:
प्रसंग : यह उद्धरण ‘चन्द्रशेखर वेंकटरमन् नामक जीवनी से लिया गया वे बडे वैज्ञानिक तथा राष्ट्रीय और अन्तर्राष्ट्रीय सम्मानों से प्रतिष्ठित थे ।
सन्दर्भ : चन्द्रशेखर वेंकटरामन अनुसंदान के क्षेत्र में कितने महान थे, उसका उदाहरण सहित वर्णन किया गया है |
व्याख्या : कलकत्ते के भारत परिषद की प्रयोगशाला में ध्वनि के कम्पन और कार्यों का सिद्धांत पर अनुसंधान किया । उन्हें वाद्यों की भौतिकी का इतना गहरा ज्ञान था । जर्मनी में प्रकाशित बीस खण्डों वाले भौतिकी विश्वकोश के खण्ड के लिए उनसे वाद्ययंत्रों की भौतिकी का लेख तैयार करवाया गया | सम्पूर्ण भौतिक कोश केवल चन्द्रशेखरन जी का लेख छोडकर अन्य सभी लेख जर्मन लेखक के ही थे । इतना सम्मान चन्द्रशेखरन जी को प्राप्त हुआ ।

विशेषताएँ :

  1. वैज्ञानिक चन्द्रशेखर वेंकटरामन् की महानता का परिचय हमें मिलता है ।
  2. भाषा सरल है ।

7. एक शब्द में उत्तर लिखिए ।

1) किसका स्मरण करने से कभी भी किसी को भी दुख नही पहुँचता ?
उत्तर:
ईश्वर का

2) कौन अपना फल खुद नही खाता है ?
उत्तर:
पेड (तरुवर)

3) माखनलाल चतुर्वेदी की भाषा कैसी है ?
उत्तर:
सरल और खडीबोली

4) “उत्पीडन” शब्द का अर्थ क्या है ?
उत्तर:
पीडा (కష్టం)

AP Inter 1st Year Hindi Model Paper Set 8 with Solutions

5) भिक्षुक किसके सहारे खडे होकर भीख माँगता है ?
उत्तर:
लकडी के सहारे

8. एक शब्द में उत्तर लिखिए | ( गद्य भाग)

1) ‘शिवाजी का सच्चा स्वसूप’ किस प्रकार की एकांकी है ?
उत्तर:
ऐतिहासिक

2) भारतीय संस्कृति में किस की महानता के बारे में स्पष्ट किया गया है ?
उत्तर:
नारी की महानता

3) घुमक्कडी होना किस केलिए आवश्यक है ?
उत्तर:
चिन्ताहीन

4) मानव जीवन केलिए क्या अनिवार्य अंग बनाना होगा ?
उत्तर:
प्रकृति की रक्षा और सुरक्षा को

5) आन्ध्रप्रदेश में कौन सी प्रांत गुडियों केलिए प्रसिद्ध है ?
उत्तर:
कोंडपल्लि, एटिकोप्पाका

खण्ड – ‘ख’

9. निम्नलिखित में से कोई एक पत्र लिखिए :
छात्रवृत्ति के लिए आवेदन पत्र लिखिए
उत्तर:

मुथोल,
दिनांक : 19.6.2018.

सेवा में,
प्राचार्य महोदय,
सरकारी जूनियर कलाशाला,
मुथोल,
जिला आदिलाबाद |
आदरणीय महोदय,

आपसे सनम्र निवेदन है कि मैं आपके कॉलेज में मीडिएट प्रथम वर्ष की छात्रा हूँ | मेरा क्रमांक एम.पीयसी. वर्ग में 34 है । मैंने मुथोल सरकारी पाठशाला मे दसवीं कक्षा की परीक्षाओं में गतवर्ष सबसे प्रथम आयी हूँ | जिले स्तर पर मेरा स्थान द्वितीय है । मैं एक गरीब परिवार से हूँ और मेरे पिताजी एक किसान-मज़दूर हैं। अपनी आर्थिक स्थिति के कारण अब तक मैं फीस जमा नहीं कर पायी हूँ | अतः आपसे प्रार्थना है कि मुझे छात्रवृत्ति मंजूर करें ताकि मैं आगे की पढ़ाई कर सकूँ ।

धन्यवाद ।

अपकी विनीत छात्रा
एम. सुमलता,
इणटरमीडिएट प्रथमवर्ष,
एम.पी.सी. क्रमांक 34.

अथवा

हिन्दी की आवश्यकता के बारे में अपने मित्र के लिए एक पत्र लिखिए ।
उत्तर:

अनन्तपुरम्
दिनांक : 19-04-2018.

प्रिय मित्र सेरेश,

कैसे हो ? मैं यहाँ कुशल हूँ। हाल ही में हमारी कक्षा में सब लोग मिलकर औद्योगिक प्रशिक्षण के लिए दिल्ली गये । उस समय मुझे हिन्दी की आवश्यकता बहुत मालूम पड़ी । रेल यात्रा करते समय जैसे ही हम आन्ध्रप्रदेश छूटकर आगे बढे, सारे प्रदेशों में हिन्दी ही प्रचलित हो रही थी । दिल्ली में तो हिन्दी का ज्ञान अत्यावश्यक है । मेरी सलाह यह है कि तुम इस गरमी की छुट्टियों में हिन्दी बोलना लिखना सीखो। इससे तुम्हारा बहुत प्रयोजन होगा ।
शेष शुभ …… पत्र की प्रतीक्षा रहेगी ।

तुम्हारा प्रिय मित्र,
नागेश्वर |

पता :
नारायण सुरेश बाबू,
मकान नंबर 19-5/4,
गुणदला पहाड़ के पास,
विजयवाड़ा – 520028
कृष्णा जिला, आन्ध्रप्रदेश |

10. किन्हीं पाँच (5) शब्दों के विलोम शब्द लिखिए ।

1) नख × शिख
2) निंदक × प्रशंसक
3) फूल × काँटा
4) बलावन × निर्बल
5) रूगण × स्वस्थ
6) लम्बा × नाटा
7) शब्द × अर्थ
8) सज्जन × दुर्जन
9) सरस × नीरस
10) सर्दी × गर्मी

11. किन्हीं पाँच (5) शब्दों के समानार्थी शब्द लिखिए ।

1) औरत – स्त्री, नारी, महिला, कामिनी ।
2) कपड़ा – वस्त्र, वसन, अम्बर, चीर ।
3) कामदेव – मदन, रतिपति, अनंग, मनसिज ।
4) गणेश – गजानन, गणपति, लम्बोदर, विनायक ।
5) झंडा – पताका, ध्वज, केतु, वैजयंती |
6) दिन – दिवास, वार, वासर, अह ।
7) सिंह – शेर, वनराज, व्याघ्र, मृगराज
8) हाथी – गज, करि, द्विप, कुंजर |
9) बिजली – विद्युत, दमिनी, चपला, सौदामिनी
10) पति – स्वामी, नाथ, भर्ता, कंत

12. किन्हीं पाँच (5) शब्दों की शूद्ध वर्तनी लिखिए |

1) ग्रिह – गृह
2) घनिष्ट – घनिष्ठ
3) पड़ाई – पढ़ाई
4) प्रन – प्रण
5) पूर्व – पूर्व
6) प्रसंशा – प्रशंसा
7) विधा – विद्या
8) मर्मग्य – मर्मज्ञ
9) लच्छमण – लक्ष्मण
10) प्रशाद – प्रसाद

13. किन्हीं पाँच (5) शब्दों का अनुवाद हिन्दी में कीजिए ।

1) Admission Test – प्रवेश परीक्षा
2) Training – प्रशिक्षण
3) Parliament – संसद
4) Editor – सम्पादक
5) Technical – तकनीकी
6) Economics – अर्थशास्त्र
7) Degree College – महाविद्यालय
8) Earth – पृथ्वी
9) Fuel – ईधन
10) Typist – टंकण

AP Inter 1st Year Hindi Model Paper Set 8 with Solutions

14. कारक चिह्नों की सहायता से रिक्त स्थानों की पूर्ति कीजिए ।

1) रमेश का कुत्ता भौंक रहा है ।
2) उपवन में फूल खिला है ।
3) विद्यार्थी ने कविता सुनाई ।
4) आकाश से तारा टूटकर गिरा ।
5) मिठाई पर मक्खी बैठी है ।

15. सूचना के अनुसार वाक्य में परिवर्तन कीजिए ।

1) अपने भाई को पत्र लिखो । (रेखांकित शब्द का लिंग बदलकर लिखिए ।)
उत्तर:
अपनी बहन को पत्र लिखो ।

2) मैं ने घर खरीदा । (रेखांकित शब्द का वचन बदलकर लिखिए ।)
उत्तर:
मैं ने कई घर खरीदे ।

3) मोहन गैरजिम्मेदार लड़का है । (रेखांकित शब्द में उपसर्ग क्या है ?)
उत्तर:
‘गैर’

4) फलवाला फल बेच रहा है । (रेखांकित शब्द में प्रत्यय क्या है ?)
उत्तर:
वाला

5) चिरंजीवि पिज्जा खा रहा है । (भविष्यतकाल में लिखिए ।)
उत्तर:
चिरंजीवि पिज्जा खायेगा ।

16. सूचना के अनुसार भाषा विभाग को पहचानिए :

1) राम ने फल लाकर मेज़ पर रखा । (इस वाक्य में संज्ञा क्या है ?)
उत्तर:
राम, फल, मेज़

2) भिखारी को कुछ दे दो । (इस वाक्य में सर्वनाम क्या है ?)
उत्तर:
कुछ

3) राम भला लड़का है । (इस वाक्य में विशेषण क्या है ?)
उत्तर:
भला

4) मोहित क्रिकेट खेलता है । (इस वाक्य में क्रिया क्या है ?)
उत्तर:
खेलना

5) संचित दिन भर सोता रहता है । (इस वाक्य में क्रिया विशेषण क्या है ?)
उत्तर:
दिनभर

AP Inter 1st Year Sanskrit Model Paper Set 5 with Solutions

Self-assessment with AP Inter 1st Year Sanskrit Model Papers Set 5 allows students to take charge of their own learning.

AP Inter 1st Year Sanskrit Model Paper Set 5 with Solutions

Time : 3 Hours
Max Marks : 100

सूचना : प्रथम, द्वितीय, तृतीयप्रश्नान् अनुवादप्रश्नं च विहाय अन्ये प्रश्नाः संस्कृतभाषायामेव समाधातव्याः ।

I. एकं श्लोकं पूरयित्वा भावं लिखत । (6 × 1 = 6 M)

अ) दानं भागो ………………… तृतीयागतिर्भवति ।।
जवाब:
दानं भोगो नाशः तिस्रो गतयो भवन्ति वित्तस्य ।
यो न ददाति न भुङ्क्ते तस्य तृतीयागतिर्भवति ॥
Three are the ways to money – Charity, enjoyment and loss. The money not given in charity or spent for enjoyment will be lost.

आ) भुक्त्वा शतपथं ………………. किं प्रयोजनम् ।।
जवाब:
भुक्त्वा शतपथं गच्छेत् ताम्बूलं तदनन्तरम् |
वामपार्श्वे तु शयनम् औषधैः किं प्रयोजनम् ।।
After taking meals, one should walk for hundred steps, and after that eattamboolam. While sleeping one should lie on left side. Then where is the need for medicine?

II. एकं निबन्धप्रश्नं समाधत्त । (6 × 1 = 6M)

अ) “मायावटुः” इति पाठ्यभागस्य सारांशं संक्षेपेणलिखत ।
जवाब:
Introduction : The lesson “Mayavatu” is taken from the work Kumarasambhava written by Kalidasa. The fifth canto of the work describes the arrival of Siva in the guise of a celibate to test Parvati.

The false celibate: A celibate, shining with spiritual glow, wearing deerskin and holding a staff of Palasa entered the hermitage of Parvati like the embodiment of Brahmacharya. After receiving the hospitality of Parvati he asked her whether fuel sticks and kusa grass were easily available. He further said that physical body is the primary means of dharma. शरीरमाद्यं खलु धर्मसाधनम् ।

The celibate’s query: Then he asked her the reason for her penance saying that friendship with the good happens in seven steps or words. मनीषिभिः साप्तपदीनमुच्यते | She was born in a noble family, beautiful, wealthy and young. She could not face any humiliation in her parental house. Her father’s abode was divine. She should not search for a husband as a gem does not search, it is searched for. न रत्नमन्विष्यति मृग्यते हि तत् । He offered her half of his penance to get her desired husband.

The reason for Parvati’s penance: The friend of Parvati told the celibate that having rejected Indra and others, Parvati wanted Siva as her husband. He was not to be won over by beauty as he destroyed Manmatha. When Parvati confirmed it, the celibate said that he would not support her as he knew Siva as one indulged in inauspicious practices.

Censure of Siva by the celibate: The celibate said that it would be inappropriate to let ashes take the place of sandal paste on the bosom of Parvati. People would smile seeing her riding a bull. Siva had deformed body. His birth was unknown. His wealth was indicated by his nakedness.

Parvati’s reply: Then Parvati replied angrily that he did not know Siva properly. Stupid people despise the acts of the noble. Though penniless, Siva was the source of riches. He was the lord of the three worlds. No one knew whether his body shone with ornaments or snakes, elephant hide or fine garments, and whether he held a skull or sported the crescent moon on the crest. Indra, who rode an elephant bowed to him. How could anyone know the origin of Siva, who was considered the cause of the creator? She said that her mind was fixed on him with the feeling of love.

When Parvati was about to go from there, Siva revealed himself to her and said that he became her slave. Fruition makes exhaustion fresh again. : yafaai विधत्ते ।

आ) उपवासदीक्षिताः कपयः कथं बुभुक्षापीडिताः ?
जवाब:
Introduction: The lesson: was written by Srisailam Tatacharya, who was known as D. T. Tatacharya. He authored Kavyalankara and Mugdhanjali. In this satirical work he advises that imitating others is not good.

The greatness of the monkey race: Once the lord of the monkeys assembled all his followers and told them that previously their forefathers became great by their valour, strength and learning. Hanuman, who crossed the ocean, torched the city of Lanka and mastered the nine grammars belonged to their race only. He asked them that by what quality the humans became superior to them.

The decision of the monkeys: Then he asked them to find out why the monkeys should not have the same authority as the humans had. He told them that they should follow the customs or otherwise their animalhood would be well established.
He suggested fasting on the day after as it was the Ekadasi day. The monkeys agreed to it.

Fasting of the monkeys on the Ekadasi day: The next day the monkeys took bath in the river and started their fast. But who could control his natural impulses? The monkeys became restless in a moment. They yawned, shook their hands, stretched their legs and kissed the tips of their tails. Then one of them suggested that they should go to the Jambu tree so that the next day breaking the fast would be easier. The others agreed to it and moved to the tree. As they sat under the tree in a row, one of them suggested that they should climb the tree, and all of them did so.

The juicy Jambu fruits: The monkeys passed the minutes like aeons. As the juicy fruits fell down making sounds when the branches were moved by the wind, their mouths watered. Then one of them said that they could as well keep the ripe fruits in the bag between their chins so that there would be no need to search for fruits the next day. Agreeing to it the monkeys jumped down and started to fill their mouths with the fruits.

The poet ends the story saying that what should be done afterwards need not be told to them.

III. एकं निबन्धप्रश्नं समाधत्त । (6 × 1 = 6M)

अ) भोजस्य औदार्यम् अधिकृत्य लिखत |
जवाब:
Introduction: The lesson taken from the Bhojaprabandha written by Ballala. The poet lived in the eleventh century AD. He described the generosity of Bhoja in this lesson.

Bhoja meets Govinda : One day, while going to the pleasure garden, king Bhoja met a Brahmin. The Brahmin did not bless the king, and closed his eyes on seeing the king. When the king asked the Brahmin replied that on seeing a non-generous person in the morning, one would incur loss. Kings like Karna, Sibi and others lived even after death because of their charitable nature. One should earn fame in the perishable bodies. तस्मात् देहेष्वनित्येषु कीर्तिमेकामुपार्जयेत् ।

The king donated one lakh to that Govinda, and asked him to bring poets and scholars to the court. He became famous as a generous king. He removed his minister who told him kings became victorious by the strength of their treasury. He ordered that money and villages be granted to poets and scholars.

The Kalinga Poet : Once a poet from Kalinga came. He said that on seeing Bhoja, the enemies would drop their weapons, poets their suffering and women their saree-knot. The king gave him one lakh. The king donated five lakhs to a tribesman for his singing. The poet expressed his indignation in a verse referring to a lotus whose merits did not shine. यल्लक्ष्मीवसतेस्तव मधुपैरुपभुज्यते कोश: । The king donated him another lakh saying that the kings honoured talent only, and not lineage.

Other poets: Then five or six poets arrived there. The Kalinga poet said that it was the fault of the fragrance of the lotus that bees haunted its every petal. The king was pleased and gave another lakh to him. Later one of the poets said that it was a great lake where the filled and the unfilled pots did not collide. The king gave him one lakh.

Another poet said that the lake would satisfy the thirst of every one if only there was no crocodile inside. The king gave two lakhs to him. When Govinda expressed his dissatisfaction, the king removed him from his post. He appointed one of those poets in his place. The king told his minister that if uneducated, even a Brahmin should be sent out of the city, and if educated, even a potter should stay in the city. कुम्भकारोऽपियो विद्वान्सतिष्ठतुपुरेमम । In no time his court was filled with five hundred great poets including Vararuchi, Bana, Mayura etc.

Poet Sankara: One day a poet came to the court and read a verse in praise of the king. He said that whenever the fame of Bhoja increased as if to colour the three worlds white, he was afraid about the black hair of his wife. The king gave twelve lakhs to that poet Sankara.

आ) शबरि रामाय को कथामुक्तवती ?
The Teacher’s Query:
Once a teacher, while going to the forest along with his students asked them which water was the best in the world. The students gave different answers such as the water of the Gangas, rain water and tears. The teacher then told them a story.

Sabari’s Hospitality :
While searching for Sita in the forest, Rama and Lakshmana met Sabari. She worshipped Rama, and offered him sweet fruits. There were beautiful and fra- grant flowers in front of the place where Sabari lived. Rama asked her about those
flowers.

Matanga and his students :
Sabari told him that the hermitage of Matanga used to be there at that place. Many students came for their studies to the hermitage. Once it was the end of the summer, and monsoon was about to set in. But no arrangement to store fuel sticks was made.

Sage Matanga and his students went to the forest, and cut wood. The young and the old carried the load on their heads and returned to the hermitage. Every one was drenched in sweat. Beads of sweat dropped from the bodies on the ground.

The fragrance :
The next day everyone at the hermitage was surprised at the fragrance that spread there. The students found that the fragrance came from the direction of the forest where they went the day before. They went there and saw beautiful flowers blossomed here and there. Those flowers were not there the day before. The students ran back to Matanga, and reported to the teacher that strange inci- dent. Matanga went there, and touched those flowers affectionately.

The flowers of sweat :
Matanga told his students that the flowers were born from their sweat. Sweat born of labour was pure. The heart of Mother Earth seemed to have bloomed up on seeing their effort. He told them that there was nothing purer than physical labour. If the farmer and the weaver did not work, there would be no crop or clothes. The world moved on because of the labourers. He advised them to respect the workers. The students promised Matanga that they would respect the labourers and see that they were comfortable.

Rama and Lakshmana bowed to those flowers and went away.

The real sweat :
Having told the story the teacher explained to his students that the sweat from which any useful thing was produced was the sacred and true sweat. He advised his students to make the lives of the workers happy when they grew up.

IV. चतुर्णां प्रश्नानां समाधानानि लिखत । (4 × 2 = 8 M)

अ) चाणकयः कस्य राज्ञः अमात्यः ?
जवाब:
चाणक्यः चन्द्रगुप्तस्य राज्ञः अमात्यः । सः नन्दवंशम् उन्मूल्य चन्द्रगुप्तं सिंहासने अध्यारोपयत् ।

आ) सर्वेषां प्राणिनां प्राणधारणं कस्याधीनं वर्तते ?
जवाब:
सर्वेषां प्राणिनां प्राणधारणं अन्नाधीनं वर्तते । अन्नकारणादेव जगतः प्राणधारणं भवति ।

इ) सर्वे छात्राः बोपदेवं किमिति कथयन्ति स्म ?
जवाब:
सर्वे छात्राः बोपदेवं मूर्खशिखामणि इति कथयन्ति स्म ।

ई) कृपणस्य अग्रे गमनं दृष्ट्वा सर्वे किं कृतवन्तः ?
जवाब:
कृपणस्य अग्रे गमनं दृष्ट्वा सर्वे अट्टहासं कृतवन्तः ।

उ) युवकः वणियः गृहे केन रूपेण उद्योगं प्राप्तवान् ?
जवाब:
युवक: वणिजः गृहे भृत्यरूपेण उद्योगं प्राप्तवान् ।

ऊ) द्वितीयः राजमार्गे गच्छन् कं दृष्टवान् ?
जवाब:
द्वितीयः राजमार्गे गच्छन् एकं वृद्धं रोगीं दृष्टवान् ।

V. द्वयोः संदर्भ व्याख्यानं लिखत | (2 × 3 = 6 M)

अ) पितुस्त्वमेव मे गत्वा शीध्रमाचक्ष्व राधव |
जवाब:
परिचयः – एतत् वाक्यं दशरथस्य पश्चात्तापः इति पाठ्यभागात् स्वीकृतम् । अयं पाठ्यभागः वाल्मीकिरामायणे अयोध्याकाण्डतः स्वीकृतः ।
सन्दर्भः – तापसकुमारः दशरथम् एवम् उक्तवान् ।
भावः – राजन्, भवान् एवं शीघ्रं गत्वा मम पितरम् एनम् वृत्तान्तम् वदतु ।
विवरणम् – मृगयार्थं गतः दशरथः गजः इति मत्वा रात्रौ बाणेन तापसं हतवान् । नदीतीरे पतितः तापसः दशरथम् उवाच यत् भवान् मम पितरं मम मरणवार्तां वदतु इति ।

आ) शरीरमाद्यं खलु धर्मसाधनम् ।
जवाब:
परिचयः – एतत् वाक्यं मायावटुः इति पाठ्यभागात् स्वीकृतम् । एषः भागः कालिदासस्य कुमारसम्भवे पञ्चमसर्गात् स्वीकृतः ।
सन्दर्भः – मायावटुः पार्वतीम् एवं वदति ।
भावः – शरीरम् प्रथमं धर्मसाधनम् भवति ।
विवरणम् :- तपोवनं एकः जटिलः प्रविवेश । सः अपृच्छत् यत् पार्वती स्वशक्त्या तपसि प्रवर्तते । किं च शरीरम् प्रथमं धर्मसाधनम् । इति ।

इ) भवासि सकलभाषाजन्मदात्री यतस्त्वमृ ।
जवाब:
परिचयः – एतत् वाक्यं अमरवाणीप्रशस्तिः इति पाठ्यभागात् स्वीकृतम् । अस्य कविः श्रीमान् ई. शठकोपाचार्यः ।
सन्दर्भः कविः अमरवाणीं प्रशंसन् एवं वदति ।
भावः – त्वम् सकलभाषाणां जननी ।
विवरणम् – जगति सर्वाः भाषाः संस्कृतसंपर्केण विराजन्ते । संस्कृतभाषा सकलभाषाजननी |

ई) प्रयेण शक्ता न वयं भवेम ।
जवाब:
परिचयः – एतत् वाक्यं कपीनामुपवासः इति पाठ्यभागात् स्वीकृतम् । अस्य कविः श्रीशैलं ताताचार्यः ।
सन्दर्भः – सर्वे कपयः एवं वृक्षं प्रति गन्तुं निश्चितवन्तः ।
भावः – गन्तुं समर्थाः न भवेम ।
विवरणम् – व्रतोपवासेन पीडिताङ्गाः वयं प्रातः वृक्षं प्रति गन्तुं न समर्थाः भवेम इति कपयः चिन्तितवन्तः ।

VI. द्वयोः संदर्भ व्याख्यानं लिखत । (2 × 3 = 6 M)

अ) मूढो हि मदनेन आयास्यते ।
जवाब:
परिचयः एतत् वाक्यम् ‘कपिञ्जलोपदशः इति पाठ्यभागात् स्वीकृतम् । अस्य मूलग्रन्थः कादम्बरी | अस्य कविः बाणः ।
सन्दर्भः – महाश्वेतायाम् अनुरक्तम् पुण्डरीकम् सन्मार्गे प्रवर्तयितुं कपिञ्जलः एवम् उपदिशति । अर्थ:- मूर्खः एव मदनस्य वशे पतति ।
विवरणम् – कपिञ्जलः पुण्डरीकं वदति यत् पुण्डरीकस्य स्थितिः तस्य अनुरूपा न । तेन यत् आरब्धं तत् किम् ? तपः वा, व्रतं वा, नियमः वा ? मूर्खजनः एव मदनस्य वशे पतति ।

आ) यल्लक्ष्मीवसतेस्तव मधुपैरुपभुज्यते ।
जवाब:
परिचयः एतत् वाक्यं भोजस्य औदार्यम् इति पाठ्यभागात् स्वीकृतम् । अस्य कविः बल्लालः ।
सन्दर्भः – कलिङ्गकविः पद्ममिषेण एवम् अवदत् ।
भावः – लक्ष्मीनिवासम् अपि तव कोशम् भ्रमराः आस्वादयन्ति ।
विवरणम् – पुलिन्द्पुत्राय भोजः पञ्चलक्षं ददौ । ततः क्लिन्नः कविः पद्ममिषेण अवदत् – हे पद्म, व गुणाः न प्रकाशन्ते । यत्र लक्ष्मीः वसति तत्कोशः मधुपैः भुज्यते ।

इ) न हि परिश्रमात् ऋते पवित्रम् इह किमपि विद्यते ।
जवाब:
परिचयः – एतत् वाक्यं स्वेदस्य पुष्पाणि इति पाठ्यभागात् स्वीकृतम् । अस्य रचयित्री डा. माधवी जोषी ।
सन्दर्भः – मातङ्गः छात्रान् एवम् उपदिष्टवान् ।
भावः – परिश्रमं विना अन्यत् पवित्रम् नास्ति ।
विवरणम् – श्रमजातः स्वेदः पवित्रः इति मातङ्गः उपदिष्टवान् । परिश्रमः एव पवित्रः । श्रमस्य स्वेदः न भवति चेत् जगत् न प्रचलति । इति उक्तवान् ।

ई) स्वच्छतायै प्रयतामहे ।
जवाब:
परिचयः – एतत् वाक्यम् वर्षापरिदेवनम् इति पाठ्यभागात् स्वीकृतम् । अस्य रचयिता आचार्य गुल्लपल्लि श्रीरामकृष्णमूर्तिः ।
सन्दर्भः – स्वच्छतापरिरक्षणार्थं प्रयत्नं करणीयमिति वदन्ती वर्षादेवी एवं प्रबोधयति ।
भावः – स्वच्छतार्थं प्रयत्नं कुर्मः ।
विवरणम् – विश्वं कलुषितम् इति दुःखित्वा वर्षादेवी स्वच्छतापरिरक्षणार्थं जनान् प्रबोधयति । स्वच्छतार्थं प्रयत्नं कुर्मः । यः क्रियावान् स पण्डितः । इति ।

VII. त्रयाणां प्रश्नानां समाधानानि लिखत । (3 × 2 = 6 M)

अ) मुनिः दशरथं किमिति शशाप ?
जवाब:
मुनिः दशरथं राजन्, पुत्रशोकेन त्वं कालं करिष्यसि इति शशाप |

आ) धर्मसाधनेषु आद्यं किमृ ?
जवाब:
धर्मसाधनेषु आद्यं शरीरम् ।

इ) सततं भूषणं किमृ ?
जवाब:
क्षीयन्ते अखिलभूषणानि । वाग्भूषणं सततं भूषणम् ।

ई) किम् अश्वमेधेन समं विदुः ?
जवाब:
दरिद्राय कृतं दानं शुन्यलिङ्गपूजनम्, अनाथप्रेतदहनं च अश्वमेधसमं विदुः ।

उ) कविभिः कीदृशी आलङ्कारशोभा आनीता ?
जवाब:
कालिदासादिकविभिः अनुपमा अलङ्कारशोभा आनीता ।

ऊ) उपवासे कपिः किं कृतवात् ?
जवाब:
उपवासे कपिः जम्बूफलानि खादितवान् ।

VIII. त्रयाणां प्रश्नानां समाधानानि लिखत । (3 × 2 = 6 M)

अ) पुण्डरीकः किमर्थं विचारग्रस्थः बभूव ?
जवाब:
पुण्डरीकः महाश्वेतायाम् अनुरक्तः अभवत् । तपोजपादिनियमेषु अनासक्तः अभवत् । ताम् एव चिन्तयन् विचारग्रस्तः बभूव ।

आ) विद्वत्प्रियः भोजः किमिति प्रथामगात् ?
जवाब:
कविभ्यः दानं कुर्वन् भोजः ‘विद्वत्प्रियः’ इति प्रथाम् अगात् ।

इ) कयोः मध्ये विवादः वर्तते ?
जवाब:
शशकपिञ्जलयोः मध्ये विवादः वर्तते ।

ई) कः स्वेदः पवित्रः ?
जवाब:
श्रमजातः स्वेदः पवित्रः । कर्मणां स्वेदः पवित्रः ।

उ) पत्रगतसन्देशः कः ?
जवाब:
यावत् पर्यन्तं गृहे भवन्तः अतिथिसत्कारं करिष्यन्ति तावत् पर्यन्तं भोजनार्थं मम श्रमः न भविष्यति | अहं दैवकार्यार्थं गमिष्यामि । मां मा अन्विष्यन्तु ।

ऊ) चत्वारः पुरुषाः के ?
जवाब:
सर्वः, कश्चित्, यः कोऽपि, न कोऽपि इति चत्वारः पुरुषाः ।

IX. पुत्रः पितरं प्रति पत्रं लिखत । (1 × 5 = 5 M)
जवाब:

अनन्तपुरम्
दिनाङ्कः 03-11-2018

सविधे –
श्री. एन्. रविकान्तः
डो.नं. 3-2-11,
गान्धीवीथी,
कडप
परमपूज्यानां पितृपादानां प्रणामाः । अहम् अत्र कुशली । भवन्तः सर्वे कुशलिनः इति मन्ये । अत्र म़म पठनं सम्यक् चलति । आगामिमासे अध्यापकैः सह मम कलाशालातः छात्राः उज्जयिनीनगरं प्रति विज्ञानयात्रायै गमिष्यन्ति । मम कृते अनुमत्या सह व्ययार्थं सहस्त्र रूप्यकाणि प्रेषयन्तु इति प्रार्थयामि । मातृचरणयोः मम वन्दनानि सूचयन्तु ।

इति
भवदीयपुत्रः / पुत्रिका

(अथवा)

पुस्तकप्रेषणविषये पत्रं लिखत ।

जवाब:

कडप
दिनाङ्कः 20-01-2018

सविधे –
मान्यसञ्चालकाः,
सरस्वतीविद्याप्रकाशन्,
विजयवाटिकॉ
मान्याः !

भवद्भिः प्रकाशितेषु अधोनिर्दिष्टानि पुस्तकानि मया यथानिर्दिष्टम् अपेक्ष्यन्ते ।

क्र.सं पुस्तकनाम कविः प्रतय:
1 कुमारसम्भवम् कालिदासः 5
2 श्रीमद्रामायणम् वाल्मीकिः 4
3 पञ्चतन्त्रम् विष्णुशर्मा 6
4 भर्तृहरिसुभाषितानि भर्तृहरिः 3

कृपया एतानि पुस्तकानि वि.पि. पि. द्वारा अधोसूचितं सङ्केतं प्रति प्रेषयन्तु ।
सधन्यवादम् ।

भवदीयः । भवदीया
नाम : ……………..
गृहसंख्या 2-11-18
गान्धीरोड्, कडप |

X. द्वयोः शब्दयोः सविभक्तिकरूपाणि लिखत । (2 × 6 = 12 M)

अ) शिव
जवाब:
AP Inter 1st Year Sanskrit Model Paper Set 5 with Solutions 1

आ) गीता
जवाब:
AP Inter 1st Year Sanskrit Model Paper Set 5 with Solutions 2

इ) वन
जवाब:
AP Inter 1st Year Sanskrit Model Paper Set 5 with Solutions 3

ई) तद् (पुं.)
जवाब:
AP Inter 1st Year Sanskrit Model Paper Set 5 with Solutions 4

XI. द्वयोः धातुरूपाणि लिखत । (2 × 3 = 6M)

अ) धावतु
जवाब:
AP Inter 1st Year Sanskrit Model Paper Set 5 with Solutions 5

आ) अचलत्
जवाब:
AP Inter 1st Year Sanskrit Model Paper Set 5 with Solutions 6

इ) अवन्दत
जवाब:
AP Inter 1st Year Sanskrit Model Paper Set 5 with Solutions 7

ई) वन्दिष्यते
जवाब:
AP Inter 1st Year Sanskrit Model Paper Set 5 with Solutions 8

XII. त्रीणि सन्धिनामनिर्देशपूर्वकं विघटयत । (3 × 2 = 6M)

अ) वधूहः
जवाब:
वधू + ऊहः = सवर्णदीर्घ सन्धिः

आ) मङर्षिः
जवाब:
महा + ऋषिः = गुणसन्धिः

इ) तेवैश्वर्यम्
जवाब:
देव + ऐश्वर्यम = वृद्धि सन्धिः

ई) धात्रंशः
जवाब:
धातु + अंश यणादेश सन्धिः

उ) पावकः
जवाब:
पौ + अंकः = अयवायाव सन्धिः

ऊ) सोऽपि
जवाब:
सः + अपि = पूर्वरुप सन्धिः

XIII. त्रीणि सन्धिनामनिर्देशपूर्वकं सन्धत्त । (3 × 2 = 6M)

अ) पितृ + ऋणमर्
जवाब:
पितृणम् = सवर्णदीर्घ सन्धिः

आ) गण + ईश:
जवाब:
गणेशः = गुण सन्धिः

परम + ईशः
जवाब:
परमौषधिः = वृद्धि सन्धिः

ई) साधु + इति
जवाब:
साध्विति = यणादेश सन्धिः

उ) गै + अकः
जवाब:
गायकः = अयवायाव सन्धिः

ऊ) कार्ये + अपि
जवाब:
कार्येऽपि = विसर्ग सन्धिः

XIV. आन्ध्रभाषायां वा आङ्ग्लभाषायां वा अनुवदत । (5 × 1 = 5M)

अ) त्वं जलं पिबसि ।
जवाब:
Your drink water.

आ) सा क्षीरम् अपिबन् ।
जवाब:
She drank milk.

इ) त्वं कुत्र गमिष्यसि ।
जवाब:
Where will you go ?

ई) वाणी सत्यं वतिष्यति ।
जवाब:
Vani will tell the troth.

उ) स्वदेशे पूज्यते राजा विव्दान् सर्वत्र पूज्यते ।
जवाब:
A Scholar is honoured every where.

XV. एकेन पदेन समाधत्त । (5 × 1 = 5M)

अ) अयोध्यानगरस्य राया कः ?
जवाब:
दशरथः ।

आ) पार्वत्याः तपोवतं कः प्रविवेश ?
जवाब:
जटिलः ।

इ) धनस्य तृतीया गतिः का ?
जवाब:
नाश: ।

ई) श्रेष्ठं धनं किम् ?
जवाब:
विद्याधनं ।

उ) अमरवाणी प्रशस्तिः केन विरचिता ?
जवाब:
ई. शठगोपाचार्येण ।

XVI. एकेन पदेन समाधत्त । (5 × 1 = 5M)

अ) धैर्यधनाः के ?
जवाब:
साधवः ।

आ) भोजस्य मुरव्यामात्यः कः ?
जवाब:
बुध्दिसागरः ।

इ) पञ्चतन्त्रं केन विरचितम् ?
जवाब:
विष्णुशर्मणा ।

ई) तुणवत् कान् न गणयेम ?
जवाब:
श्रामिकान् ।

उ) अधिकारिणः कानि प्रदाय बोधयन्ति ?
जवाब:
करपत्राणि ।

AP Inter 1st Year Zoology Question Paper March 2015

Varied difficulty levels in AP Inter 1st Year Zoology Model Papers and AP Inter 1st Year Zoology Question Paper March 2015 cater to students with diverse academic strengths and challenges.

AP Inter 1st Year Zoology Question Paper March 2015

Time : 3 Hours
Max. Marks: 60

General Instructions:
Note : Read the following instructions carefully.

  1. Answer All questions of Section A. Answer ANY SIX questions in Section B and answer ANY TWO questions in Section C.
  2. In Section A questions from Sr. Nos. 1 to 10 are of “Very Short Answer Type”. Each question carries TWO marks. Every answer may be limited to 5 lines. Answer all these questions at one place in the same order.
  3. In Section ‘B’, questions from Sr. Nos. 11 to 18 are of “Short Answer Type”. Each question carries FOUR marks. Every answer may be limited to 20 lines.
  4. InSection ‘C’, questions from Sr. Nos. 19 to 21 are of “Long Answer Type”. Each question carries EIGHT marks. Every answer may be limited to 60 lines.
  5. Draw labelled diagrams wherever necessary in Sections ‘B’ and ‘C’.

Section – A (10 × 2 = 20)

Note : Answer all the questions in 5 lines each.

Question 1.
What is trinominal nomenclature ? Give an example.
Answer:
Trinomial nomenclature is the extension of the binominal system of nomenclature. This system permits the designation of subspecies with a three worded name called trinomen. Subspecies is category below the level of species. Eg: Corvus splendens splendens.

Question 2.
Mention the animals that exhibited a “tube – within – a- tube” organisation for the first time. Name their body cavity.
Answer:
A tube – within – a tube organization for the first time is seen in the group Nematoda. Their body cavity is named as pseudo coelom.

Question 3.
Define ‘Osteon’.
Answer:
In a dense bone a Haversian canal and the surrounding lamillae (rows of osteocytes) and lacunae are collectively called a Haversian system or Osteon.

AP Inter 1st Year Zoology Question Paper March 2015

Question 4.
Distinguish between a tendon and a ligament.
Answer:

  1. Tendons attach the skeletal muscles to bones.
  2. Ligaments attach bones to other bones.

Question 5.
Which arthropod, you have studied, is called a “living fossil” ? Name its respiratory organs.
Answer:
Limulus (king crab) belonging to class Xiphosura of sub phylum chelicerata is called living fossil. It’s respiratory organs are book gills.

Question 6.
Distinguish between Proter and Opisthe.
Answer:
During Transverse binary fission of paramoecium, two daughter paramoecia are formed. The upper or anterior one is proter which receive upper contractile vacuole, cytopharynx and cytostome of its parent. The lower or posterior daughter is opisthe which receives the posterior contractile vacuole only.

Question 7.
What is meant by nocturnal periodicity with reference to the life history of a nematode parasite you have studied ?
Answer:
The microfilaria larvae of wuchereria bancrofti in man move in the peripheral blood circulation during the night time between 10 pm and 4 am. This tendency is referred to as nocturnal periodicity. Ex: Culex.

Question 8.
Name three meninges. In which group of animals do you find all of them ?
Answer:
The three meninges which are brain layers are a) duramater b) Arachnoid membrane c) Piamater.
The group mammals possess all the three layers.

Question 9.
Distinguish between lobopodium and filopodium. Give an example to each of them.
Answer:
a) Blunt finger like pseudopodia are called lobopodium and are seen in Amoeba and Entamoeba.
b) Fibre like pseudopodia are called filopodium and are seen in Euglypha.

AP Inter 1st Year Zoology Question Paper March 2015

Question 10.
Define Mutualism. Give an example.
Answer:
This type of interaction benefits both the interacting species. Example : Lichens represent an intimate mutualistic relationship between fungus and photosynthetic algae.

Section – B (6 × 4 = 24)

Note : Answer any six questions in 20 lines each.

Question 11.
What is ‘Evil Quartet’ ?
Answer:
The following are the four major causes for accelerated rates of species extinction in the world. These causes are referred to as evil quartet.
a) Habitat loss and fragmentation : These are most important reasons for the loss of biodiversity.
b) Over exploitation : When need turns to greed, it leads to over exploitation.
c) Invasion of Alien species : When Alien species are introduced into a habitat, they turn invasive and establish themselves at the cost of indigenous species.
d) Co – extinctions : In an obligate association between a plant and an animal, if a plant becomes extinct, the animal also becomes extinct as seen in a parasitic and host association.

Question 12.
Give an account of Glandular Epithelium.
Answer:
Some of the columnar or cuboidal cells that get specialised for the production of certain secretions, form glandular epithelium. The glands are of two types – unicellular glands consisting of isolated glandular cells such as goblet cells of the gut, and multicellular glands consisting of clusters of cells such as salivary glands.
AP Inter 1st Year Zoology Question Paper March 2015 1
On the basis of the mode of pouring of their secretions, glands are divided into two types namely exocrine and endocrine glands. Exocrine glands are provided with ducts; secrete mucus, saliva, earwax (cerumen), oil, milk, digestive enzymes and other cell products. In contrast, endocrine glands are ductless and their products are ‘hormones’, which are not sent out via ducts, but are carried to the target organs by blood. Based on the mode of secretion, exocrine glands are further divided into i. Merocrine glands (e.g. pancreas) which release the secretory granules without the loss of other cellular material ii. Apocrine glands (e.g. mammary glands) in which the apical part of the cell is pinched off along with the secretory product and iii. Holocrine glands (e.g. sebaceous glands), in which the entire cell disintegrates to discharge the contents.

Question 13.
Mention the salient features of Holothuroidea.
Answer:
Holothuroidea-salient features:

  1. This class includes sea cucumbers.
  2. Body is elongated in the oro-aboral axis.
  3. Skin is leathery (coriaceous) and dermis contains loose spicules.
  4. Arms, spines and pedicellariae are absent.
  5. Mouth is surrounded by retractile tentacles (modified tube feet useful for feeding).
  6. Ambulacral grooves are ‘closed’; tube feet bear suckers.
  7. Madreporite is internal (occurs in coelom).
  8. Respiratory organs are a pair of cloacal ‘respiratory trees’.
  9. Development is indirect and includes auricularia larva. Examples : Holothuria, Synapta, Thyone.

Question 14.
What are the modifications that are observed in birds that help them in flight ?
Answer:
1) Body is streamlined.

2) The fore limbs are modified into wings for flight.

3) Skin is dry and devoid of glands, except the oil or preen gland or uropygial gland at the base of the tail to protect tail feathers.

4) Exoskeleton consists of epidermal feathers (a unique feature), with interlocking system.

5) Long bones are hollow with air cavities (pneumatic). Skull is monocondylic. Vertebrae are heterocoelous. The last thoracic, lumbar, sacral and anterior few caudal vertebrae are fused to form a synsacrum. it is fused with pelvic girdle to provide support to hind limbs. A few posterior most caudal vertebrae are fused to form the pygostyle that provides support to the tail feathers. Sternum has a keel/carina for the attachment of flight muscles (except in the ratite birds). Both the clavicles are fused with the interclavicle to form a ‘V’ – shaped bone, called furcula or ‘Wish bone’ or ‘Merry thought bone’.
AP Inter 1st Year Zoology Question Paper March 2015 2
6) All modern flying birds are provided with powerful breast muscles (flight muscles), chiefly the pectoralis major and pectoralis minor.

AP Inter 1st Year Zoology Question Paper March 2015

Question 15.
What are lateral appendages ? Based on their presence and absence, write the various types of flagella giving at least one example for each type.
Answer:
Lateral appendages : Some flagella bear one or two or many rows of short, lateral hair like fibrils called lateral appendages. They are of two types namely ‘mastigonemes’ and ‘flimmers’.

Types of Flagella : Based on the presence or absence and / or the number of rows of lateral appendages, five types of flagella are recognised.

a. Stichonematic : This flagellum bears one row of lateral appendages on the axoneme. Eg : Euglena and Astasia.

b. Pantonematic : This flagellum has two or more rows of lateral appendages on the axoneme. Eg : Peranema and Monas.

c. Acronematic : This type of flagellum does not bear lateral appendages and the terminal part of the axoneme is naked without the outer sheath at its tip.
Eg : Chlamydomonas and Polytoma.
AP Inter 1st Year Zoology Question Paper March 2015 3
d. Pantacronematic : This type of flagellum is provided with two or more rows of lateral appendages and the axoneme ends in a terminal naked filament. Eg. Urceolus.

e. Anematic or simple : In this type of flagellum, lateral appendages and terminal filament are absent. Hence, it is called anematic (a – no; nematic – threads).
Eg : Chilomonas and Cryptomonas.

Question 16.
Why is adolescence is considered vulnerable phase ?
Answer:
Adolescence : It is the time period between the beginning of puberty and the beginning of adulthood. In other words, it is the bridge linking childhood and adulthood. The age between 12 – 18 years is considered adolescence period. It is both ‘a period and a process’ during which a child becomes mature. It is accompanied by several biological and behavioural changes. Thus, adolescence is a very vulnerable phase of mental and psychological development of an individual.

Question 17.
Draw a neat and labelled diagram of Ommatidium.
Answer:
AP Inter 1st Year Zoology Question Paper March 2015 4

Question 18.
Discuss the causes and effects of “Global Warming”. What measures need to be taken to control “Global Warming” ?
Answer:
Global warming is causing climate changes and is “ilso responsible for the melting of polar ice caps and other snow caps of mountains such as the Himalayas. Over many years, this will result in a rise in sea levels all over the world that can submerge many coastal areas. The total spectrum of changes that global warming can bring about is a subject that is still under active research.

Global warming – Control measures :

  1. The measures include cutting down use of fossil fuels
  2. Improving efficiency of energy usage.
  3. Planting of trees, and avoiding deforestation
  4. Slowing down the growth of human population.

AP Inter 1st Year Zoology Question Paper March 2015

Section – C (2 × 8 = 16)

Note : Answer any two questions in 60 lines each.

Question 19.
Explain the structure and life cycle of Entamoeba histolytica with the help of neat and labelled diagram.
Answer:
Structure : Entamoeba histolytica passes through three distinct stages in its life cycle, namely :
i) Trophozoite stage
ii) Precystic stage and
iii) Cystic stage

Trophozoite stage : It is the most active, motile, feeding and pathogenic stage that lives in the mucosa and sub-mucosa membranes of the large intestine. It moves with the help of a single blunt finger like pseudopodium called lobopodlum which is produced anteriorly. The body of the trophozoite is surrounded by plasmalemma. Its cytoplasm is differentiated into an outer clear, viscous, non-granular ectoplasm and the inner fluid like, granular endoplasm. Ribosomes, food vacuoles and a vesicular, cartwheel shaped nucleus are present in the endoplasm. However contractile vacuoles, endoplasmic reticulum, Golgi apparatus and mitochondria are absent.

The absence of mitochondria indicates the ‘obligate anaerobic nature’ of Entamoeba histolytica. It produces the proteolytic enzyme called histolysin due to which the species name histolytica’ was assigned to it. Due to the effect of this enzyme, the mucosa and sub-mucosa of the gut wall are dissolved releasing some amount of blood, tissue debris which are ingested by the trophozoites. Hence, the food vacuoles are with erythrocytes, fragments of epithelial cells and bacteria. The mode of nutrition is holozoic. Presence of ‘RBC in food vacuoles’ and cartwheel shaped nucleus are the characteristic features of the trophozoites of Entamoeba histolytica.
AP Inter 1st Year Zoology Question Paper March 2015 5

Precystic stage : It is the non – feeding and non-pathogenic stage of Entamoeba histolytica that is found in the lumen of the large intestine. It is a small, spherical or oval, non- motile form. The cytoplasm of the precystic stage stores glycogen granules and chromatoid bars (made of ribonucleo protein) which act as reserve food.
AP Inter 1st Year Zoology Question Paper March 2015 6

Cystic stage : It is round in shape and is surrounded by a thin, delicate and highly resistant cyst wall. It is found in the lumen of the large intestine. The process of development of cyst wall is called encystation which is a means to tide over the unfavourable conditions that the parasite is going to encounter while passing to a new host. Soon after the encystation, the nucleus undergoes two successive mitotic divisions to form four daughter nuclei. This type of cystic stage is called tetra nucleate cyst or mature cyst which is ‘the stage infective to man’.
AP Inter 1st Year Zoology Question Paper March 2015 7

Life cycle : The trophozoites undergo binary fissions in the wall of the large intestine and produce a number of daughter entamoebae. They feed upon the bacteria and the host’s tissue elements, grow in size and again multiply. After repeated binary fissions, when the trophozoites increase in number, some of the young ones enter the lumen of the large intestine and transform into precystic stages. Here, the precystic stages transform into cystic stages which in turn develop into the tetranucleate cysts.

The entire process is compelted only in a few hours. These tetranucleate cysts come out along with the faecal matter and can remain alive for about 10 days. These cysts reach new host through contaminated food and water. They pass into the small intestine of a new human host where the cyst wall gets ruptured by the action of the enzyme trypsin, releasing the tetranucleate amoebae. Such tetranucleate excystic amoebae are called metacysts.

The four nuclei of the metacyst undergo mitotic divisions and produce eight nuclei. Each nucleus gets a bit of the cytoplasm and thus eight daughter entamoebae or ‘metacystic trophozoites’ are produced. These young ones develop into feeding stages called trophozoites. They invade the mucous membrane of the large intestine and grow into mature trophozoites.

Pathogenicity :
The trophozoites ‘dissolve’ the mucosa! lining by histolysin, go deep into sub-mucosa and cause ulcers. These ulcers contain cellular debris, lymphocytes, blood corpuscles and bacteria. It leads to the formation of abscesses in the wall of large intestine . Ultimately it results in stool with biood and mucous. This condition is called amoebic dysentery or intestinal amoebiasis or tropical amoebiasis. Some people do not exhibit any symptoms. Such people are called ‘carriers or asymptomatic cyst passers’ as their stool contains the tetranucleate cysts. They help in spreading the parasites to other persons.

Extra-intestinal amoebiasis : Sometimes, the trophozoites may “upture the wall of capillaries, enter the blood stream, and primarily reach the liver where they may cause ‘abscesses’ (some call it ‘secondary amoebiasis’). From there, they may go to lungs, heart, brain, kidneys, gonads, etc., cause abscesses in those parts leading to severe pathological conditions.

AP Inter 1st Year Zoology Question Paper March 2015

Question 20.
Describe the Blood circulatory system of Periplaneta in detail and draw a neat and labelled diagram of it.
Answer:
Circulatory system of Periplaneta : The circulatory system helps in the transportation of digested food, hormones etc., from one part to another in the body. Periplaneta has an open type of circulatory system as the blood, or haemohymph, flows freely within the body cavity or haemocoel. Blood vessels are poorly developed and open into spaces. Visceral organs located in the haemocoel are bathed in the blood. The three main parts associ-ated with the blood circulatory system of Periplaneta are the haemocoel, heart, and blood.
AP Inter 1st Year Zoology Question Paper March 2015 8
Haemocoel : The haemocoel of cockroach is divided into three sinuses by two muscular, horizontal membranes, called dorsal diaphragm or ‘pericardial septum’ and ventral diaphragm. Both the diaphragms have pores. There is a series of paired triangular muscles, called alary muscles. Every segment has one pair of these muscles situated on the lateral sides of the body. These are attached to the pericardial septum by their broad bases and to the terga by their pointed ends or apices. The three sinuses of the haemocoel are known as pericardial haemocoel or the ‘dorsal sinus’, the perivisceral haemocoel or the ‘middle sinus’ and eternal haemocoel or ‘vental sinus’ or ‘perineural sinus’. The middle sinus is very large as it contains most of the viscera. The dorsal and ventral sinuses are small as they have only heart and nerve cord, respectively.

Heart : the heart lies in the pericardial haemocoel or dorsal sinus. It is a long, muscular, contractile tube found along the mid dorsal line, beneath the terga of the thorax and abdomen. It consists of 13 chambers. Every chamber opens into the other present in front of it. Three of the thirteen chambers are situated in the thorax and ten in the abdomen. Its posterior end is closed while the anterior end is continued forward as the anterior aorta. At the posterior side of each chamber, except the last, there is a pair of small apertures called ostia’ one on each side. Ostia have valves which allow the blood to pass only into the heart from the dorsal sinus.

Blood : The blood of Periplaneta is colourless and is called haemolymph. it consists of a fluid called plasma, and free blood corpuscles or haemocytes, which are ‘phagocytic’. The phagocytes are large in size and can ‘ingest’ foreign particles such as bacteria. There is no respiratory pigment in the blood and so it plays no major role in respiration. The important functions of the blood are :

  1. It absorbs digested food from the alimentary canal and distributes it to the rest of the body.
  2. It brings nitrogenous wastes from all parts of the body to the excretory organs for their elimination.
  3. It carries defensive phagocytes to the places of infection where they engulf the germs and disintegrating tissue parts.
  4. It transports secretions of the ductless glands to the target organs.

Circulation of blood: The blood flows forward in the heart by the contractions of its chambers. At the anterior end of the heart, the blood flows into the aorta and from there it enters the sinus of the head. From the head sinus, the blood flows into the perivisceral and sternal sinuses. On contraction of the alary muscles the pericardial septum is pulled down. This increases the volume of the pericardial sinus. Hence blood flows from the perivisceral sinus into the pericardial sinus through the appertures of the pericardial septum. On relaxation of the alary muscles, the pericardial septum moves upwards to its original position. This forces the blood, to enter the chambers of the heart through the ostia from the pericardial sinus.
AP Inter 1st Year Zoology Question Paper March 2015 9

AP Inter 1st Year Zoology Question Paper March 2015

Question 21.
Give an account of flow of energy in an ecosystem.
Answer:
Energy Flow : Except for the deep sea hydro-thermal ecosystem, sun is the only source of energy for all ecosystems on Earth. Of the incident solar radiation less than 50 per cent of it is photosyn- thetically active radiation (PAR). We know that plants and photosynthetic bacteria (autotrophs), fix Sun’s radiant energy to synthesise food from simple inorganic materials. Plants capture only 2 – 10 percent of the PAR and this small amount of energy sustains the entire living world.

So, it is very important to know how the solar energy captured by plants flows through different organisms of an ecosystem. All heterotrophs are dependent on the producers for their food, either directly or indirectly. The law of conservation of energy is the first law of thermodynamics. It states that energy may transform from one form into another form, but it is neither created nor destroyed. The energy that reaches earth is balanced by the energy that leaves the surface of the earth as invisible heart radiation.

The energy transfers in an ecosystem are essential for sustaining life. Without energy transfers there could be no life and ecosystems. Living beings are the natural proliferations that depend on the continuous inflow of concentrated energy.

Further, ecosystems are not exempted from the Second Law of thermodynamics. It states that no process involving energy transformation will spontaneously occur unless there is degradation of energy. As per the second law of thermodynamics – the energy dispersed is in the form of unavailable heat energy, and constitutes the entropy (energy lost or not available for work in a system). The organisms need a constant supply of energy to synthesize the molecules they require.

The transfer of energy through a food chain is known as energy flow. A constant input of mostly solar energy is the basic requirement for any ecosystem to function. The important point to note is that the amount of energy available decreases at successive trophic levels. When an organism dies, it is converted to detritus or dead biomass that serves as a source of energy for the decomposers. Organisms at each trophic level depend on those at the lower trophic level, for their energy demands.

Each trophic level has a certain mass of living material at a particular time, and it is called the standing crop. The standing crop is measured as the mass of living organisms (biomass) or the number of organisms per unit area. The biomass of a species is expressed in terms of fresh or dry weight (dry weight is more accurate because water contains no usable energy).

The 10 percent Law: The 10 percent law for the transfer of energy from one trophic level to the next was introduced by Lindeman (the Founder of the modern Ecosystem Ecology). According to this law, during the transfer of energy from one trophic level to the next, only about 10 percent of the energy is stored / converted as body mass / biomass. The remaining is lost during the transfer or broken down in catabolic activities (Respiration).
AP Inter 1st Year Zoology Question Paper March 2015 10
Lindeman’s rule of trophic efficiency /Gross ecological efficiency is one of the earliest and most widely used measures of ecological efficiency. For example, if the NPP (Net primary production) in a plant is 100 kJ, the organic substance converted into body mass of the herbivores which feeds on it is 10 kJ only. Similarly the body mass of the carnivore -I is 1 kJ only.

TS Inter 1st Year Political Science Model Paper Set 2 with Solutions

Thoroughly analyzing TS Inter 1st Year Political Science Model Papers Set 2 with Solutions helps students identify their strengths and weaknesses.

TS Inter 1st Year Political Science Model Paper Set 2 with Solutions

Time: 3 Hours
Max. Marks: 100

Section – A
3 x 10 = 30 Marks

Note: Answer any THREE of the following questions In not exceeding 40 lines each. Each question carries 10 Marks.

Question 1.
Write about the importance of Political Science.
Answer:
Introduction: Politiçal Science is a premièr social science. It is mainly concerned with the study of the state in its relation with Society, Citizens, Associations and the world at large. Arisot1e is regarded as the Father of olitica1 Science. He wrote famous book “THE POLITICS”.

Meaning: The word politics is derived from the ancient Greek word “POLIS” meaning city. State and polity from ‘Poletieia’ meaning government of constitution. Politics came to mean the study of state and government and the Institutions of the state.

Definitions: Political Scientists gave various definitions on Political Science. They are as follows:
1. J.W. GARNER: “Political Science begins and Ends with the State”.
2. R.G. GETTLE: “Political Science is” The historical investigation of the state in the past, an analytical study of the state of present and what the state ought to be in the future”.
3. ROBERT DAHL: “Political Science is the Scientific study of importance of power, Authority and influence.”

The political science helps to bring out certain changes in the political system and also suggests solutions for the political problems. It also helps to promote good citizenship and formation of responsible government susceptible to public opinion.

1. Political Science explains concepts and theories: Political Science studies the relations between the individual, society, and state. It helps to protect the liberty and freedom of the individuals. The study of Political Science aids to know the political theories, concepts, and ethical principles useful for the creation of a progressive society.

2. Political Science examines forms and organs of the government: The study of Political Science enhances the knowledge of the systems of government. It details about monarchy, aristocracy democracy, dictatorship and other forms of government. It speaks about the organs of the government like legislature, executive and judiciary, their functions, and inter-relations.

3. Political Science enlightens on rights and duties: Fundamental rights are essential for the people to lead a good life in a political society. The observations made in Political Science help to enlighten the citizens about their respective rights and duties all their pros and cons to contribute for a good citizenship.

4. Political Science provides knowledge of the political thinkers: The study of Political Science provides knowledge of the political thinkers and theories which had influenced the world in different times. There were many political theories, which became popular in different times. The philosophers like Rousseau and Voltaire laid the foundations for French Revolution through their works. In the same way, the writings of Karl Marx led to a revolution in Russia and the ideas of Mao led to a revolutionary trend in China. Mahatma Gandhi provided constructive political leadership to Indian freedom struggle. The study of political science educates about political thinkers.

5. Political science deals with International Relations: The significance of the Political Science gradually increased as many Sovereign independent countries joined the world political system. It speaks about the inevitable cooperation between different countries.

The international relations as a network became very important due to the Industrial Revolution, and its effects. Modernisation, technological development, and transport facilities led to formation of regional groups between neighboring countries resulting in growing importance of international relations. Study of Political Science enlightens about all these subjects in detail.

6. Political Science explains world organisations: The United Nations Organisation has been putting continuous efforts to promote peace, cooperation and friendship among nations of the world. Its agencies are spread throughout the world undertaking various activities for the development of humankind. Political Science acquaints us with all these matters.

TS Inter 1st Year Political Science Model Paper Set 2 with Solutions

Question 2.
Discuss Monistic theory of Sovereignty.
Answer:
Introduction : John Austin was a great English jurist of eighteenth century. He published a famous book ‘Lectures on Jurisprudence’ in 1832. Austin was the chief exponent of the monistic theory or legal theory of sovereignty. He was influenced by the writings of Hobbes and Bentham.

Main features of John Austin’s Theory:
1. It is Determinate: Sovereignty is a determinate person or a body of persons. The sovereign need not be a single person alone. The state is legal order wherein there must be a determinate authority. This determinate authority acts as the final source of the power.

2. Sovereign receives habitual obedience: That the deterrninate human superior receives habitual obedience from the bulk of society. The obedience should come from the bulk of the society, where the habitual obedience from the bulk of the society is not for the coming there is no sovereign power.

3. Sovereign is indivisible: That the sovereign power is not divisible. it is a unified one and therefore cannot be divided. There is no limitation on his sovereignty and it cannot be divided.

4. Sovereignty is essential: Sovereign power is essential to every political society. A non-sovereign society is neither political nor independent:

5. Law is a command: Law is a command of the sovereign and whatever may be command is, Law. Since it is a command, failure to obey law is to be punished.

6. The subjects have no right against the state: Rights are those which are legally permitted by the sovereign and upheld by him.

Criticism: Austin’s monistic absolute theory of sovereignty has been criticized by many writers like A.V.Diecy, H.J.Laski, and J.C.Gray. Henry Maine, and Sidgwick are prominent. They criticized this theory on the following grounds. ,
1. It is Unhistorical: Henry Maine stated there was no historical evidence for Austin’s therory.

2. Opoose Democracy: Austin’s theory of sovereignty is against the spirit of democracy becasuse it supported the concept of absolute and relentless sovereignty.

3. Indivisibility not real: According to Austin indivisibility is an important attribute of sovereignty. This seems to be an unsustainable proposition. In every political society there is a division of functions- and without such division no government can be conducted successfully.

4. Sovereignty cannot be located: Sovereignty in the Austinian sense cannot be located in a modern democratic state.

5. State not supreme: Austin treated the state as the omnipotent and supreme organization32. But that is not real. Stae did not poses, such a characteristic feature. There are several agencies, institutions, and organizations in the state. State is one among them. The state is not the only organization in human society.

6. Customs and traditions ignored: Austin’s theory failed to recognize the influence and importance of customs and traditions of the people. It stands that the determinate human superior (having sovereignty) could not exercise the administrative affairs without recognizing the various customs and traditions. But it is a known fact that customs and traditions have been influencing The law making process since ancient period.

Conclusion: In spite of the above criticism Austin’s theory is recognized as significance in political science. It has acquired special place in political science and in jurisprudence in a short time.

Question 3.
Explain the relationship between liberty and law.
Answer:
Law and Liberty are the two fundamental concepts in political science. These two concepts are interdependent. There is no unanimity of opinion among political philosophers in regard to the relation between Law and Liberty.

There are two different schools which gave contradictory opinions. One school of thought believed that Lãw and Liberty are antithetical to each other. The other school of thought believed that Law and Liberty are interrelated to each other. Let us explain the two versions.

i) Law and Liberty are Antithetical: Individualists like J.S.Mill, Herbert Spencer, David Ricardo, and Adam Smith believed that law always restricts the activities of human beings. The state is the principal agency which destroys individual’s liberties. It will not allow the citizens to take active part in the affairs of state and Government.

Similarly, it becomes a hurdle in performing the economic activities of the nation. The recent liberalised economic policies in many countries changed the pace, of their economies. These policies enabled the people to freely participate in economic activities. Therefore, individualists believed that state is a necessary evil institution. They stated that the government is the best which governs the least. Therefore, law and liberty are antithetical to each other.

ii) Law and Liberty are Complementary: The socialists and communists believed that Law and Liberty are complementary to each other. They regarded the state as ä welfare agency. Law imposes restrictions essential for the social welfare. ltjs a fact that the capitalist class exploited the working class. The state shall eradicate the evils of exploitation by making necessary laws.

The idealists believed that state is a Moral Agency. The state represents the general will of the community. Individuals will be free when they obey the laws of the state. Moussolini gave a slogan “Nothing against the State”. Law always protects the interests of the people. Therefore, both the concepts of Law and Liberty are complementary to each other.

Question 4.
What is Individualism? Explain it.
Answer:
Meaning: Individualism means the state should leave the individual alone. This theory is also known as the laissez-faire theory. Laissez-faire is a French term which means leave alone. It regards the individual as the centre of social life. According to this theory, the individual freedom should be given maximum scope and the state interference should be reduced to the minimum.

Ethical Argument: According to J.S.Mill, state interference goes against the development of the individual personality and character. When government interferes and takes upon its shoulders the responsibility of doing what the individual should do, the individual loses the sense of responsibility and self-reliance and his personality is destroyed. He even advocated against the tyranny of the majority over the individual.

Economic Argument: Adam Smith put forth the economic argument in favour of individualism. Every individual tries to get the maximum and would do his work well in which he is personally interested. He spoke in terms of the enlightened self-interest of the individual. The state must not interfere in the economic activities of men like trade, commerce and industry etc. and with its interference, the individual loses all his incentive for economic activity. Free competition will lead to improvement in the quality of industrial output and will also result in lowering of prices.

Biological Argument: Herbert Spencer put forth the biological argument to support individualism. According to him, just as in the animal world the fittest survives, in society also, the individual should struggle for himself and survive or perish. Survival of the fittest is the law of nature and the progress of the society depends upon the elimination of the unfit by the fit. The duty of the state is simply to allow the fullest scope in the struggle for existence. The state has no business to come forward to help the poor, the aged and the sick.

Empirical Argument: Experience shows that wherever and whenever the state regulated and controlled industry the result has been unnecessary delay, waste and inefficiency. It was argued that whenever the state had tried to control and regulate the social or economic life of the community, it had miserably failed in its attempts. Moreover, state management means routine, red-tapism, unnecessary delay, bad economy and corruption.

Criticism: Individualists regarded the state as a necessary evil, but actual experience has shown that it is not bad. The state has to interfere in the larger interest of society. It does exist for the sake of good life.

Individualists contend that laws restrict liberty. This is wrong contention. Laws do not curtail liberty, but maintain and promote it. The argument of the survival of the fittest is misleading, cruel, in human, dangerous and unethical. According to this principle, the weak, the Old and infirm have no right to live. Such view is observed. Hence, such a cruel philosophy is worth rejection.

TS Inter 1st Year Political Science Model Paper Set 2 with Solutions

Question 5.
Discuss Judiciary and its Functions.
Answer:
The Judiciaiy is the third important organ of the government. Legislature enacts the laws. The executive implements them. But it is the judiciary which decides the constitutional validity of these laws both in theory and practice. The judiciary keeps the democratic government within the constitutional limits. The judiciary also has to protect the law abiding citizens and punish the criminals. It consists of the judges and magistrates éharged with the duty of administering justice.

In brief, it is the branch of the government which settles disputes and administers justice.

Functions of the Judiciary:
Dispensing Justice: It has to solve the disputes between the citizens, the citizens and the government and different governments. It punishes the criminals after due trial.

Protection of Civil rights’: The courts of law protect the fundamental rights given to the citizens by the Constitution. The citizens can approach the court of law when their fundamental rights are violated. The courts issue writs for enforcement of these rights. The citizens can obtain stay orders from courts in advance when they apprehend violation of their rights. In order to protect the rights of the individuals, the courts of law can issue writs like Habeas Corpus, Mandamus, Prohibition, etc.

Constitutional protection: The Judiciary safeguards the constitution in the democratic countries. The Judiciary annuls the unconstitutional decisions of the legislature and the executive. By interpreting the constitution and laws it reviews the laws of the legislature and the actions of the executive and strikes down unconstitutional measures. This authority of the judiciary is called as ‘Judicial Review’.

Protection of equilibrium in federal system: Judiciary plays a key role in the federal system. The courts of law solve the disputes between the provinces and the central government ‘.nd a1o between one province and another. It supervises to see that neither the disputes between the central and provincial governments that arise due to matters relating to the division and distribution of powers. Advisory functions: The highest court of justice provides advice to the head of the State on request. For instance in India, the President takes the advice of the Supreme Court on certain problems of constitutional applications. Appellate Jurisdiction: The highest court of justice has to provide justice on the appeals made against the judgements of the lower courts. At times, it ratifies the judgements given in the lower courts and some other times, such judgements are reversed.

Protection of records: The judiciary has to preserve all the cases along with their judgements. These records will help lawyers and judges in the trial of similar cases in future.

Providing service regulations: The courts of justice prescribe service regulations of employees working in courts of law.

Functioning as ‘the Head of State: In some countries and under certain conditions, as identified in India, the Chief Justice of the Supreme Court can function as acting Head of the State when there is no Vice-President in office.

Section – B
8 x 5 = 40 Marks

Note: Answer any EIGHT of the following questions in not exceeding 20 lines each. Each question carries 5 Marks.

Question 1.
Discuss the relationship of Political Science and Economics.
Answer:
Political Science – Economics: Economics studies the aspects like wealth, production, distribution and exchange of goods. It studies about various methods to accumulate wealth. Economics is a sociological study of the aspects like wealth, production and distribution. All the social institutions and political theories place the human life on a right track. A clearly defined political system is very essential for a man to become a good and ideal citizen. Economics helps in different ways to study the human welfare.

Economics tries to coordinate the methods of satisfying unlimited wants with limited resources. Lack of peace and dissatisfaction prevail in a society when the economic needs are not satisfied. The Primary needs like food, clothing, shelter, education and medical aid are to be satisfied. Otherwise, life becomes sorrowful. If the basic needs are not satisfied, the individual has to spend all his energy for that purpose only.

A poverty-stricken society gives scope for the prevalence of immorality and anti-social elements. An individual suffering from hunger, ignorance, and ill-health cannot be in a position to assess his political aims and responsibilities. He resorts to many crimes to satisfy his hunger. Such individual entertains a spirit to adopt illegal means for his progress. He cannot use his rights properly and discharge hisduties. A citizen without basic needs cannot understand the value of right to vote. The communists feel that democracy cannot be successful without a socialist economy. Aristotle warned that economic in equalities lead to social revolutions.

Even though Political Science and Economics are two different disciplines, their common aim is the welfare of people. The policies related to the production, consumption, proper use of exchange units, removing inflation, contributing for the accumulation of national wealth, promotion of industrial development are very much a part of the activities of a modern State. The impact of economic policies is very much considerable on all the above policies. It is quite possible to solve many economic problems through a political system only.

TS Inter 1st Year Political Science Model Paper Set 2 with Solutions

Question 2.
Explain the difference between state and Government.
Answer:

State Government
1. State is broader. It consists of all the people. 1. Government is narrower. It is a body of few citizens.
2. State is permanent. 2. But Government is transitory Government keeps on changing. For ex: In India NDA at the national level has been replaced by the UPA.
3. State – has sovereign power. 3. Government does not have sovereignty. But, Governments exercise the sovereign power in the name of state.
4. All states are universal and similar in nature and characteristics. 4. Government differs from state to state depending upon the wishes of the people or the constitutions of the respective States. For ex: Parliamentary, Presidential Govt, etc.
5. Loyalty of the citizens to their state is compulsory. 5. In a democracy, people have the right to control the acts of omission and commission of the Government.
6. State is a whole. It consists of four essential elements like population, territory, government and sovereignty. 6. Government is one of the four elements of the state. Hence, it is the part and parcel of the state.
7. The state is master; it can place and replace the governments according to their efficiency and popularity. 7. The status of the government in relation to the state is that of a master and servant. The survival of the Government depends on the pleasure of the state.
8. Membership of the State is Compulsory No one is exempted from its membership. 8. Membership of the Government is not compulsory ‘It depends upon the will of the person concerned to become the member of the Government or not.

Question 3.
Describe the merits of Nationalism.
Answer:
The Nationalist charactor of the Modern State has the following merits.

  1. Nationalism removed ‘the mutual differences, personal animosities and internal feeds. It promoted unity integrity and solidarity among the people of a nation. It enabled them to understand the neighbour’s point of view. So, it promoted closer understanding among the people.
  2. Nationalism made the people obey the government.
  3. It helped in achieving the progress of a nation in a short period.
  4. It accelerated the pace of the development of the state. It provided a democratic bare to the government and those strengthened the administrative system.
  5. It is anti-imperialistic. So it does not allow economic exploitation.
  6. It secured political stability and peaceful social atmosphere.

Question 4.
Define Law and mention the features of Law.
Answer:
Law is an mportant concept in the study of political science. it is an important feature of modern štate. Law regulates the external behaviour of individuals.

Meaning: The term Law’ is derived from the Teutonic word which means something fixed.

Features of Law: The following are some important features of law.

  1. Law comprises some rules and regulations which are approved by the sovereign.
  2. It is enforced by the state. it is valid because it is sanctioned by the State.
  3. It is definite, precise and universal.
  4. It reflects the will of the people.
  5. Any violation of law leads to punishment.
  6. Laws are compulsory and cohesive in nature.
  7. Law aims at securing and promoting the individual and general welfare.
  8. Law is dynamic as it goes on changing according to the needs of the people.

Question 5.
Explain the concept of Satyagraha.
Answer:
Gandhi explained Satyagraha not as a philosophical doctrine but as a means to fight against the foreign rule and to achieve social and economic justice.

Gandhi formulated the word satyagraha when he was in South Africa. He called satyagraha as “Love Force” and “Soul Force”. Truth cannot tolerate violence. Even the guilty should not be punished with violence. A sin for one may not be to the other. At once, the search for truth must be only on non-violent means. We have to try to remove the holds on untruth and injustice from his ways by inflicting suffering upon himself. By satyagraha means, Gandhi said that inflicting suffering not on the evil-doer but upon himself;

Political faith of Satyagraha:
Politically, Satyagraha depends upon three principles of faith:

  1. Absolute faith in non-violence
  2. The basis of any Government is the consent of the people.
  3. No country can develop without self-suffering, self-sacrifice, trials and tribulations. This is like the labour pains a mother suffers to deliver a child.

Principle of a satyagraha:
A true, satyagraha has to follow the following principle alone:

  1. Truth means not to lie. It is divine. The evil laws are to be disobeyed through non-violent means.
  2. Non-violence means not to kill. The dynamic factor in it is “Love. Its essence is to love the entire life on earth.
  3. A satyagraha has to observe complete celibacy (Brahma charya) should not look any man or women with amorous looks.
  4. Should not eat more than necessary
  5. Should not steal. It does not mean stealing the things of others.
  6. One has to live on his labour (Bread Labour).
  7. Should not purchase or possess foreign goods. He has to purchase and use only swadesh.
  8. He should tread fearlessly. To love and to search for truth fear lessness is an essential.
  9. Should not observe untouchability is not sufficient he has to fight against it.
  10. Observe religious tolerance.

Forms of Satyagraha:

  1. Civil disobedience.
  2. Non-co-operation.
  3. Hunger strike.
  4. Hartal.
  5. Hazrat.

These methods are to be used by a satyagraha according to the necessity to fight against foreign rule and all types of injustice. Satyagraha has many forms in practice. Non-co-operation to the evil-doer is a mild form of satyagraha. Civil disobedience is an intensive, potent and powerful weapon of influence. Civil disobedience may be of individual or of mass public.

Non-co-operation. Hartal, Hazrat are other forms of satyagraha. The methods of satyagraha are also different. Hunger strike is one form of satyagraha. One should use hunger strike (non-eating) against those who intimately associate and love us.

TS Inter 1st Year Political Science Model Paper Set 2 with Solutions

Question 6.
Explain Political Rights.
Answer:
Political rights are those rights which enable the individuals to participate in the political affairs of the state.
The following are the important political rights:
1. Right to vote: Right to vote is the most important political right enjoyed by the citizens in modern democratic states. It serves as a powerful weapon for adult citizens in choosing their representatives to various legislative bodies. It makes them as real sovereign All the citizens are entitled to this right without any discrimination based on creed, colour, language, race, region, religion, sex etc. However, persons such as aliens and minors are deprived of this right.

2. Right to contest in elections: This right empowers the citizens to contest as candidates to various legislative bodies in the state. Especially this right enables those, who have political sagacity, enthusiasm and dynamic nature, to actively participate in the political dynamics of the state. As a result, it increases political enthusiasm among the citizens. Such an element is considered as a base of democratic polity.

3. Right to hold public offices: This right provides opportunities to the citizens to hold various public offices for a definite period. It gives no scope for exclusion of citizens or conferring special privileges to some at the cost of others. This helps the citizens to exercise authority in a dignified manner.

4. Right to petition: This right enables the citizens to forward petitions denoting their requirements or grievances. It is considered as a vital political right in the modem state. The citizens could be able to find solutions to their immediate or long pending issues by bringing them to the notice of the government through this right. It also helps the public authorities to know the grievances of the people and attend to them properly and promptly.

5. Right to criticism: This right gives opportunity to the citizens to criticize the various public policies and programmes. It also enables them to highlight the omissions and commissions of the leaders, and administrative personnel at various levels. It also gives scope for the citizens to render positive and constructive criticism about the on goings in the government from time to time. Ultimately it keeps the administrative authorities and policy makers to be vigilant in dis charging their obligations.

Question 7.
How citizenship is lost?
Answer:
Citizens loose their citizenship under the following conditions:
1. Renunciation: A person is deprived of his citizenship, if he wishes to become the citizen of another state. One will lose the citizenship of one’s parent state and may become the citizen of a foreign state by naturalization. In India’ the Constitution prescribes that a person who voluntarily acquires citizenship of any other state will no longer be an Indian citizen.

2. Marriage: Generally a woman lose her citizenship when she marries an alien. However some states allow retention of citizenship. For instance in Britain, there is an option to retain British citizenship who marries an alien.

3. Accepting Foreign Service: A person lose his citizenship when he enters into the service of another state. If a person accepts a permanent job in the government of a foreign state, he foregoes the citizenship of his native state.

4. Obliging Foreign Decorations or Titles: When a citizen obliges to receive foreign decorations or titles,’it may lead to the forfeiture of his Citizenship.

5. Prolonged Absence: Prolonged absence in the native state beyond a certain period may lead to the loss of citizenship. In some states like France and Germany citizens who are absent themselves from their native country for more than ten years will loose their citizenship.’

6. Treason or Crime: Involvement of a citizen in a serious crime and subsequent proof of his action will also lead to the loss of citizenship. Especially those persons who directly or indirectly participate or extend assistance to anti-state, anti-social and anti-governmental activities, will loose their citizenship by a special notification to that effect.

7. Desertion from Army: Desertion from army thereby jeopardizing the security of a state leads to the forfeiture of citizenship.

Question 8.
Define Democracy and mention its merits.
Answer:
Definitions:

  1. Abraham Lincoln: Democracy is a government of the people, by the people and for the people’.
  2. lord Bryce: ‘Democracy is that form of government in which the ruling power of the state is vested not in a particular class but in the members of the community as a whole’.
  3. JR. Seeley: ‘Democracy is a government in which everyone has a share”.

Merits: Democracy has the following merits.
a) Efflclent government: Prof Garner described democracy as an efficient and effective government. The government in democracy carries all its activities efficiently and effectively both in normal times and emergencies.

b) Upholds individual liberties: Democracy is the only government that upholds individual liberties. It guarantees certain civil rights to the people thereby providing an opportunity for them to become ideal and responsible citizens.

c) Assures equality: Democracy assures equality of individuals in political and economic spheres. The people living in demoratic nation enjoy all the political, civil, and economic rights and privileges equally without any discrimination.

d) Educates the masses: democracy is described as a laboratory for large-scale experiments in public education. The masses in democracy are educated and enlightened through public meetings, election campaigns, distribution of pamphlets, etc.

e) Promotes patriotism: The people in a democracy think that the country is their own property. When the country is in difficulties, they come forward to protect the interests of the nation. Therefore democracy develops patriotic spirit in the people.

f) Develops sense of responsibility: J.S. Mill says that “democracy promotes a better and higher form of national character than any other policy what ever Since it is a rule by themselves, the people behave with a great sense of responsibility.

g) Training school for citizenship: De Tocqueville, a French writer said that democracy serves as a training school for citizenship. Democracy promotes intellectual and moral qualities among the people.

h) A rational government: Democracy is based on the principle that no man is infallible. It adopts a process of discussion and criticism which serves as necessary correctives to the abuse of power. Besides, they safeguard the rational nature of the political system.

Question 9.
Explain any four merits of Secularism.
Answer:
Merits of Secularism: The following are some of the important merits of secularism.
1. Equality: Secularism forms as the basis of equalitarian society. It treats the people belonging to all religious denominations as equal. It gives no recognition to the man-made inequalities and discriminations based on caste, colour, community, region, religion, language, race etc. People will have a strong favourable impression towards the nation.

2. Religious freedom: Secularism enables the individuals to enjoy their religious freedom to their full extent. The state will not interfere in the Religious affairs of individuals. The Constitution and various laws in a Secular State will provide individuals with complete freedom to embrace, profess, practice, and propagate any religion as they like.

3. Law and order: Now a days one can observe unhappy, miserable and pro-religious movements that are organized by different sections of government, state and other departments. The maintenance of communal harmony became a challenging task for the state in Spacifying the feelings the people belonging to various religious denominations. Secularism avoids communal clashes and religious bigotry and animosities in the šociety, This is due to the fact that secularism ultimately promotes religious harmony among the people.

4. Rule of law: Secularism accords recognition to the concept of Rule of Law. A state following secularism will enact laws and implements them keeping in view the interest of not a particular religious denomination, the people belonging to all religious denominations. It will not take into account the religious dogmas while making laws. Similarly, it makes no discrimination between the people on the ground of religion.

TS Inter 1st Year Political Science Model Paper Set 2 with Solutions

Question 10.
Define Constitution. Explain its features.
Answer:
Features of the Constitution:
1. Preamble: Every Constitution will have a preamble. The preamble denotes the aims and aspirations of the Constitution. It is like the soul of the Constitution. Hence, preamble is considered as an important feature of the Constitution.

2. Clarity: Clarity is another important feature of the Constitution. The Constitution clearly explains about the different policies and methods of governance. It is written in a simple and clear language.

3. Incorporation of Fundamental Rights: Every Constitution includes some fundamental rights. These fundamental rights are meant for safeguarding the freedoms of the citizens. They enable the citizens to realise their personality in various spheres. They help the citizens for leading a happy and honourable life in the state.

4. Brevity: Brevity is another feature of a Constitution. Brevity avoids confusion among the individuals in understanding and interpreting provisions. Unnecessary elements are not included in the Constitution. It should be precise. It must not contain large number of clauses.

5. flexibility: The Constitution must be flexible for adapting the wishes are aspirations of the people from time to time. There must be a scope of amending the provisions of the Constitution if necessary. Frequent changes in the Constitution tend to weaken the spirit of the Constitution. But, at the same time, the Constitution of a modern state should be adaptable to the progressive changes.

6. Permanence: Permanence is one more feature of the Constitution. The Constitution must have everlasting values for the welfare of the whole nation. It represents the actual structure of the state and its political institutions. It obliges the customs of the people.

Question 11.
Write brief note on Role of law.
Answer:
Rule of law is a unique feature of Brtish constitution. It at first originated in England later many states like India and the United States have adopted this feature. Rule of law is purely based on the principle of common law. It denotes that law should be general in form. It should be uniformly applicable to all the citizens living in a state. There should not be different types of law for various sections of the community.

In other words the government must treat all the citizens equally as per the law. The governance and administration in a particular state should be carried on in accordance with the laws. A.V. Dicey in his “Law of the constitutions (1885) gave a precise explanation of the rule of law.

‘According to him, rule of law stands for equality before the law. This implies equal subjection of all classes including the officials or common man. Law makes no discrimination between individuals. A.V Dicey says that Every official from Prime Minister to the attender, all are equal before the law”.

Therefore, Rule of law is helpful to all the citizens in protecting their responsibilities in a more effective manner. The constitution of India too recognizes the Rule of law as a basic feature of India constitution, The Supreme Court of India time and again declared Rule of law as a basic structure of Indian Constitution. Articles 14 to 21 of the Indian Constitution have in corporated the spirit of this concept.

However, the scope of this concept is gradually shrinking owing to the overburdening of legislative work with enormous functions. On the whole, the cardinal virtue of Rule of law is that ‘All are equal before law and no one must arbitrarily be punished” constitutes the core value of any democratic system in the world.

Question 12.
Distinguish between Unitary and Federal form of Government.
Answer:
Distinguish between Unitary and Federal Governments:

Unitary Government Federal Government
1. May or may not be written constitution. 1. There will be written constitution.
2. It has Flexible constitution. 2. It has rigid constitution.
3. There is only one Government for the entire country. 3. There will be two types of governments. i.e., union level and provincial level.
4. Centralization of powers. 4. Decentralization of powers between centre and state governments.
5. Government is not as much Democratic form of government. 5. It is purely Democratic government which all governments take their part in Decision-making.
6. There are uniform laws throughout the country. 6. There are central laws and state laws.
7. No need of independent Judiciary. 7. A special judiciary with wide powers.
8. Possibility of despotism. 8. Centre and state work according to constitution, so no chance for despotism.
9. Government machinery is simple and flexible. 9. Government machinery is complex and rigid.
10. Suitable for smaller states. 10. Suitable for larger states.
11. Legislature may be bicameral (Britain) or unicameral (China). 11. Legislature should, have two chambers.
12. Constitution may be supreme (Japan) / or may not be supreme (Britain). 12. Supremacy of the Constitution.
13. Scope for political stability and integrity. 13. Limited scope for political Stability and integrity.
14. The powers of regional governments are easily altered by the central government. 14. The powers of regional governments cannot be altered by the central government.


Section – C

15 × 2 = 30 Marks

Note: Answer any FIFTEEN of the following questions in not exceeding 5 lines each. Each question carries 2 Marks.

Question 1.
Define Political Science.
Answer:
Political Scientists gave various definitions on Political Science.
They are as follows.

  1. J.W. GARNER: ‘Political Science begins and ends with the State”.
  2. R.G. GETTLE: “Political Science is ” The historical investigation of the state in the past, an analytical study of the state of present and what the state ought to be in the future”.
  3. ROBERT DAHL: “Political Science is the Scientific study of importance of power, Authority and influence.’

Question 2.
Post-Behaviouralism.
Answer:
Post – Behaviourahsm is a reaction to Behaviouralism. It brought a change of perspective in political science. Post behaviouralism considered ethics and values as equally important along with the facts and methods.

Question 3.
Monistic theory of Sovereignty.
Answer:
‘Monistic theory of sovereignty was advocated by John Austin, an eminent British Jurist in his famous book” Lectures on Juris Prudence in 1832. According to John Austin Sovereignty is a determinate person. This determinate Authority acts as the final source of the power.

The determinate humaa superior receives habitual obedience from the bulk of society. The sovereign power is not divisible. It is a unified one and therefore cannot be divided. There is no limitation on his sovereignty and it cannot be divided.

TS Inter 1st Year Political Science Model Paper Set 2 with Solutions

Question 4.
Define Nation.
Answer:
Nation is derived from two Latin words – “Nates” and “Natio” which means birth, Lord Bryce defined it as “A Nation is a nationality which has organised itself into a political body, either independent or desiring to be independent. ‘It means That the people of country are called as a National if they are united by characteristics of nationality and have.a strong desire for political independence or if they are politically free.

Question 5.
What is Positive Law?
Answer:
Positive Law: Positive law is created by the human agency. It is also known as political law. It is framed on the basis of the existing social and political conditions. it is sanctioned by the Sovereign Political Authority Violation of positive law leads to punishment.

Positive law is further classified into two categories.

  • National Law and
  • International Law.

Question 6.
What is legal equality?
Answer:
Civil equality implies availability of civil rights without any discrimination on the basis of caste, colour, creed, place of birth, religion and sex. When all citizens are subject to the same law and when law neither confers special privileges on some nor makes any individual suffers owing to his social status, religious beliefs, political views, race or caste etc., there exists civil and legal equality. In a society civil equality is said to prevail when there is equality before law and equal protection of laws. This civil and legal equality emerged in modern times with adoption of democracy.

Question 7.
What is Legitimacy?
Answer:
The word legitimacy has been derived from the Latin word Legitimas which means lawful. According to Max Weber legitimacy is based in belief and gets obedience from the people power is effective only if it is legitimate. Undoubtedly, power has the right to use coercion but that is not its chief element power should be based on legitimacy, otherwise it would invite trouble and may prove ineffective. J.C.Pleno and R.E.Riggs define legitimacy as the quality of being justified or willingly accepted by subordinates that convert the exercise of political power into rightful authority.

Question 8.
J.S.Mill.
Answer:
John Stuart Mill was the most prominent political thinker of 19th century. He was the most influential intellectual who propagated the Ideas of Liberty, Equality and Welfare State. He paved the way form the emergence and spread of individùalism against the hitherto.

Question 9.
Satyagraha.
Answer:
Gandhi explained Satyagraha not as a philosophical doctrine but as a means to fight against the foreign rule and to achieve social and economic justice. Gandhi formulated the word satyagraha when he was in South Africa. He called satyagraha as love Force and ‘Soul Force. By satyagraha means, Gandhi said that inflicting suffering not on the evildoer but upon himself.

Question 10.
Significance of Human Rights.
Answer:
The United Nations reaffirmed that the people and governments of every state must strive for respecting individual freedoms and human rights. The concerned authorities and agencies of the United Nations held several international conferences and invited internationally acclaimed intellectuals, jurists, and heads of states for eliciting their valuable opinions on extending human rights to every section of human communities throughout the world.

Question 11.
Moral rights.
Answer:
Moral rights denote claims based on the moral code of the community. These rights are morally prescribed to men in the society. The ethical or moral principles in the society act as the basis of the moral rights. Customs, traditions, and usages are regarded as the basic source of these rights. Men enjoy these rights in a civil society. These rights are based on the moral conscience of the people. They don’t have legal support. However, they are backed by the society So violation of these rights is not considered as a crime. Individuals could be punished for their violation. Moral rights are indefinite and vague. But they are popular in nature. The State cannot ignore these rights for a long time.

Question 12.
What is dual citizenship?
Answer:
Dual Citizenship means possession of two citizenships in two States. Ex: Children born to American citizens in other States acquire citizenship in both the States – one in their parent State and the other in the State, where there are born. Dual citizenship applies to the children until they attain adulthood. Later they have to choose citizenship of any one of the two states.

Question 13.
Define indirect Democracy.
Answer:
Indirect Democracy: Indirect democracy is also known as representative democracy. In this type of democracy a clear distinction is made between the immediate sovereign and the ultimate sovereign. The legislature which consists of the elected representatives of the people formulates and expresses the will of the state. Hence, the legislature is the immediate sovereign authority.

In this type of democracy, the people elect their representatives periodically and review their activities during their full, term. If their activities are proven to be unsatisfactory; the people can with draw their trust in them and choose new representatives. Representative democracy thus combines efficient administration with popular sovereignty.

In representative democracy, the parties articulate and organize the will of the people and act as the transmission belt between the government and the governed. In a representative democracy, the ultimate source of authority remains the people.

Question 14.
What are Referendum?
Answer:
Referendum means ‘Refer to’. This method is used to ascertain the public opinion on important legislation. In some regions, the public opinion is sought on the problems of constitutional law and ordinary law. This is called referendum. Referendums is of two types.
They are

  • Compulsory referendum
  •  Optional referendum.

Question 15.
Define Secular state.
Answer:
D.E Smith defined Secular State as “None while guaranteeing individual and corporate freedom of religion, which deals with the individual as a citizen irrespective of his religion’.

TS Inter 1st Year Political Science Model Paper Set 2 with Solutions

Question 16.
What is a Flexible Constitution?
Answer:
A flexible constitution is one whose provisions can be amended easily It requires no special procedure for changing its provisions. It can be amended by the authorities by adopting the same procedure of ordinary laws. So we do not find differences between ordinary and constitutional laws. Flexible constitutions were prevalent in the ancient period.
Ex: British Constitution.

Question 17.
What is preamble?
Answer:
Every Constitution will have a preamble. The preamble denotes the aims and aspirations of the Constitution. It is the soul of the Constitution. Hence, preamble is considered as one of the important features of the Constitution.

Question 18.
What is independence of the Judiciary?
Answer:
Independence of judiciary implies an opportunity to the judges to perform their duties without fear or favour and act impartially. The judgs should have no relation with the legislature and the executive. Both the organs should not interfere in the functioning of the judiciary. It should be protected from political pressure and influence. Otherwise, the very purpose of judiciary stands defeated.

Question 19.
House-of lords.
Answer:
The House of Lords, also known as The House of peers, is the upper House of the parliament of the United Kingdom. Membership is granted by appointment or else by heredity. Like the House of common, it meets in the palace of West Minister. Currently, there are 300 members in the House of Lords members of whom 240 are Elected members and 60 appointed. Independent members up to 12 Church of England Bishops may sit in the House as ex officio Lords spiritual. Elected members will serve a single non-renewable term of 15 years.

Question 20.
President.
Answer:
Presidential government confers both the Nominal and Real executive powers in a single person namely the president. So the president is not only a nominal executive but also the Real ‘executive. He serves as both the head of the state and government. He enjoys all executive powers both in name and in practice. He makes Independent decisions keeping in view the popular wishes and national interests. He implements the policies and programmes of the government with the help of some secretaries who owe their existence, continuance and survival to him only.

AP Inter 1st Year Zoology Model Paper Set 8 with Solutions

Varied difficulty levels in AP Inter 1st Year Zoology Model Papers Set 8 cater to students with diverse academic strengths and challenges.

AP Inter 1st Year Zoology Model Paper Set 8 with Solutions

Time : 3 Hours
Max. Marks: 60

General Instructions:
Note : Read the following instructions carefully.

  1. Answer All questions of Section A. Answer ANY SIX questions in Section B and answer ANY TWO questions in Section C.
  2. In Section A questions from Sr. Nos. 1 to 10 are of “Very Short Answer Type”. Each question carries TWO marks. Every answer may be limited to 5 lines. Answer all these questions at one place in the same order.
  3. In Section ‘B’, questions from Sr. Nos. 11 to 18 are of “Short Answer Type”. Each question carries FOUR marks. Every answer may be limited to 20 lines.
  4. In Section ‘C’, questions from Sr. Nos. 19 to 21 are of “Long Answer Type”. Each question carries EIGHT marks. Every answer may be limited to 60 lines.
  5. Draw labelled diagrams wherever necessary in Sections ‘B’ and ‘C’.

Section – A (10 × 2 = 20 Marks)

I. Answer all the questions (Very short answer type)

Question 1.
“Zoos are tools for classification” explain.
Answer:
Zoos are the places where wild animals, taken out of their natural habitat, are placed in protected environment under human care. This enables us to study the various aspects of animal living. Thus it enables us to systematise the organism and position it in the animal world.

Question 2.
What is monaxial heteropolar symmetry ? Name the group of animals in which it is the principal symmetry.
Answer:
When any plane passing through the central axis of the body divides an organism into two identical parts it is called monaxial heteropolar symmetry. In cnidarians it is the principal symmetry.

Question 3.
What are the two chief morphological body forms of cnidarians? What are their chief functions ?
Answer:
The two chief morphological body forms of cnidarians are 1) Polyp 2) Medusa. Polyp is sessile, cylindrical form and produce medusae asexually by budding. Medusa is umbrella shaped and free swimming form and sexually produce polypoid forms.

AP Inter 1st Year Zoology Model Paper Set 8 with Solutions

Question 4.
List out the characters shared by chordates and echinoderms.
Answer:
Chordates share deuteostomeate condition, radial and indeterminate type of cleavage and enterocoelom with the echinoderms.

Question 5.
What do you mean by parasitic castration ? Give one example.
Answer:
Some parasites cause the degeneration of gonads of the host, making it sterile. This effect in called parasitic castration. eg : Sacculina (a crustacean) causes degeneration of ovaries in the crab.

Question 6.
What are the structures with which cockroach walks on smooth surfaces and on rough surfaces respectively.
Answer:
The claws and the arolium help in locomotion on rough surfaces whereas plantulae are useful on smooth surfaces.

Question 7.
What is intima ?
Answer:
Trachea in cockroach is made up of three layers, outer basement membrane, a middle one cell thick epithelium and an inner layer of cuticle called intima. The intima is produced into spired thickenings called taenidia.

Question 8.
What is the effect of light on body pigmentation ?
Answer:
Light influences the colour of the skin. The animals which live in regions of low intensity of light have less pigmentation than that of the animals exposed to light.

Question 9.
What is Periphyton ?
Answer:
The animals that are attached to / creeping on the aquatic plants, such as the water snails, nymphs of insects bryozoans, hydras constitute the periphyton.

Question 10.
Explain the process of leaching.
Answer:
By the process of leaching, water soluble in organic nutrients go down info the soil and get precipitated as unavailable salts.

AP Inter 1st Year Zoology Model Paper Set 8 with Solutions

Section – B (6 × 4 = 24 Marks)

II. Answer any six questions (Short answer type)

Question 11.
Define species. Explain the various aspects of “Species”.
Answer:
Species : Species is the ‘basic unit’ of classification. Species is a Latin word meaning ‘kind’ or ‘appearance’. John Ray in his book ‘Historia Generalis Plantarum’, used the term ‘species’ and described it on the basis of common descent (origin from common ancestors) as a group of morphologically similar organisms. Linnaeus considered species, in his book ‘Systema Naturae’, as the basic unit of classification.

Buffon’s biological concept of species explains that species is an interbreeding group of similar individuals sharing the common ‘gene pool’, and producing fertile offspring. Species is considered as a group of individuals which are :

  1. Reproductively isolated from the individuals of other species – a breeding unit.
  2. Sharing the same ecological niche – An ecological unit.
  3. Showing similarity in the karyotype – a genetic unit.
  4. Having similar structure and functional characteristics – an evolutionary unit.
  5. Species is dynamic.

Question 12.
Describe the three types of cartilage.
Answer:
The three types of cartilage are
1. Hyaline cartilage
2. Elastic cartilye
3. Fibrous cartilage.

1. Hyaline cartilage: It is bluish-white, translucent and glass-like cartilage. Matrix is homogeneous and shows delicate collagen fibres. It is the weakest and the most common type of all the cartilages. Perichondrium is present except in articular cartilages. It forms the embryonic endoskeleton of bony vertebrates, endoskeleton of cyclostomes and cartilaginous fishes. It forms the articular cartilages (free surfaces of long bones that form joints), costal cartilages (sternal parts of ribs), and the epiphyseal plates. It also forms the nasal septal cartilage, cartilaginous rings of trachea, bronchi and cartilages of larynx.
AP Inter 1st Year Zoology Model Paper Set 8 with Solutions 1
2) Elastic cartilage : It is yellowish due to elastic fibres. Matrix has abundance of yellow elastic fibres in addition to collagen fibres. It provides strength and elasticity. Perichondrium is present. It is found in the pinnae of the external ears, Eustachian tubes and epiglottis.

3) Fibrous cartilage : Matrix has bundles of collagen fibres. Perichondrium is absent. It is the strongest of all types of cartilages. It occurs in the intervertebral discs and pubic symphysis of the pelvis.

Question 13.
What are the chief characters of the crustaceans ?
Answer:

  1. They are aquatic.
  2. Head and thorax fuse forming the cephalothorax (covered by chitinous carapace). In some the exoskeleton is hardened by calcium carbonate (crabs and lobsters).
  3. Cephalic region bears two pairs of antennae (antennules and antennae – unique feature), one pair of mandibles and two pairs of maxillae.
  4. Thoracic and abdominal appendages are ‘biramous’.
  5. Respiratory organs are gills (branchiae).
  6. Excretory organs are green glands or antennary glands.
  7. Sense organs include antennae compound eyes, statocysts, etc.
  8. Development is indirect and includes different larval forms. Examples : Palaemon (freshwater prawn), Cancer (crab), Balanus (rock barnacle), Sacculina (root-headed barnacle), Astacus (crayfish), Daphnia (waterflea).

AP Inter 1st Year Zoology Model Paper Set 8 with Solutions 2

AP Inter 1st Year Zoology Model Paper Set 8 with Solutions

Question 14.
List out eight characteristics that help distinguish a fish from the other vertebrates.
Answer:
Characteristics of group PISCES :

  1. Fishes are completely aquatic poikMo-thermic (cold blooded) animals.
  2. Body of a fish is usually streamlined and differentiated into head, trunk and tail.
  3. Exoskeleton consists of mesodermal scales or bony plates. A few are scaleless.
  4. Endoskeleton may be cartilaginous or bony. Skull is monocondylic. Vertebrae are amphicoelous (centrum is concave at both anterior and posterior faces).
  5. Locomotion is assisted by unpaired (median and caudal) fins along with paired (pectoral and pelvic) fins.
  6. Mouth is ventral or terminal. Teeth are usually acrodont, homodont and polyphyodont.
  7. Exchange of respiratory gases is performed by the gills.
  8. Heart is ‘two chambered’ and is described as ‘branchial heart’ as it supplies blood only to the gills.
  9. Lateral line sensory system is well developed.

AP Inter 1st Year Zoology Model Paper Set 8 with Solutions 3

Question 15.
Describe the process of longitudinal binary fission in Euglena.
Answer:
AP Inter 1st Year Zoology Model Paper Set 8 with Solutions 4
During the process of binary fission, the nucleus, basal granules, chromatophores, cytoplasm undergo division. The nucleus divides by mitosis into two daughter nuclei. Then the kinetosomes and the chromatophores also divide. At first, a longitudinal groove develops in the middle of the anterior end. This groove extends gradually towards the posterior end until the two daughter individuals are separated. One daughter Euglena retains the parental flagella. The other daughter individual develops new flagella from the newly formed basal granules. The stigma, paraflageliar body and contractile vacuole of the parent disappear. They develop afresh in both the daughter euglenae. The longitudinal binary fission is known as symmetrogenic division, because the two daughter euglenae resemble each other like ‘mirror images’.

Question 16.
Draw neat labelled diagram of Paramecium and label its important structures.
Answer:
AP Inter 1st Year Zoology Model Paper Set 8 with Solutions 5

Question 17.
What are the adverse effects of tobacco ?
Answer:
Effect : Smoking increases the carbon monoxide (CO) level and reduces the oxygen level in the blood. Nicotine stimulates the adrenal gland to release adrenaline and nor-adrenajine into blood.

These hormones raise the blood pressure and increase the heart rate. Smoking is associated with bronchitis, emphysema, coronary heart disease, gastric ulcer and increases the incidence of cancers of throat, lungs, urinary bladder etc. Smoking also paves the way to hard drugs. Yet smoking is very pravalent in society, both among young and old. Tobacco chewing is associated with increased risk of cancer of the oral cavity.

Question 18.
Describe the formation of schizocoelom and enterocoelom.
Answer:
Formation of Schizocoelom : Animals in which the body cavity is formed by ‘splitting of mesoderm’ are called schizocoelomates. Annelids, arthropods and molluscs are schizocoelomates in the animal kingdom. All the schizocoelomates are protostomians and they show ‘holoblastic’, ‘spiral’ and ‘determinate’ cleavage. The 4d blastomere or mesentoblast celi of the early embryo divides to form mesodermal blocks between the ectoderm and the endoderm and replaces the blastocoel. The split that appears in each mesodermal block leads to the formation of Schizocoelom. e.g : Annelida, Arthropoda, Mollusca.
AP Inter 1st Year Zoology Model Paper Set 8 with Solutions 6
Formation of enterocoelom : Animals in which the body cavity is formed from the mesodermal pouches of archenteron are called enterocoelomates. Echinoderms, hemichordates and chordates are the enterocoelomates. In these animals, mesodermal pouches that evaginate from the wall of the archenteron into the blastocoel are fused with one another to form the enterocoelom. All the enterocoelomates are deuterostonies and they show radial and indeterminate cleavage.
AP Inter 1st Year Zoology Model Paper Set 8 with Solutions 7

AP Inter 1st Year Zoology Model Paper Set 8 with Solutions

Section – C (2 × 8 = 16 Marks)

III. Answer any two questions (Long answer type)

Question 19.
Describe the life cycle of Plasmodium Vivax in man.
Answer:
Life cycle of Plasmodium in man (The human phase) : In man, the Plasmodium reproduces by asexual reproduction called schizogony. It occurs in liver cells (hepatocytes) as well as in RBC. In liver cells, it is called hepatic schizogony and in RBC it is called erythrocytic schizogony.

Hepatic schizogony : This was discovered by Shortt and Garnham. Whenever, a mosquito infected by Plasmodium bites a man, nearly 2000 sporozoites are released into the blood of man through its saliva. Within half an hour, they reach the hepatocytes where they undergo pre-erythrocytic and exo- erythrocytic cycles.

Pre-erythrocytic cycle : Whenever the sporozoites reach the liver cells, they transform into trophozoites. Theyfeed on the contents of the hepatic cells, assume spherical shape and attain the maximum size. This stage is called schizont stage. Its nucleus divides several times mitotically, followed by the cytoplasmic divisions resulting in approximately 12,000 daughter individuals called cryptozoites or the lstgeneration merozoites. They enter the sinusoids of the liver by rupturing the cell membrane of the schizont and the liver cells. This entire process is completed approximately in 8 days. Now these first generation merozoites have two options i.e., they can enter either fresh liver cells and continue exo-erythrocytic cycle or they can enter RBC and continue erythrocytic cycle.

Exo-erythrocytic cycle : If the cryptozoites enter the fresh liver cells, they undergo the changes similar to that of the pre- erythrocytic cycle and produce the second generation merozoites called metacryptozoites. These are of two types – the smaller micro- metacryptozoites and larger macro- metacryptozoites. This entire process is completed approximately in two days. The macro-metacryptozoites attack fresh liver cells and continue another exo – erythrocytic cycle, whereas the micro- metacryptozoites always enter blood stream and attack fresh RBC to continue erythrocytic cycle.

Prepatent period : The interval between ‘the first entry of Plasmodium into the blood in the form of sporozoites and the second entry of Plasmodium into the blood in the form of cryptozoites is called prepatent period. It lasts approximately 8 days. During this period, the host does not show any clinical symptoms of the disease. It is only a means of multiplication.

Erythrocytic cycle : It was first described by Camillo Golgi. Hence it is also called Golgi cycle. This cycle is initiated either by the cryptozoites of pre-erythrocytic cycle or the micro- metacryptozoites of exp – erythrocytic cycle. In the fresh RBC, these stages assume spherical shape and transform into trophozoites. It develops a small vacuole which gradually enlarges in size, pushing the cytoplasm and nucleus to the periphery. Now the Plasmodium looks like a finger ring. Hence this stage is called signet ring stage. Soon it loses the vacuole, develops pseudopodia and becomes amoeboid stage. With the help of pseudopodia, it actively feeds on the contents of the RBC and increases in size. As a result, the RBC grows almost double the size.

This process is called hypertrophy. The malaria parasite digests the globin part of the ingested haemoglobin and converts the soluble haem into an insoluble crystalline haemozoin. It is called the ‘malaria pigment’ which is a disposable product. During this stage, small red coloured dots appear in the cytoplasm of the RBC known as Schuffner s dots. These are believed to be the antigens released by the parasite. Now the Plasmodium loses the pseudopodia, further increases in size, occupies the entire RBC and becomes a schizont. It undergoes schizogony similar to that of the pre-erythrocytic cycle and produces 12 to 24 erythrocytic merozoites. They are arranged in the form of the petals of a rose in the RBC. Hence, this stage is called the rosette stage. Finally the erythrocyte bursts and releases the merozoites along with haemozoin into the blood. This cycle is completed approximately in 48 hours.

Incubation Period : The period between ‘the entry of Plasmodium into the blood in the form of sporozoite and the first appearance of symptoms of malaria in man’ is called incubation period. It is approximately 10 to 14 days.

Formation of gametocytes : After repeated cycles of erythrocytic schizogony, when the number of fresh RBC decreases, some merozoites enter the RBC and transform into gametocytes instead of continuing the erythrocytic cycle. This process generally takes place when the RBCs are present in spleen and bone marrow.

The gametocytes are of two types namely, smaller microgametocytes or male gametocytes and larger macrogametocytes or female gametocytes. The gametocytes cannot undergo further development in man as the temperature and pH of the blood of man are not suitable for further development. These gametocytes reach the blood circulation and wait to reach the next host. They degenerate and die if they are not transferred to mosquito within a week.
AP Inter 1st Year Zoology Model Paper Set 8 with Solutions 8

AP Inter 1st Year Zoology Model Paper Set 8 with Solutions

Question 20.
Describe the blood circulatory system of peripianeta in detail and draw a neat labelled diagram of it.
Answer:
Circulatory system of Periplaneta : The circulatory system helps in the transportation of digested food, hormones etc., from one part to another in the body. Periplaneta has an open type of circulatory system as the blood, or haemohymph, flows freely within the body cavity or haemocoel. Blood vessels are poorly developed and open into spaces. Visceral organs located in the haemocoel are bathed in the blood. The three main parts associ-ated with the blood circulatory system of Periplaneta are the haemocoel, heart, and blood.
AP Inter 1st Year Zoology Model Paper Set 8 with Solutions 9
Haemocoel : The haemocoel of cockroach is divided into three sinuses by two muscular, horizontal membranes, called dorsal diaphragm or ‘pericardial septum’ and ventral diaphragm. Both the diaphragms have pores. There is a series of paired triangular muscles, called alary muscles. Every segment has one pair of these muscles situated on the lateral sides of the body. These are attached to the pericardial septum by their broad bases and to the terga by their pointed ends or apices. The three sinuses of the haemocoel are known as pericardial haemocoel orthe ‘dorsal sinus’, the perivisceral haemocoel or the ‘middle sinus’ and eternal haemocoel or ‘vental sinus’ or ‘perineural sinus’. The middle sinus is very large as it contains most of the viscera. The dorsal and ventral sinuses are small as they have only heart and nerve cord, respectively.

Heart : the heart lies in the pericardial haemocoel or dorsal sinus. It is a long, muscular, contractile tube found along the mid dorsal line, beneath the terga of the thorax and abdomen. It consists of 13 chambers. Every chamber opens into the other present in front of it. Three of the thirteen chambers are situated in the thorax and ten in the abdomen. Its posterior end is closed while the anterior end is continued forward as the anterior aorta. At the posterior side of each chamber, except the last, there is a pair of small apertures called ostia’ one on each side. Ostia have valves which allow the blood to pass only into the heart from the dorsal sinus.

Blood : The blood of Periplaneta is colourless and is called haemolymph. it consists of a fluid called plasma, and free blood corpuscles or haemocytes, which are ‘phagocytic’. The phagocytes are large in size and can ‘ingest’ foreign particles such as bacteria. There is no respiratory pigment in the blood and so it plays no major role in respiration. The important functions of the blood are :

  1. It absorbs digested food from the alimentary canal and distributes it to the rest of the body.
  2. It brings nitrogenous wastes from all parts of the body to the excretory organs for their elimination.
  3. It carries defensive phagocytes to the places of infection where they engulf the germs and disintegrating tissue parts.
  4. It transports secretions of the ductless glands to the target organs.

Circulation of blood: The blood flows forward in the heart by the contractions of its chambers. At the anterior end of the heart, the blood flows into the aorta and from there it enters the sinus of the head. From the head sinus, the blood flows into the perivisceral and sternal sinuses. On contraction of the alary muscles the pericardial septum is pulled down. This increases the volume of the pericardial sinus. Hence blood flows from the perivisceral sinus into the pericardial sinus through the appertures of the pericardial septum. On relaxation of the alary muscles, the pericardial septum moves upwards to its original position. This forces the blood, to enter the chambers of the heart through the ostia from the pericardial sinus.
AP Inter 1st Year Zoology Model Paper Set 8 with Solutions 10

AP Inter 1st Year Zoology Model Paper Set 8 with Solutions

Question 21.
Write an essay on water as an ecological factor.
Answer:
Water : Water is another important factor influencing the life of organisms. Life is unsustainable without water. Its availability is so limited in deserts that only certain special adaptations make , it possible for them to live there. You might think that organisms living in oceans, lakes and rivers should not face any water-related problems, but it is not true. For aquatic organisms the quality . (chemical composition, pH, etc.,) of water becomes important.

The salt concentration is less than 5 percent in inland waters, and 30 – 35 percent in the seawater. Some organisms are tolerant to a wide range of salinities (euryhaline), but others are restricted ( to a narrow range (stenohaline). Many freshwater animals cannot live for long in sea water and vice versa because of the osmotic problems, they would face.

Adaptations in freshwater habitat : Animals living in freshwaters have to tackle the problem of endosmosis. The osmotic pressure of freshwater is very low and that of the body fluids of freshwater organisms is much higher. So water tends to enter into bodies by endosmosis. To maintain the balance of water in the bodies, the fresh water organisms acquired several adaptation such as, contractile vacuoles in the freshwater protozoans, large glomerular kidneys in fishes, etc.

They send out large quantities of urine, along which some salts are also lost. To compensate the ‘salt loss’ through urine, freshwater fishes have ‘salt absorbing’ ‘chloride cells’ in their gills. The major problem in freshwater ponds is – in summer most of the ponds dry up. To overcome this ) problem most of the freshwater protists undergo encystment. The freshwater sponges produce asexual reproductive bodies, called gemmules, to tide over the unfavourable conditions of ‘ the summer. The ‘African lungfish’, Protopterus, burrows into the mud and forms a ‘gelatinous cocoon’ around it, to survive, in summer.

Adaptations in marine habitat : Seawater is high in salt content compared to that of the body fluids. So, the marine animals continuously tend to lose water from their bodies by exosmosis and face the problem of dehydration. To overcome the problem, of water loss, marine fishes have aglomerular kidneys with less number of nephrons. Such kidneys minimize the loss of water through urine. To compensate water loss the marine fish drink more water, and along with this water, salts are added to the body fluids and disturb the internal equilibrium.

To maintain salt balance (salt homeostasis) in the body, they have salt secreting chloride cells in their gills. Marine birds like sea gulls and penguins eliminate salts in the form of salty fluid that drips through their nostrils. In turtles the ducts of chloride secreting glands open near the eyes. Some cartilaginous fishes retain urea and trimethylamine oxide (TMO) in their blood to keep the body fluid isotonic to the sea water and avoid dehydration of the body due to exosmosis.

Water related adaptations in brackish water animals : The animals of brackish water are adapted to withstand wide fluctuations in salinity. Such organisms are called euryhaline animals and those that can’t withstand are known as stenohaline. The migratory fishes such as salmon and Hilsa are anadromous fishes i.e., they migrate from the sea to freshwater, for breeding; Anguilla bengalensis is a catadromous fish i.e., it migrates from the river to sea, for breeding. In these fishes their glomerular kidneys are adjusted to changing salinities. The chloride cells are adapted to excrete or absorb salts depending on the situation. On entering the river salmon drinks more freshwater to maintain the concentration of body fluids equal to that of the surround water.

Water related adaptations for terrestrial life: In the absence of an external source of water, the kangaroo rat of the North American deserts is capable of meeting all its water requirements through F oxidation of its internal fat (in which water is a by product metabolic water). It also has the ability to concentrate its urine, so that minimal volume of water is lost in the process of removal of their excretory products.

TS Inter 1st Year Political Science Model Paper Set 1 with Solutions

Thoroughly analyzing TS Inter 1st Year Political Science Model Papers Set 1 with Solutions helps students identify their strengths and weaknesses.

TS Inter 1st Year Political Science Model Paper Set 1 with Solutions

Time : 3 Hours
Max. Marks: 100

[Section – A]
3 x 10 = 30 Marks

Note: Answer any THREE of the following questions in not exceeding 40 lines each. Each question carries 10 Marks.

Question 1.
Define Political Science and explain its scope.
Answer:
Introduction: Political Science is a premier social science. It is mainly concerned with the study of the state in its relation with Society, Citizens, Associations and the world at large. Aristotle is regarded as the Father of Political Science. He wrote famous book ‘THE POLITICS”.

Meaning: The word politics is derived from the ancient Greek word ‘POLIS” meaning city. State and polity from ‘Poletieia’ meaning government of constitution. Politics came to mean the study of state and government and the Institutions of the state.

Definitions: Political Scientists gave various definitions on Political Science.

They are as follows:

  1. J.W. GARNER: “Political Science Begins and Ends with the State”.
  2. KG. GETTLE: “Political Science is” The historical investigation of the state in the past, an analytical study of the state of present and what the state ought to be in the future”.
  3. ROBERT DAHL: ‘Political Science is the Scientific study of importance of power, Authority and influence.”

Scope of Political Science: The scope of Political Science means the subject matter covered by it Or the topics which are included in its study. It may be explained in the following ways:
i) Study of man in relation to the Society State and Government: Political science is concerned with the perennial and central issue of establishing proper relationships among state, society and government with individual. Aristotle stated that Man is a social and political animal as well.

ii) Study of State: Political science explains the origin, evolution and purpose of the state and its intimate, relationship between the state and the citizens. It explains the various theoties of the origin of the state and it also studies the nature, functions and various theories of the state.

iii) Study of the Government: Scope of political science in cludes the study of government Political science explains the relationship between state and government. The state realises it’s aims and objectives through the government. Government formulates vaneous policies, programs and their implementation for well being of the people. Political science also studies various forms and structures of the government and their merits and demerits.

iv) Study of Associations and Institutions: There are several associations and institutions which influence the life of the individual. Political science studies various associations, institutions and their relationship with the state. Political science explains structure, nature and functions of the various associations and institutions. It also studies voluntary organisations and their role in the political processes.

v) Study of Rights and Duties: Scope of Political Science includes the study or rights and duties of citizens. In recent times, issues relating to civil rights, human rights and civil society got significance in the study of political science.

vi) Studies of National and International Issues: The scope of Political science comprises the study of national and international affairs. The political science deals with the matters relating to nation-state, territorial integrity and its sovereignty. It also studies international aspects like armaments and disarmaments, balance of power, defense and security studies. It also covers international law, international organisations etc.,

vii) Study of Comparative Government and Polities: The importance of the comparative study of government and politics has been increased in recent times. Political science covers the study of various world governments, their structures and functions. It studies the relationship among the different political systems in the world.

viii) Study of Modern Political Analysis: The 20th century political science is regarded as a study of sharing and shaping of power. and it’s execution in a day-to-day political process. Political science studies the modern concepts like, political socialisation, political participation, political development, political culture, and political communication.

ix) Study of Public Policies: Modern political scientists like David Easton, Gabriel A. Almond, Charles Merriam argued that ‘political science is a policy science’. They considered political science as the study of formulation, execution, and evaluation of public policies.

They also emphasised the study of political parties, pressure groups, mass media and organs of the governments and their influence in the formulation of the public policies. It also studies major policies like Agriculture policy, Industrial policy, Environmental policy, Reservation policy and Education policy, etc.

Thus, the scope of political science has, in recent times, extended to the study of above concepts and theories and has become one of the most relevant social sciences.

TS Inter 1st Year Political Science Model Paper Set 1 with Solutions

Question 2.
What is State and explain its essential elements.
Answer:
Introduction: State is an important political organisation. The study of political science begins and ends with the state. The term state for the first time, was used by an Italian political thinker, Machiavelli in his famous book ‘The Prince” in 16th century.

Meaning: The word state is derived from a Tuetonic word “status” which means political organisation.
Definitions:

  1. ‘State is a people organised for law thin a definite Terntory-Woodrow Wilson.
  2. ‘State is a politically organised people of a definite Terntory – Bluntschlli.
  3. ‘State is a territorial society divided into government and subjects claiming within its allotted physical area, a supremacy over all other institutions. – Harold. J. Laski.

Essential elements of state: State is the predominant and superior politico-social institution existing in the society. It consists of 4 essential elements. These elements of state may be explained in a detailed way in the following paragraphs.

  1. Population: Population is the fundamental and essential element of state. There can be no state without population. Plato, Aristotle, Rousseau and others considered this feature as an important one. The famous poet Sri. Gurajada Apparao also states that it is the people, rather than the land, that comprise the state. Political writers differ in their opinions regarding the exact size of population possessed by the state. While Plato fixed 5,040, Rousseau fixed 10,000 to be an ideal population for a state. But today we can find the countries like China and India which have more than 100 crores of population on one hand and the countries like Andora, San Marino are having small number of people, on the other hand, is the modern world.

2. Territory: Territory is another essential element of the state. It is necessary for the origin and existence of the state. There can be no state without territory. Every state must have more or less territory of its own. There is no unanimous opinion among the political writers regarding the size of territory of the state. Some preferred vast territory whereas others preferred small territory But today we can find the countries like America and Canada having large territory on one hand, and the countries like Vatican, Monaco having very less territory on the other hand in the modern world.

3. Government: Government is the third essential element of the state. There can be no state without government. State enforces its authority through the government Government consists of 3 organs namely
Legislature – which makes laws
Executive – which implements laws and
Judiciary – which interprets laws.
Government are of different kinds namely, Unitary; Federal, Parliamentary and Presidential governments. Governments are at different levels like Local, State level and National level.

4. Sovereignty: Sovereignty is the most essential element of the state. It is spirit and soul of the state. There can be no state without sovereignty; It distinguishes the state from other associations and institutions. Sovereignty is the supreme political power of the state over citizens and subjects.

5. International recognition: This is another and recent element of the state. In Modem times many nations bave grown and many International organizations have come into being. Therefore some scholars have argued that International Recognition has become an essential element of state.

It should also be recognized as a state by other states. Every state requires recognition of other sovereign states. Such recognition is rendered by some International organizations like the united Nations Organisation. The UNO membership is a means for recognising state’s sovereignty. Whenever a new state comes into existence, it’s recognition by other states and by UNO is considered as very essential.

Question 3.
What is Liberty and elucidate its safeguards?
Answer:
Introduction: The concept of Liberty is of great significance in the study of political science. Liberty is an essential condition without which man cannot develop his personality. It became a source of inspiration to the millions of the people living all over the world.

Meaning: The term liberty is derived from the Latin word ‘ LIBER which means free from restrictions.
Definition:

  1. “Liberty means the absence of restraints”. – J.R. Seeley
  2. “Liberty means the positive power of doing or enjoying

something worth doing or enjoying”. – T.H. Green Safeguards of liberty: Liberty’s the most cherished ideal of human beings. Hence, it must be safeguarded in the larger interest of the society and state. In this context, the following safeguards of liberty are worth mentioning.

1. Democratic rule: Democratic rule is considered as a heaven to liberty. Liberty flourishes only in a democratic state. The reason is that democratic state extends protection to individual’s liberties through various laws. It creates a conducive atmosphere for the individuals to enjoy their liberties freely and impartially. It makes the people to participate in the government process directly or indirectly.

It makes the people to participate in the governmental process directly or indirectly. It makes the government answerable to the people. It allows the people the right to change the government through public opinion or ballot when the government acts improperly.

2. Written and rigid constitution: A written and rigid constitution is considered the most important safeguard of individual liberty. Such a constitution incorporates the various freedoms of individuals in several provisions. It acts as a custodian of peoples rights and liberties. It demarcates the spheres of governmental activity.

It mentions about the various measures to be taken in case of people’s freedoms are infringed or confiscated by others including governmental authorities. It also .imposes restraints on the political parties by not allowing them to amend the constitutional provisions for furthering their partisan interests.

3. Independent judiciary: An independent and impartial judiciary is another safeguard of individual liberty. The judiciary will uphold the constitution and keeps the government accountable to the people. It prescribes various safeguards for protecting the fundamental rights of citizens. The judges in higher courts will deliver justice to the people on. fair, free, and impartial manner. Prof. Laski, while recognizing this safeguard, stated that good governance depends upon the effective functioning of judiciary.

4. Rule of law: Rule of law is another safeguard of liberty. It is prevalent in many states like Britan, India, United States etc. Rule of law safeguards individual liberties in three ways. Firstly, it treats all individuals as equal. Secondly, it makes arrangement for the application and enforcement of uniform laws throughout the state. Thirdly, it exercises restraints on the executive against the use of arbitrary powers.

5. Fundamental rights: Provision of fundamental rights will safeguard rights to a great extent. Citizens enjoy their liberties without restraints when these rights are enshrined in the constitution. Fundamental rights enable the citizens to develop their talents and realise their personality in various walks of life.

6. Economic equality: Economic equality too acts as an important safeguard of individual liberties. It implies provision of adequate conditions for the people to come out of the evil effects of hunger poverty and unemployment etc. Liberty becomes real when there exists economic equality Economic equality presupposes economic justice. It is guaranteed by the state. Absence of glaring inequalities is a precondition of safeguarding liberty.

7. Decentralization of powers: Liberty will be better safeguard through decentralisation of powers. lidividuals could enjoy their liberties when the country is free from the centralization of governmental powers and authority. When the powers of the government are allocated among the union, state, and local governments, there arises no scope for despotism and infringement of individual liberties.

8. Freedom of press: Some regarded freedom of press as a safeguard of individual liberty. Individuals could enjoy their liberties when the various agencies of press and other media have autonomy in their functioning. The press will be able to serve as an important agent for creating, consolidating, and expressing public opinion. It, through its impartial editorials and honest presentation of news and views, will be able to safeguard individual liberties.

9. Strong opposition: A strong opposition is a necessary condition for promoting individual liberty The opposition will act as a watchdog of individual liberty. Whenever the party in power or persons at higher levels of government try to subvert or circumscribe the freedoms of individuals by their oppressive and despotic acts and activities through legislation and execution, the opposition will strongly resist such attempts. It, by moving a no-confidence motion in the last resort, will uphold the liberties of the individuals.

10. Eternal vigilance: The best safeguard to liberty is the spirit of the people. It is rightly said, “Eternal vigi]ance is the price of liberty’. People must be ready to fight for their liberty They should have the courage even to rebel against the government whenever their liberty is curbed by it. In the words of Laski, “It is the proud spirit of the citizens, that is their most real safeguard”.

Question 4.
What is Socialism? Discuss its main principles.
Answer:
Socialism is considered as the most important theory in political science. It was advocated and popularised to oppose, the defects in individualism and capitalism.The term socialism is derived from the worked ‘Socio’ which means society It was used for the first time In 1833. It was first enunciated by Robert Owen and Saint Simon Later on it was developed by Reyband, Louis Blanc, and Proudhon. It was explained on scientific basis by Karl Max in 1848.

Definitions: The term socialism is defined by many writers in many ways. Some of them are as follows.

  1. Robert Bland: ‘Socialism is a system which keeps all the factors of production and exchange in social control and sees that they belong to all equally’.
  2. Bertr and Russel: “Socialism is the adovcacy of common ownership of land and property”.
  3. George Bernard Shah: “Socialism means equality of incomes and nothing else”.

Main Principles: The following are some of the main principles of socialism.
1. Importance to Society: Socialism assigns greater importance to society rather than the individual. It emphasised that individuals interests are subordinate to those of society. It, also gave importance to the production of those commodities which are essential for common people. It is not guided by the profit motive of a few wealthy persons. It considers the production of luxurious commodities as waste and superfluous. Lastly, it assigns importance to cooperative services motto than profit motive in productive operations.

2. Organic unity of Society: Socialism regards that labourers in capitalist society do not enjoy liberties and freedom. It suggests adequate opportunities to common men for encouraging them to involve in the process of productoin. It points out that only a few persons enjoyed more liberty in a society dominated by inequalities. It is not proper to keep the majority of the poor people without liberties and freedoms. Socialism stands for a society where there prevails no inhabitations on individuals and where everyone is granted basic facilities.

3. Abolition of capitalism: Socialism desires for the abolition of capitalism. The socialists felt that the labours are exploited in the capitalist society. The capitalists aim at acquiring, more profits and more acquisition of capital. They do not favour the provision of salaries, allowances, and other facilities as determined by law to the labourers.

The state shows favour to the capitalist sections. This makes the position of the labourers miserable. Hence the socialists strongly believed that it is through the abolition of capitalism that the interests of labourers will be safeguarded. They pointed out that capitalist system should be dissolved for regulating the unproductive expenditure, for just distribution of the wealth, and for promoting the interests of the labourers.

4. Abolition of competition: Socialism advocates the abolition of competition in economic affairs that too especially in productive matters. It stands for co-operation in the place of competition. It states that competition leads to certain evils like corruption, monopolies, illegal acts, deterioration of values etc. It also results in excessive or underproduction thereby causing great sufferings to the common men.

That is why the socialists felt that co-operation, instead of competition, should be encouraged at local, provincial and national levels in the economy.

5. Belief in Equality: Socialism is based on the principle of equality. Even though it did not support absolute equality, it suggested for the prevalence of relative equality among individuals. It recognised the fact that certain elements like ment, outstanding efficiency, talent, skill etc. make differences among the individuals. It pointed out that the long standing excessive inequalities among men must be wiped out and a new society must be formed.

6. Opposition to private property: Socialism opposes individual ownership and control over lands, buildings, factories and other productive means. It suggested that productive means should not be utilised for selfish personal needs and benefits. It enunciated that no person created land and other things and all are the gifts of nature and no one can change their quantity The factors of production must’ be utlised for collective welfare. The socialists advocated for keeping all the factors of production under the control of the society.

7. Social ownership of material factors: Socialism believes that all materialist factors must be kept under the control of the society. For that purpose it suggests for their nationalisation. It treats private property as the possession by thieves. In order to avoid the irregularities and flow of private property, Socialism strongly desired for social ownership of factories, industries, mines etc.,

8. Centralised Planning System: Socialism considers that centralised planning system is essential for the progress ofthe nation. It suggests planning as the best means for achieving rapid economic development.

TS Inter 1st Year Political Science Model Paper Set 1 with Solutions

Question 5.
Explain in detail the powers and functions of the legislature.
Answer:
Legislature may be Bicameral or Unicameral. In Bicameralism there are two houses, namely, lower and upper house or first chamber and second chamber. The lower house represents the will of the people. The upper house represents the interests of the states. Exs in India Rajyasabha is the upper house and Loksabha is the lower house.

The members of the upper house are elected from the component states. These members represent their respective states in the central government. The people directly elect the members of the lower house through universal adult franchise.

Functions of the legislature: The basic function of the legislature is to enact laws as per the will of the people. The legislature in modern times is discharging certain administrative and judicial duties besides forming the laws. The functions of the legislature also depend upon the form of the government. The role of the legislature is limited a presidential form of government. But, it has in extensive role in the parliamentary government. The functions fo the legislature can be analysed under different heads like: Legislative functions, Executive functions, Financial functions, JudiciaL functions, Constitutional functions and other functions.

1. Legislative functions: To frame the laws is the basic function of the legislature. It is the important duty of the legislature to pass laws as per the will of the people. The legislature has powers to frame new laws and also to change, revise or cancel the laws which are outdated. The legislature not only makes laws but also conducts detailed discussions and consultations on different subjects. The legislature discusses extensively every bill before it becomes law.

2. Functions to control executive: In the parliamentary government, the council of ministers is responsible to the legislature. The legislature exercises control on the council of ministers in dealing with the problems cropping up in the country, and their solutions through different resolutions and questions. It can also pull down the government through a no-confidence motion if such a need arises.

3. Financial functions: The financial functions of the legislature are of much importance in democratic countries. The legislature has to approve the income and expenditure under different heads shown in the budget. The legislatuie can decide the amount of expenditure under different heads. Levying new taxes or abolishing already existing taxes can be taken up with the consent of the legislature only.

4. Judicial functions: The Legislature, especially the upper house performs some judicial functions. The House of Lords in England functions as the highest court of justice. In U.S.A. and India, the legislature has to try impeachment cases against the president or the Justices of Supreme Court and High Courts. Sometimes the legislature conducts enquiry through separate committees on the allegations against the government. The legislature has the authority to punish anybody who commits breach of privileges of members and those who violate the rules of the house.

5. Constitutional functions: The legislature has the right to amend the constitution besides framing the laws. It can amend the constitution as per the needs and requirements of the country which change from time to time.

6. Other functions: Besides the above functions, the legislature has some more functions like, keeping or rejecting the ordinances declared by the government; electing the speaker, appointing necessary committees to enquire into the lapses of the government a – to formulate rules and procedures of the legislative business.

Section – B
8 x 5 = 40 Marks

Note: Answer any EIGHT of the following questions in not exceeding 20 lines each. Each question carries 5 Marks.

Question 1.
Discuss the relationship between Political Science and History.
Answer:
Political Science has intimate relation with other social sciences like History and Economics. Such interrelation between political science and History as well as relation between political science and Economics can be detailed as below.

a. Political Science – History
b. Political Science – Economics.

a. Political Science – History: History describes the past. The development of man-kind and society can be known through History. History being the story of man, functions as a treasure house of human experiences. It is like a laboratory to all social sciences. The political, economic, social, cultural, religious and literary activity of man can be known only through History.

It describes different associations of man from earliest times. History conveys information to the present society, the developments in the past in the areas like state, civilization, culture, religion and economic activity. History is a written record of different events, movements, their causes and inter-relations. History provides information to study the political activity inthe past. The birth and development of political ideas and institutions is known through history.

“History without Political Science has no fruit.
Political Science without History has no root”

There has been continuous transformation and development of political institutions since the earliest period of History The evolution of different political institutions through the ages is recorded in History History is the foundation of Political Science. A comparative study of the previous political institutions and the contemporary political activity provides a scope to find ideal and stable political institutions in future.

The knowledge of political activity is very much essential to understand the events like founding of the Indian National Congress. the French Revolution: the Russian Revolution and the theories like the Nation Theory of the Muslim League and also to estimate their impact.

In the same way it is possible to study the concepts proposed by Plato, Aristotle and other Philosophers in the light of the knowledge of history of Ancient Europe. Different pohtical thinkers like Machia velli, Montesquieu and Lord Bryce developed their respective theories basing on the informatión found in history. As Robbern opined, it is essential for a student to know about the history of his own race to study the constitution and foreign policy of his race. The knowledge of Political Science is essential to history in the same way as the knowledge of History to Political Science. History and Political Science can contribute for the development of any civilised society in the spirit of mutual co-operation.

Question 2.
What are the different kinds of Sovereignty?
Answer:
Introduction: Sovereignty is the most important characteric of the modern state. The state is distinguished from other Associations or Institutions only by sovereign power.

Definition:
Willoughby: Sovereignty is the supreme will of the state’.
Jean Bodtn: ‘Sovereignty is the supreme power of the state over citizens and subjects unrestrained by law”.
Kinds of Sovereignty: Sovereignty Is classified into several kinds as mentioned below;

  1. Nominal sovereignty
  2. Real sovereignty
  3. Legal sovereignty
  4. Political sovereignty
  5. popular sovereignty
  6. Dejure sovereignty
  7. Defacto sovereignty

1. Nominal sovereignty: Nominal sovereignty is also known as titular sovereignty. Nomial sovereignty implies possession of sovereign powers only in name. The Queen in Britain, the Emperor of Japan, the President of India etc., are sorne examples of nominal sovereignty.

2. Real sovereignty: The real sovereign actually possesses the sovereign power. He discharges such authority on behalf of the nominal sovereign head. Accordingly the nominal sovetgn person exercises his powers only on the advice of the real sovereign persons or body of persons. The Prime Ministers of England and india etc., are some best examples of this kind of sovereignty

3. Legal sovereignty: Legal Sovereignty denotes the supremacy in terms of formal law. Only the legal sovereign is competent to issue the highest orders. It transcends even the divine law or the normal laws and the public opinion. The court of law recognizes only the legal sovereign and accepts its orders. The best example of legal sovereign is the king-in-parliament in England. The legal sovereignty is definite, deliberate, decisive and precisely known. The president of India is also an example of legal sovereignty.

4. Political sovereignty: Behind the legal sovereignty there is the political sovereignty. In a democratic state while the legal sovereign is the supreme law making power; there is behind it another sovereign whose will is the ultimate and final source of authority.

Garner observes: “Behind the legal sovereign, however, is another power, legally unknown, unorganized, and incapable of expressing the will of the state in the form of legal command, yet with a power to those mandates the legal sovereign will in practice bow and whose will must ultimately prevail in the state.

5. Popular sovereignty: The concept of popular sovereignty is different from that of political sovereignty It means that sovereignty ultimately resides in the people. The doctrine of popular sovereignty is the product of the sixteenth and seventeenth centuries. It emerged as an expression of resentment of the people against the despotic authority of the kings and their reliance on the theory of Divine Right.

Popular sovereignty attributes ultimate sovereignty to the people. This theory, first hinted by the John Locke, was later expounded by Rousseau and it becomes the slogan of the French Revolution.

6. Dejure sovereignty: The terni-De Jure’ denotes authority exercised according to law. De Jure sovereignty is the power possessed and exercised by a legally competent authority It issues orders and enjoys command overall persons, institutions and organizations in the state. The Queen in Britain and the President in India are some examples of the De Jure sovereignty.

7. De facto sovereignty: The French term De facto implies ‘real’. De facto sovereign is a person or a body of persons who exercise such authority in the last resort and at the final stage. De facto sovereign may not be a legal sovereign. His authority is based not on law but on physical force. De facto sovereign may be a king, dictator or religious priest.

Question 3.
Define Nationality. What are the essential elements of Nation ality?
Answer:
Introduction: The concepts of Nation and Nationality have become important components in the domain of International Relations and political science respectively. Both inspired the people of several countries with patriotic feeling prior to the two world wars.

The Events that took place in the erstwhile Soviet Union, ethnic Riots between serbians and Croatians in the former Yugoslavia, the unification of East and west Germanies, the peace talks between Israel and Palestine Liberation Organization (PLO) on West Asia etc., reflect the serious concern of the people for realising. Nationality and.

Nation States.
Meaning: The Word “Nation” is derived from a latin word NATTO” which means “BORN” (BIRTH) or “Common Descent”.

Definitions:

  1. R.G Gettle: “Nationality is a population having the common bonds of Race, Language, Religion, Traditions and History.
  2. J.H. Rose: ‘Nationality is a union of Hearts once made and never unmade”.
  3. J.W. Garner: “Nationality is a grout or pottion of population which is united by Racial and other bonds.

Essential Elements of Nationality:
1. Purity of Race: Racial purity helps in the formation and strengthening of the idea of Nationality. Race is a physical phenomenon. It depends on certain distinctions of skull, stature, hair, complexion etc. These distinctions serve as a cementing bond among the members of a group.

But we should remember that common race is not an indispensable factor in the growth of Nationality Modern races are so mixed that none of them can claim to be pure. Pura races have disappeared because of wars and migrations. Racial purity is now a myth only.

Ex: Canada and United states have transformed into single nations inspite of their racial diversities in their respective populations, Similarly, Australia and Britain are two distinct Nations although they belong to one racial stock.

2. Common Language: Language plays a key role in the promotion of nationality. The philosophers and scientists said that common language is essential for the development of nationality. Language is a medium to express all their feelings. It helps to express ones ourselves to have cordial relations and to share the miseries and happiness in a group languages also promotes common feelings and traditions. Common language promotes the feeling of oneness and keeps the entire race on single track.

3. Common Religion: Religion is one important factor to strengthen nationality. There are many instances when people of different nationalities with common religion remain citizens in the same state. For instance, the main reason for the partition of Indian subcontinent into India and Pakistan in 1947 lies in the religion.

4. Geographical Unity: Geographical unity is necessary for the emergence of nationality. Nationality sentiments prevail and develop among the people living in a single. geographipical area. The people residing in such an area love, worship their country and make sacrifices for the sake of their motherland. People, who belong to one religion, converse the same language, same race living in a geographical area inculcate and improve their nationality sentiments. The formation of Israel in 1946 was purely due to the feelings of the hitherto wandering Jewish people to live ina single geographical area. Hence their desire of live in a territory made them united. This ultimately transformed them as patriotic persons.

5. Common History: Common History is considered as an important element of Nationality. It invokes an inspiration among the people and binds them together. Some historical incidents may give a chance to the people to develop national sentiments. Ex: Indians have learnt the lessons of Nationalism from the British legacy.

6. Common Culture: Culture in its broad sense means a way of life. It is reflected through certain common elements like dress, customs, conventions, food habits, religious beliefs, ethical values etc. They easily develop into a single Nation. These elements bind the people together and hold together.

7. Common Political aspirations: Nationality sentiments prevail and develop among the people having common political aspirations. The political ideas, conventions and institutions which wereformed due to the single political rule will have a considerable I’m pact and influence òver the people. For instance, the Swiss people love very much their direct democratic devices in political matters.

Similarly the Americans express the feeling of worship towards their constitution. The British people also feel proud of their political and judicial institutions like rule of law, parliamentary democracy and judicial review etc.

8. Common Economicties .This element of nationality has been stressed by ‘Karl Marx Since then onwards the importance of this element has been increasing. The Russians have great regard for their economic system, even though there exist diversities. Their unflinching love for socialism inspired nationalism among them. They successfully repulsed the attacks of Germany during the Second World War. Thus the common economic ties made them united and integrated them into a nation.

TS Inter 1st Year Political Science Model Paper Set 1 with Solutions

Question 4.
What is Justice and explain different types of Justice.
Answer:
1. Natural Justice: Natural Justice is based on the notion that every person in the world possesses some rights for availing the natural resources. Natural resources provide support to the life of each and every creature on Earth. As the human beings are the only rational creatures, it is their responsibility to see that natural resources have to be judiciously exploited. Human beings must keep in mind the requirements of future generations in this regard.

2.’ Social Justice: Social Justice envisages a balance between rights of individuals and social control. It facilitates the fulfillment of the legitimate expectations of the individuals under the existing laws. It ensures several benefits and extends protection to the individuals against the interference or encroachment from others in society. It is consistent with the unity and the integrity of the nation. It fulfills the needs of the society.

Social Justice enforces the principle of equality before law. It also ensures eradication of social evils like poverty unemployment, starvation, disease etc. It also extends protection to the downtrodden and weaker sections of society. Ultimately it provides those conditions essential for the all round development of individuals.

3. Political Justice: Political Justice symbolises political equality. It implies provision of political rights to all the adult citizens in a state. It facilitates free and fair participation of the citizens in the governance of the country. It is manifested to the full extent in times of elections. It allows the citizens for their active participation in day to-day administration. It is based on the premise that everyone is counted as one and none for more than one. It may be noted that political justice prevails in the State when the following conditions are prevalent

  1. Rule of law
  2. Independent Judiciary
  3. Popular elections to the representative bodies.
  4. Political parties.
  5. Freedom of press and assembly
  6. Democratic rule etc.

Question 5.
Write a brief note on criticism of Marxism.
Answer:
Criticism of Marxism:
Marxism has been severely criticized on account of several limitations as explained below.
1. In his historical materialism, Marx classified history under four periods. Such a classification of historical development was quite arbitrary. History is illustrative which could be traced to many distinctive stages differing from one another. So Marxian analysis of history is not acceptable to many.

2. Marx contended that progress of society is marked by dia lectical process comprising thesis, anti-thesis and synthesis. He developed his theory on the contention of establishing a classless society In which all contradictions disappear Critics argued that dialectical materialism is a never-ending phenomenon.

3. Marx contended that only the changes in the productive forces will bring in changes in religion, polity, judicature etc. Critics argued that such changes are not automatic.

4. Marx argued that changes take place both in societies and world history through dialectics. He underestimated the role of several eminent philosophers like Socrates, Plato, Aristotle, Locke, and Rousseau. He emphasized on materialism and material foundations in the production sphere. His proposition does not carry validity.

5. Marxism was largely based on the capitalist system and industrially advanced states. Though it has no relevance the the agricultural and under-developed economies, it- was established in underdeveloped Russia with many perversions and with much bloodshed.

6. Marx emphasized only the economic factors as the main sources of conflicts and contradictions in the society. On the contrary, critics pointed out that conflicts in a state or society are not always characterized by economic and material forces. They contended that revolutions also emerge due to the conflicts in religious, cultural, spiritual and material forces.

7. Theory of Surplus Value as envisaged by Marx laid more emphasis on the aspect of labour. Marx forgot the fact that production is the result of combined efforts of the owners of factors of production besides labour. Labóur is one of the factors and not the only one factor in the production sphere. Value of goods is influenced and determined by several factors like land, labour, capital and organization.

8. Marx advocated that proletariat dictatorship would be termporary phenomenon and would eventually lead to the establishment of classless society and withering away of the state. His thesis proved to be a myth.

Question 6.
What are the Fundamental Duties mentioned in the Indian Constitution?
Answer:
There is a close relationship between Rights and Duties. The two are considered as the two sides of a same coin. Rights are incomplete in the absence of Duties. Rights imply Duties and Duties are entitled to rights. The two are inseparable. If the state gives the right to life to citizen it also imposes an obligation on him to not to expose his life to dangers, as well as to respect the life of others. A right is not just a law that allows individuals or governing bodies to door say anything they wish. The primary difference between rights and duties is that right is based on privilege granted to an individual, whereas duty is based on accountability of performing that duty by an Individual.

Duties:
Definition: It is the responsibilities or obligations of an individual, that are required to be done by the said individual.

Law: The duties of a citizen cannot be challenged by the court.
Basis: It is based on accountability of performing duties by an individual.

Question 7.
What are the methods of acquiring Citizenship?
Answer:
Introduction: Citizenship is a privilege of individual residing in democratic states. People fed that citizenship enables them to lead a happy, honourable and harmonious life in the state. Citizenship instills the feelings of patriotism, sacrifice, broad outlook etc., among the people.

Definitions:
Prof. Laski: “Citizenship is one’s contribution of instructed judgement to the public good”. EH. Marshall: “Citizenship is a status bestowed on those who are full members of a community. All who possess this status are equal with respect to the rights and duties with which the state is endowed’.

Methods of acquiring citizenship: There are two methods of acquiring citizenship.
They are :
i) Natural
ii) Naturalization.

The two methods may be studied as follows.
i) Natural Citizenship: Natural Citizenship is one which is acquired by the persons without specific application or request to the authorities.

It comprises three elements.
They are:

  • Blood relationship (Jus Sanguinis)
  • Soil (Jus Soli) and
  • Mixed principle.

i) Jus Sangulnis – (Kinship or Blood Relationship): This type of Citizenship. denotes acquiring citizenship by kinship or blood relationship. Under this method birth within the territory of a state entitles a person to have citizenship. Every person is treated as a citizen of the state where he is born. According to Jus Sanguinis, a child acquires the citizenship of the parents irrespective of its place of birth.

Here blood relationship alone determines the Citizenship. Ex: A child born to the Indian parents Will be treated as Indian citizen irrespective of its place of birth.

ii) Jus Soil (Land or Place of Birth): Jus Soil means acquisition of citizenship by the principle of place of birth. According to this method, citizenship is determined by the place of birth and not by parentage. It is the place of birth which determines citizenship.

However, this method is not more popular in modern times. It was popular in the Middle Ages when citizenship was associated with land. At present, however, this practice is observed exclusively in Argentina.

Mixed Principle: Under this method citizenship is granted by following either of the two principles bf Jus Sanguims and Jus Soil. Many states adopted both these principles. Ex: In Britain, France and United States, the above two principles are employed simultaneously. In this context there may arise duplication of citizenship. Ex: A child born to British parents in the United States becomes an american citizen according to the practice of Jus Soli. The same child becomes a citizen of Britain according to the principle of Jus Sanguinis. In such a case, the child is given option to choose one of its citizenship, after becoming a major.

ii) Naturalised Citizenship: Citizenship may also be acquired through naturalization. According to this method, an alien will become a citizen after fulfilling certain conditions. These conditions vary from state to state. Some of them may be summed up as follows.
1. Residence: An alien who resides in a state for a prescribed period automatically become its citizen. Residence in any part of the state is a must for an alien. The period of residence varies from state to state. For instance it is 5 years in Britain and United States and 10 years in France respectively.

2. Choice: The children of alien parents could receive citizenship of the state according to their option and choice.

3. Application: An alien in a state may apply for the citizenship of that state. Then the government of that state considers his application on its merits. It grants citizenship to him with or without some conditions. These prescribed conditions refer to a minimum period of residence, good moral character, financial capability and knowledge of one of the national languages. Besides, an alien must take an oath of allegiance before he assumes the citizenship of another state.

4. Fixed Assets: An alien who buys some portion of land or acquires some fixed property can acquire ëitizenship in a state.

5. Service (Public or Private): An alien who serves in the government of a state or in a private recognised enterprise could become the citizen of that state. He is entitled for such citizenship if. he serves in the public or private authorised departments. He may also be given Citizenship if the renders meritorious service in another state.

6. Marriage: An alien woman acquires citizenship of a state when she marries the citizen of that state. In some countries when a person marries an alien, Citizenship of either of the husband or wife is acquired. For instance, a British lady will acquire Indian citizenship if she marries an Indian citizen. Japanese womefl do not lose their citizenship even if they marry persons of alien states. The alien person on the other hand, acquires the citizenship of Japan if he mames a Japanese lady.

In this context it may be noted that an alien who receives the citizenship of the new state, he wifi have to forego his native citizenship. In other words, no one is allowed to have dual citizenships simultaneously.

Question 8.
Explain any four features of Democracy.
Answer:
Democracy is an important and most significant form of government. The term “Democracy’ is derived from two Greek words namely Demos’ and “Kratio’. In greek language Demos means the people and Kratio means power. Hence Democracy means power of the people.

Definitions of Democracy:
Abraham Lincoln: “Democracy is a government of the people, by the people and for the people”.
J.R. Seeley: “Democracy is a government in which everyone has a share.’
Lord Bryce: ‘Democracy is that form of government in which the ruling power of the state is vested not in a particular class but in the members of the community as a whole”.

Features of Democracy:
1. Liberty: Democratic government aims at securing maximum liberty for its citizens. It is built on the foundations of liberty and Equality. Democracy is the only government which assures liberties to the people by incorporating them in the constitution.

2. Equality: In the words of Prof. Seeley, democracy is “a government in which everyone has a share”. A democracy government gives opportunities to all for making them to take part in political matters. Every citizen has the right to vote and to contest in the election.

3. Independent Judiciary: Independent Judiciary is important principle of Democracy The higher courts in democratic states act as watch dog of the peoples freedoms. They issue sevëral writs for preserving and upholding fundamental freedoms. They serve as the custodian of peoples rights.

4. Government of the people: Democracy is government by the representatives of the people and these are elected by the adults, who are free to vote as they please without being coerced or pressurized by anyone. Democracy is the government by ballot not by bullet.

5. Majority Rule: Democracy stands for a rule of the majority with adequate safeguards to the minorities. Every state has political parties. One of the political parties comes to power by capturing the majority of seats in the legislature. This means democracy is a system of government based on the principle of majority rule.

6. Follows constitution principles: A democratic government functions strictly according to the principles of a constitution. Whether written or unwritten, this has been accepted by the people.’

7. Against to violence and revolutions: Democracy provides for a change in government according to constitutional principles and it is against any change by violent or revolutionary means.

8. Against to coercive methods: Democracy opposed to coercive methods, even if they are for the social good. A government cannot be called democratic, if it uses illegitimate coercion in the name’ of social welfare.

9. Importance to Human Rights: Democracy upholds the dignity of the human personality and gives various kinds of rights to the individual. Actually, to providing constitutional rights to the people is the fundamental principle of the democracy The constitutions of India and U.S.A had provided several rights to their people.

10. Right to speak: Democracy allows all individuals the right to speak, criticize and disagree with others constructively.

11. Encouragement to ideas: Democracy allows plurality of ideas and ideologies and stands firmly on the principles of tolerance. In the legislature there is plenty of worth full discussions occurred among the public representatives related to public issues.

12. Against imperialism and war: Democracy in the international sphere stands for the principle of self-determination and for the regulation of interstate relation on the basis of equality, justice and reason. Democracy is against aggressive nationalism, imperialism and war. Besides the above, Democracy has some more principles like Rule of law, welfare mechanism, Decentralization of powers. Judicial review etc.

TS Inter 1st Year Political Science Model Paper Set 1 with Solutions

Question 9.
What is Secularism and mention its important features?
Answer:
Secularism became a prominent philosophy in modern states in tackling the various social, economic and political issues. Secularism has been praised by many social scientists on several grounds It is viewed as a rational idea and ideal for analyzing various events that took place in and influence the modern states.

The following are some of the important merits of secularism.
1. Equality: It treats the people belonging to all religious de nominations as equal. It gives no recognition to the man made in equalities and discriminations based on caste, colour, community, region, religion, language, race etc.

2. Religious Freedom: Secularisrìi enables the individuals to enjoy their religious freedom to their full extent The state will not interface in the religious affairs of individuals. The Constitution and various laws in a secular state will provide individuals with complete freedom to embrance, profess, practice and propagate any religion as they like.

3. Law and Order: The maintenance of communal harmony became a challenging task for the state in specifying the feelings the people belonging to various religious denominations. Secularism prevents communal clashes and religious bigotry and animosities in the society. This is due to the fact that secularism ultimately promotes religious harmony among the people.

4. Rule of Law: Secularism accords recognition to the concept of Rule of Law. A state following secularism will enact laws and implements them keeping in view the interest of not a particular religious denomination but the people belonging to all religious denomiations. Similarly it makes no discrimination, between people on the gournds of religion.

5. Tolerance; Secularism preaches tolerance and kindness. It believes in universal brotherhood of man. It professes, propagates and practices the noble principles of charity, kindness, love, magnanimity non-violence etc. As a result, secularism is characterized by the preaceful co-existence of people and smooth working of the polity.

6. National Integration: Secularism serves as the best means for fostering national unity and integrity feelings among the people. It is also considered as the best cevice for achieving unity in diversity. It brings unity among the people of various religious beliefs and practices.

7. Protection to the minorities: Secularism treats all alike. It makes no discrimination between the people of majority and other sections of society At the same time it extends special facilities to the minority sections for preserving and promoting their interests against the dominance of majority religious group. It teaches the people about the significance of religious tolerance towards minority sections.

8. All-round progress: The greatest merit of secularism relates to the achievement of all round progress of the people. This is possible due to the prevalence of rule of law, religious tolerance, neufrai attitude of the government etc. Especially the government will make all efforts for the development of the people of all religious denominations in all spheres of welfare, social justice, protecting the interests of disadvantaged sectians etc.

Question 10.
What are the differences between Written and Unwritten constitutions?
Answer:
Written Constitution: A written constitution is formulated and adopted by a Constituent Assembly or a Convention. It comprises several principles and rules of the Government in a written form or document. The Constitution of India is an example of written constitution. The American Constitution is the first written constitution in the world.

Unwritten Constitution: Unwritten constitution is one whose provisions are not written in a single document. It includes several customs and traditions which are manifested in the form of the laws. The Constitution of Britain is the best example of unwritten constitution.

Differences between Written and unwritten Constitutions

Written Constitution. Un Written Constitution
1. Written constitution implies a document or few documents in which the rules customs, regulating the main institutions of Government are written down. 1. Unwritten constitution de-notes a sum of conventions and usages which have not been systematically documented.
2. All the basic principles of the State are clearly written. 2. All the basic principles of the State exist in the form of customs and traditions.
3. Written constitution is framed by a special assembly convened at a particular point of time. 3. Unwritten constitution contains some written elements also in the form of enactments of fundamental charters made from time to time.
4. It is suitable to the educated and literate people. 4. It is suitable to the uneducated and illiterate people.
5. Courts of law protect the liberties of the citizens. 5. Courts of law cannot provide much protection.
6. It is formulated at a particular time. 6. It is evolutionary in nature.
7. It provides political stability. 7. It could not ensure political stability.
8. It cannot be easily amended. 8. It can easily be amended.
9. It is useful to federal states. 9. It is advantageous to the unitary states.

Question 11.
Write a brief note on Judicial Review.
Answer:
Judicial Review means reviewing the laws made by the legislatures, The laws of the legislatures (parliament, assemblies, Councils, Parishades etc.,) must not be unconstitutional. The courts of law can nullify all laws which are unconstitutional. This is the Judicial Review.

The Judicial Review is not limited only to laws. It is applicable to all the activities of the executive at different levels of central, state and local government institutions. Every State will have a constitution. It is its basic law and all the laws must be within the limits of the constitution. The Constitution provides the courts of law with the right to see that all laws and activities conform to the Constitution. Judicial Review is not found in all the countries.

The Judicial Review first originated in United States of America in the context of the judgement given by the Supreme Court in the Marbury-Madison dispute in the beginning of the nineteenth century. The courts of law follow different theories, principles and views in reviewing the constitutional viability of the laws. The Judicial Review is based on the following theories which can be studied in detail in higher classes.

  1. Theory of the efficiency of law.
  2. Theory of separation of powers.
  3. Theory of constitutional dynamism.
  4. Theory of activity
  5. Theory of decision by experience.
  6. Theory of constitutional development.

Question 12.
What are the important features of Federal Government?
Answer:
Features of Federal Government:
a) Written Constitution: For a federal government the constitution must almost necessarily be a written constitution which deter. mines the relation between the central and provincial/ regional governments.

b) Rigid Constitution: The natural,Gorollary of the supremacy of the constitution, and it being a written document, is that it should not be altered either by the central Legislatures or by Regional Legislatures under their ordinary law- making procedure.

c) Devision of powers: In a federal political system, there is an essential feature of distribution of powers between central and regional governments under the constitution. Major sectors are vested with unionl central and National and Provincial important things are vested with regional governments. For example, external affairs, exports and ports and education, agriculture health are exercised by union and state governments respectively.

d) Bicameralism Bicameralism is another important characteristic of the federal government. In federal governments there should be Two – Chambers, representing people and states such as House of people and House states respectively.

e) Dual citizenship: In a federal political system, constitution provides for dual citizenship to the citizens. Accordingly, the citizens have membership in both the centre and states simultaneously.

f) Independent Judiciary: In a federal political system, independence of judiciary and intensity of judicial system is another important feature. Independence of judiciary will safeguard the minimum rights of people against the acts of Legislature and administrative authorities.

Section – C
15 × 2 =30 Marks

Note: Answer any FIFTEEN of the following questions in not exceeding 5 lines each. Each question carries 2 Marks.

Question 1.
Sociology
Answer:
Sociology is the root of all social sciences. Sociology studies the changes in different social institutions. It discusses the social, ethical, economic and cultural systems in a society. It also studies the human relations, social conditions, origin, growth and development of different systems, their respective forms, the rules, customs and traditions.

Question 2.
Behaviouralism.
Answer:
It is a modern interdisciplinary approach in Political Science. It seeks to make political Science a real Science. It. originated in 1925 but became popular in the USA after the second World War. David Easton, Robert Dahi, Gabriel Almond are important supporters of this approach. It studies political behaviour of people by using Scientific methods of data collection. It is a protest against the Traditional Approach.

Question 3.
The differences between ‘State’ and ‘Society.
Answer:

Sate Society
1. State is a political organization. 1. Society is a social organization.
2. Where as the state has its own fixed territory and limits. 2. Society does not have definite territorial boundaries.
3. The state has a definite government. 3. Society does not have any legal and prescribed organization.
4. State possesses the power of Compulsion. Disobedience to its laws is followed by punishment. 4. Society does not enjoy the power compulsion. Disobedience to its principles is not followed by any physical punishment.

Question 4.
Nationalism.
Answer:
Nationalism is an effective force in modern politics. In theoretical for Mulations and practical political movements, the fire of national Sentiment has been supported by writers and politicians. The purpose of national movement has however, remained the same every where. It has sought to concentrate human allegiance into a small and homogeneous unit- the nation-state.

Question 5.
Rule of law.
Answer:
Rule of Law is an important type of adminitration of justice. It originated in England. It implies: .

  1. Legal Equality: All are equal before law.
  2. No Arbitrary Action: Punishment is given only when an existing law is violated.
  3. No Special Rights: No individual is above law and law does not recognise any special privileges.

Question 6.
Economic Equality.
Answer:
Economic equality: This kind of equality is a precondition for the enjoyment of social and political equalities. Its absence leads to several social and political problems. Economic equality does not mean equal distribution of social wealth among all the people. It only means the elimination of inequalities in wealth, income and property. The basic needs like food, shelter and clothing must be available to all.

In the opinion of Laski, “Economic equality means the abolition of unfettered and irresponsible will in the industrial world’. In Barkers view, “Economic equality is partly a matter of status and partly a matter of property and income”.

Question 7.
Power and Authority.
Answer:
Power denotes the ability of a person to fulfill his desires or to achieve his objectives, power is a relationship in which one person or group is able to determine the actions of another in the direction of the former’s ends. H.V. Wiseman defined power as “the ability to get one’s wishes carried out despite resistance”.

Authority:
The concept of authority is closely linked with the concept of power. Authority means legitimate power. Hamuel Arendt Portrays authority as power based on consent. Following are the definitions of authority.

  • ‘Authority is the capacity to justify by a process of reasoning what is desired from the point of view of man” – Fredrick
  • “Legitimate power is often called Authority”. – Robert.A.Dahl

Question 8.
Neo-liberalism.
Answer:
Neo-liberalism or,libertarìanism stands for the contemporary version of classical liberalism which seeks to restore ‘Laissez Faire individualism. It denounces the welfare State; opposes state intervention and control of economic activities. Champions of neo-liberalism stand for rolling back’ the state which has immensely expanded its sphere of activities. The chief exponents of neo-liberalism include F.A. Fayek (1899 – 1992), an Australian thinker, Milton Freedman (1912 – 2006), an American economist, and Robert Nozick (1938 – 2002), an American philosopher.

Question 9.
Non-violence.
Answer:
The literal meaning of Non-violence is “Not doing Violence”. In a Nutshell, it means ‘Not to Kill”, “Not to do harm” is its wider meaning. Mahatma Gandhi used Non-violence as a potent weapon not only against the British colonialism but also in the movements waged against all types of deeply entrenched evils of the society. Truth and fearlessness are the essential conditions of Non-Violence Gandhi regarded and equated non-violence on par with self-torture of the soul, Mercy, Love, Fearlessness, Kindness, and non-indulgence

Question 10.
Surplus Value.
Answer:
Surplus Value: Marx developed the theory of Surplus Value based on the Ricardos theory which states that the value of every commodity is proportional to the quantity of labour put into it. According to Marx the Surplus Value is the value that is created over and above what the labourer is paid for and he ragards it as the source of all profit. Marx felt that commodification of labour results in alienation of labourer from its product, price, fellow human being and him self due to mechanization and industrial production. Division of Labour makes man mechanical. Marx’s theory of Surplus Value is more of a theory of just price rather than a theory of price.

Question 11.
National Human Rights Commission.
Answer:
The United Nations reaffirmed that the people and governments of every state must strive for respecting individual freedoms and human rights. The concerned authorities and agencies of the United Nations held several international conferences and invited inter-nationally acclaimed intellectuals, jurists and heads of states for eliciting the valuable opinions on extending human rights to every section of human communities throughout the world.

Question 12.
The Fundamental Rights mentioned in the Indian Constitution.
Answer:
Indian Constitution and Human Rights: The Constitution of India incorporated many of the human rights, mentioned in the U.N. Declaration of Human Rights. Especially Articles 19 to 28 of Chapter III on Fundamental Rights are in consonance with the human rights of individuals. The relate to the life, liberty, equality and dignity of individuals. Article 19 of Indian Constitution guaranteed freedoms of speech, expression, association and assembly. It also conferred freedoms of occupation, settlement, movement etc. Article 20 provided protection to the personal life of individuals.

Question 13.
Mention any four qualities of Good citizenship.
Answer:

  • Good Character: Good character is essential for a good citizen. A good citizen should be courageous, just, helpful, kind-hearted, sympathetic, truthful, and virtuous in letter and spirit.
  • Sound Health: A good citizen should have good health and strength. Healthy citizens make the nation healthy and wealthy.

Question 14.
Recall.
Answer:
Recall means To call back. The representatives will be called back by the people in case they are inefficient. Hence, this method helps the representatives in discharging their responsibilities properly for fear of being called back on the grounds of inefficiency.

Question 15.
Direct Democracy.
Answer:
Abraham Lincoln: “Democracy is a government of the people, by the people, and for the people”. J.R. Seeley: “Democracy is a government in which everyone has a share.”
Lord Bryce: “Democracy is that form of government in which the ruling power of the state is vested not in a particular class but in the members of the community as a whole”.

Question 16.
Theocratic State.
Answer:
The state having an official religion is called a theocratic state. In such state all other religions or religious activities are either prohibited or discouraged by the state all the official and important offices of the state are either appointed or elected only those from the official religion. State officially participates in the religious affairs and rituals.

Question 17.
Rigid Constitution.
Answer:
Rigid Constitution is one whose provisions can be changed easily. In this system, the constitutional amendment methods are different from those of ordinary laws. There will be a special procedure for amending the provisions of the Rigid Constitution. The Rigid Constitution will have firmness due to its special procedures of amendment.

TS Inter 1st Year Political Science Model Paper Set 1 with Solutions

Question 18.
Judicial Activism.
Answer:
According to the idea of judicial activism judges should use their powers to correct injustices, especially when the other branches a Government do not act to do so. The courts play an active role in shaping social policy on such issues as civil rights, political unfairness protection of individual rights and public morality.

Judicial activism is policy-making function of judiciary in competition with policy making by the legislative and executive. This element is associated with the doctrine of judicial review. The essence of true judicial activism lies in rendering decisions by the judiciary which are in tune with the temper and tempo of the times.

Behind every judicial decision, judicial activism and judicial restraint are the two aspects that describe the philosophy and motivation. The concept of judicial activism is the polar opposite of judicial restraint. Judicial activism refers to a theory of judgment that takes into account the spirit of the law and the changing times, whereas judicial restraint relies on a strict interpretation of the law and the importance legal precedent.

Judicial activism is a dynamic process of judicial outlook in a changing society Arthur Schlesinger Jr. Woduced the term Judicial activism’ in 1947. According to Black’s Law Dictionary, “Judicial activism is a judicial philosophy which motivates judges to depart from traditional precedents in favour of progressive and new social policies”.

Question 19.
Plural Executive.
Answer:
Among various types of Executive, Plural Executive is one. If the executive powers are exercised by a body of persons having coequal authority, it is called “Plural Executive. The Swiss Federal Council and the President in the former U.S.S.R. are the examples of Plural Executives. This Plural Executive combines the merits of both the parliamentary and the presidential executives, avoiding their defects.

Question 20.
Prime Minister.
Answer:
Parliamentary government is described as Prime Ministerial government. The Prime Minister in this system acts as the Real executive head of the government. He acts as the Leader of the Majority party or coalition Ministry in the lower house of the legislature. He remains as the main pillar to the structure of union cabinet and Union Council of Ministers. He is central to the formation, continuance and survival of the Ministry. He Presides over the meetings of the union cabinet and decides its agenda. He enforces the Principle of Collective Responsibility. All the Ministers take oath, of office, assume powers and discharge their Public and political obligations under his stewardship.